0% found this document useful (0 votes)
58 views477 pages

Quant Practice for Exam Prep

The document contains a comprehensive collection of extra questions and solutions across various quantitative topics including Arithmetic, Statistics, Algebra, and more. Each section is organized by topic and includes multiple-choice questions designed to test knowledge and application of mathematical concepts. The document serves as a resource for practice and preparation in quantitative reasoning.
Copyright
© © All Rights Reserved
We take content rights seriously. If you suspect this is your content, claim it here.
Available Formats
Download as PDF, TXT or read online on Scribd
0% found this document useful (0 votes)
58 views477 pages

Quant Practice for Exam Prep

The document contains a comprehensive collection of extra questions and solutions across various quantitative topics including Arithmetic, Statistics, Algebra, and more. Each section is organized by topic and includes multiple-choice questions designed to test knowledge and application of mathematical concepts. The document serves as a resource for practice and preparation in quantitative reasoning.
Copyright
© © All Rights Reserved
We take content rights seriously. If you suspect this is your content, claim it here.
Available Formats
Download as PDF, TXT or read online on Scribd

Quant Basics

Topicwise Extra Questions


Table of Contents

Topic Name Page

Arithmetic Part 1 Extra Questions 3-19

Arithmetic Part 2 Extra Questions 20-37

Statistics Extra Questions 38-55

Numbers Extra Questions 56-75

Algebra Extra Questions 76-92

Coordinate Geometry & Graphs Extra Questions 93-111

PnC , Probability Extra Questions 112-128

Inequality & Modulus Extra Questions 129-144

Arithmetic Part 1 Solutions 146-185

Arithmetic Part 2 Solutions 186-226

Statistics Solutions 227-270

Numbers Solutions 271-310

Algebra Solutions 311-354

Coordinate Geometry & Graphs Solutions 355-396

PnC , Probability Solutions 397-436

Inequality & Modulus Solutions 437-477


Arithmetic Part 1 Extra Questions

PERCENTAGES

1. In a basket of fruits, 60% are mangoes and the remaining 40% are apples. 25% of the apples
are green and the rest 75% are red. Of the mangoes, 80% are red and the rest of the mangoes
are green. What percentage of the green fruits are mangoes?

A. 63.63%
B. 54.54%
C. 40%
D. 30%
E. 20%

2. In an exam, a student scored 50% of the maximum marks and yet failed by 12 marks. If he
had scored 10% more than what he scored, he would have just managed to get the pass
percentage. What are the maximum marks of the paper?

A. 100
B. 120
C. 180
D. 200
E. 240
3. Mira's expenditure and savings are in the ratio 3: 2. Her income increases by 10% and her
expenditure increases by 12%. By what percentage does her savings increase ?

A. 2%
B. 4%
C. 5%
D. 7%
E. 10%

4. Of the total amount received by Karan, 20% was spent on purchases and 5% of the remaining
on transportation. If he is left with Rs 1520 , the initial amount was ?

A. 4500
B. 3000
C. 2500
D. 2000
E. 1800

5. If prices decrease by 25%, by what percentage can consumption increase without affecting
the expenditure?

A. 25%
B. 20%
C. 15%
D. 33.33%
E. 40%
6. Since the price of mangoes decreased by 25%, I can purchase 4 more mangoes for Rs. 60.
What is the original price of one mango?

A. 5
B. 10
C. 12
D. 15
E. 20

7. If the area of the circle is increased by 21%, then what is the percentage increase in the
circumference of the circle?

A. 5%
B. 10%
C. 15%
D. 18%
E. 25%

8. The radius of a circle has increased by 20%. By what percentage does the circumference and
the area increase ?

A. 10%,10%
B. 10%,20%
C. 20%,20%
D. 40%,40%
E. 20%,44%
9. Two numbers are respectively 19% and 70% more than the third number. What percentage is
the first number out of the second ?

A. 50%
B. 60%
C. 70%
D. 80%
E. 90%

10. Salaries of A, B and C are in the ratio 1:2:3. Salary of B and C together is Rs. 6,000. By what
percentage is the salary of C more than that of A ?

A. 100%
B. 150%
C. 180%
D. 200%
E. 250%

11. In a physical measurement, by mistake, Shyam gave his height as 25% more than normal. In
the interview panel, he clarified that his actual height was 5 feet 5 inches. Find the percentage
correction made by the candidate from his stated height to his actual height.

A. 20%
B. 30%
C. 40%
D. 50%
E. 60%
12. A number is mistakenly divided by 2 instead of being multiplied by 2. Find the percentage
change in the result due to this mistake.

A. 100%
B. 125%
C. 200%
D. 75%
E. 150%

PROFIT LOSS DISCOUNT

1. A man sells 2 items for Rs. 4,000 each, neither gaining nor losing in the deal. If he sells one
cow at a gain of 25%, then the other cow must have sold at a loss of ?

A. 16.66%
B. 18.22%
C. 25%
D. 30%
E. 20%

2. A salesman sold one-fourth of his stock at 50% profit and the remaining at 80% profit. Find
the net profit % he made on his total stock.

A. 72.5%
B. 80%
C. 50%
D. 65%
E. 40%
3. Aditya purchases toffees at Rs. 10 per dozen and sells them at Rs. 12 for every 10 toffees.
Find the gain or loss percentage.

A. 20%
B. 44%
C. 50%
D. 60%
E. 75%

4. An off-season discount of x % is being offered at the discount store. An additional 12.5%


discount is given if the value of purchase is more than Rs. 500. After the discounts, a person
pays Rs. 525 for a jean whose list price is Rs. 750. Find the value of x.

A. 5
B. 10
C. 20
D. 25
E. 30

5. Inspite of giving a discount of 10%, a shopkeeper manages to make a profit of 8%. By what
percentage does the shopkeeper mark- up his goods ?

A. 5%
B. 10%
C. 15%
D. 20%
E. 30%
6. A shopkeeper gives 2 items free with every 3 items purchased. In effect, what discount
percentage is being offered ? [ Assume that all the items are identical ]

A. 20%
B. 25%
C. 40%
D. 50%
E. 66.66%

7. A man bought 80 kg of rice for Rs. 88 and sold it at a loss of as much money as he received
for 20 kg. At what price did he sell it ?

A. 60 paise
B. 70 paise
C. 75 paise
D. 80 paise
E. 88 paise

8. A person earns 15% on an investment but loses 10% on another investment. If the ratio of
the two investments be 3: 5 respectively, what is the gain or loss on the two investments taken
together ?

A. 0.0625%
B. 0.625%
C. 6.25%
D. 62.5%
E. None of the above
SI CI

1. Suhit borrowed a sum of $6300 from Vikas at the rate of 14% per annum simple interest
for 3 years . He then added some more money to the borrowed sum and lent it to Mohit at the
rate of 16% of simple interest for the same time. If Suhit gained $618 in the whole transaction,
then what sum did he lend to Mohit?

A. $6000
B. $6800
C. $7200
D. $7500
E. $7800

2. If a sum of money invested under simple interest amounts to $3,200 in 4 years and $3,800 in
6 years , what is the rate at which the sum of money was invested ?

A. 10%
B. 12%
C. 15%
D. 20%
E. 24%

3. Simple interest on a certain sum for 3 years at 8% per annum is equal to half the compound
interest on $4000 for 2 years at 10% per annum. What is the sum placed on simple interest?

A. $840
B. $1250
C. $1550
D. $1750
E. $1950
4. A sum of $3310 is to be paid back in 3 equal instalments. How much is each instalment if the
interest is compounded annually at 10% per annum ?
(CALCULATION INTENSIVE ; GOOD FOR PRACTICE)

A. $1311
B. $1331
C. $1103
D. $2331
E. $2311

5. Samantha purchased a five-year municipal bond that pays simple interest for $600. After the
bond matured five years later, she had received a total of $720 in principal plus interest
payments. What is the interest rate of Samantha's bond?

A. 4%
B. 5%
C. 10%
D. 12%
E. 20%

6. What amount, in dollars, invested for one year at an interest rate of 2 percent compounded
semiannually would produce the same final balance at the end of the year as $10,000 invested
for one year at an interest rate of 4 percent compounded quarterly?

A. $10,000
B. $10,201
C. $10,500
D. $10,801
E. $20,000
7. The simple interest for 10 years is Rs. 6,000. The compound interest for 2 years is Rs. 1,400.
Find the rate of interest and the principal.

A. 30%, 1,800
1
B. 33 3 %, 1,600
1
C. 33 3 %, 1,800
D. 35%, 1,500
2
E. 66 3 %, 1,600

8. An amount of X dollars is invested at R% simple annual interest and yields a total interest of
$100 in 2 years. In terms of X, what dollar amount invested at the same rate of interest will yield
a total of $400 in 4 years ?

𝑋
A. 2
B. X
3𝑋
C. 2
D. 2X
5𝑋
E. 2
Ratios and Mixtures

1. From a 100 ml can full of milk, 10 ml is removed and 20 ml water is added. Next 20 ml of the
solution is removed and 30 ml water is added. What is the quantity of milk in the solution now?

800
A. 11
ml
810
B. 11
ml
820
C. 11
ml
830
D. 11
ml
840
E. 11
ml

2. There are three vessels of equal capacity which are full . Vessel A contains Milk and water in
the ratio 4:3 , Vessel B contains Milk and water in the ratio 2:1 and Vessel C contains Milk and
water in the ratio 3:2 . If the mixture in all the three vessels is mixed up , what will be the ratio
of milk and water ?

A. 9:6
B. 3:2
C. 118:126
D. 193:122
E. 201:132
3. As a result of a chemical reaction between compounds A and B , 1/4 of each compound was
transformed into an equal amount of the other compound, while the remaining 3/4 of each
compound was not transformed. If the amount of B was initially 4 times the amount of A , what
was the ratio of the amount of A to the amount of B after the reaction ?

A. 1 to 3
B. 3 to 4
C. 4 to 7
D. 7 to 13
E. 7 to 16

4. In Smithtown, the ratio of right-handed people to left-handed people is 3 to 1 and the ratio of
men to women is 3 to 2. If the number of right-handed men is maximised , then what percent of
all the people in Smithtown are left-handed women?

A. 50%
B. 40%
C. 25%
D. 20%
E. 10%

5. Rice of two different varieties A and B are mixed together, by a seller, in the ratio of 4: 7.
Upon selling the mixture at the price of $17 per kg, the seller incurred a loss of 15%. If the cost
price of B is $22 per kg, what is the cost price per kg of A?

A. 15
B. 16.5
C. 17.75
D. 18
E. 19
6. The ratio of x to y to z is 3 : 4 : 12. If the ratio is changed so that the ratio of x to z is doubled
and the ratio of y to z is halved, which of the following is the new ratio?

A. 6 : 2 : 24
B. 6 : 1 : 24
C. 6 : 1 : 12
D. 3 : 1 : 6
E. 3 : 8 : 6

7. At a certain university, the ratio of the number of teaching assistants to the number of
students in any course must always be greater than 3:80. At this university , what is the
maximum number of students possible in a course that has 5 teaching assistants?

A. 130
B. 131
C. 132
D. 133
E. 134

8. If A : B = 3 : 4 , B : C = 8 : 9 , C : D = 15 : 16 , find A: B: C: D.

A. 10 : 20 : 30 : 48
B. 20 : 40 : 45 : 48
C. 30 : 40 : 45 : 64
D. 30 : 40 : 45 : 48
E. None of the above
9. A precious stone worth Rs. 6,800 is accidentally dropped and breaks into 3 pieces. The
weights of these 3 pieces are in the ratio 5:7:8. The value of the stone is proportional to the
square of its weight. Calculate the loss in the values, if any , incurred because of the breakage.

A. Rs 2344
B. Rs 3144
C. Rs 3554
D. Rs. 4454
E. Rs 5544

10. In two alloys the ratios of copper to tin are 3 : 4 and 1 : 6 respectively. If 7 kg of the first
alloy and 21 kg of the second alloy are mixed together to form a new alloy, then what will be the
ratio of copper to tin in the new alloy ?

A. 11 : 3
B. 3:8
C. 3 : 11
D. 3 : 14
E. 11 : 14
11. 5 liters of 30% alcohol solution is mixed with 10 liters of 45% alcohol solution. What is the
concentration of the resultant solution ?

A. 35%
B. 38%
C. 40%
D. 42%
E. 44%

12. How many kilograms of salt costing Rs. 42 per kilogram must a man mix with 25 kilograms of
salt costing Rs. 24 per kilogram, such that on selling the mixture at Rs. 40 per kilogram, there is
a gain of 25% on the outlay ?

A. 10 kg
B. 12 kg
C. 15 kg
D. 18 kg
E. 20 kg
ANSWER KEYS FOR REFERENCE

PERCENTAGES

Question Answer Question Answer

1 B 7 B

2 E 8 E

3 D 9 C

4 D 10 D

5 D 11 A

6 A 12 D

PROFIT LOSS DISCOUNT

Question Answer Question Answer

1 A 7 E

2 A 8 B

3 B

4 C

5 D

6 C
SI CI

Question Answer Question Answer

1 B 7 C

2 C 8 D

3 D

4 B

5 A

6 B

Ratios and Mixtures

Question Answer Question Answer

1 B 7 D

2 D 8 D

3 D 9 D

4 C 10 C

5 B 11 C

6 D 12 E
Arithmetic Part 2 Extra Questions

Time Work

1. Pump A, pumping water at a constant rate, can fill a certain swimming pool in 6 hours. Pump
B, pumping water at a constant rate, can fill the same pool in 4 hours. If both pumps begin
filling the pool simultaneously when the pool is empty and pump B breaks down 1 hour after
they begin filling the pool, how many hours will it take pump A alone to finish filling the pool ?

A. 1.4 hrs
B. 2.3 hrs
C. 3.5 hrs
D. 4 hrs
E. 4.8 hrs

2. Working alone, Printers X, Y, and Z can do a certain printing job, consisting of a large number
of pages, in 12, 15, and 18 hours, respectively. What is the ratio of the time it takes Printer X to
do the job, working alone at its rate, to the time it takes Printers Y and Z to do the job, working
together at their individual rates?

4
A. 11
1
B. 2
15
C. 22
22
D. 15
11
E. 4
3. In order to complete a reading assignment on time, Terry planned to read 90 pages per day.
However, she read only 75 pages per day at first, leaving 690 pages to be read during the last 6
days before the assignment was to be completed. How many days in all did Terry have to
complete the assignment on time?

A. 15
B. 16
C. 25
D. 40
E. 46

4. A team of 10 workers can paint the wall in 8 days. After working 2 days, x additional workers
joined the team and together, they finished the job in 4 days. What is the value of x ?
(Rate of each worker remains the same)

A. 5
B. 6
C. 12
D. 4
E. 9

5. If 20 diggers, each working at the same constant rate, can dig a 100-meter long trench in hard
soil in 3 days, how many diggers are needed to dig a 180-meter long trench in soft soil in the
same 3 days, given that a digger is 20% more productive in soft soil than in hard soil?

A. 25
B. 28
C. 29
D. 30
E. 32
6. A cistern is filled by an inlet valve in 5 hours. However because of a leak, it takes 30 minutes
more to fill up. In how much time will the leak empty the filled cistern independently ?

A. 50 hrs
B. 55 hrs
C. 60 hrs
D. 65 hrs
E. 70 hrs

7. A force of 60 men has food for 28 days. 8 days later , reinforcements arrive leaving the
number of days the food would last to 15 days. What was the strength of the reinforcement ?

A. 5
B. 10
C. 16
D. 20
E. 24

8. A and B can separately do a piece of work in 20 and 15 days respectively. They worked
together for 6 days, after which B was replaced by C. If the work was finished in the next 4 days,
then find the number of days in which C alone could do the work.

A. 20
B. 30
C. 35
D. 40
E. 45
9. Three men can complete a piece of work in 6 days. Two days after they started, 3 more men
joined them. How many days will they take to complete the remaining work ?
( Assume they all have equal efficiency )

A. 0.25
B. 0.50
C. 1
D. 2
E. 3

10. 2 men and 3 boys can do a piece of work in 10 days, while 3 men and 2 boys can do the
same work in 8 days. In how many days can 2 men and 1 boy do the work ?

A. 10
B. 11
C. 12
D. 12.5
E. 15
Time Speed Distance

1. A predator is chasing its prey. The predator takes 4 leaps for every 6 leaps of the prey and the
predator covers as much distance in 2 leaps as 3 leaps of the prey. What is the ratio of the speed
of the predator to that of the prey ?

A. 11 : 9
B. 10 : 9
C. 1:1
D. 9 : 10
E. 9 : 11

2. Airplane A flew against a headwind a distance of 900 miles at an average speed of (s - 50)
miles per hour. Airplane B flew the same route in the opposite direction with a tailwind and
traveled the same distance at an average speed of (s + 50) miles per hour. If Airplane A’s trip
took 1.5 hours longer than Airplane B’s trip, how many hours did Airplane B’s trip take?

A. 1.5
B. 2
C. 2.5
D. 3
E. 3.5

3. If a man walks at the rate of 5 kmph, he misses a train by 7 minutes. However, if he walks at
the rate of 6 kmph, he reaches the station 5 minutes before the departure of the train. Find the
distance to the station.

A. 2 km
B. 3 km
C. 4 km
D. 5 km
E. 6 km
4. A car traveled 75% of the way from town A to town B by traveling at T hours at an average
speed of V mph. The car travels at an average speed of S mph for the remaining part of the trip.
Which of the following expressions represents the average speed for the entire trip ?

A. 0.75V + 0.25S
B. 0.75T + 0.25S
𝑉𝑇
C. (3𝑆)
4𝑉𝑇
D. (𝑇+𝑆)
3
4𝑉𝑆
E. (3𝑆+𝑉)

5. Sarah is traveling from City A to City B. The distance between these two cities is 185 miles,
and she planned to arrive at City B in 2 hours and 30 minutes. Her average speed for the first 1
hour and 40 minutes was 75 miles per hour, but then she realized that she should slow down to
drive more safely. If she still wanted to arrive at her destination on time, at what speed should
she drive ?

A. 60.0 miles per hour


B. 64.0 miles per hour
C. 68.0 miles per hour
D. 70.5 miles per hour
E. 72.0 miles per hour
6. A turtle climbed to the top of a plateau at a rate of 4 miles an hour, crossed the plateau at a
2
rate of x miles per hour, and descended the other side of the plateau at a rate of 𝑥 miles per
hour. If each portion of the journey was equal in distance, what was the turtle's average speed
for the entire trip, in terms of x ?

2𝑥
A. 𝑥+2
2

B. (𝑥 + 2) 3
2
C. (𝑥 + 2)
2
4𝑥
D. 2
(𝑥+2)
2
12𝑥
E. 2
(𝑥+2)

7. Amitabh sees Uma standing at a distance of 200 m from his position. He increases his speed
by 50% and hence takes 20 s now to reach her. What was his original speed (in km/hr)?

A. 10
B. 12
C. 16
D. 24
E. 30

5
8. A train met with an accident 150 km from station A. It completed the remaining journey at 6
of the previous speed and reached 15 min late at station B. Had the accident taken place 30 km
further, it would have been only 7 min late. Find the distance between the two stations A and B.

A. 170 km
B. 180 km
C. 192.25 km
D. 200.50 km
E. 206.25 km
9. A runs 25% faster than B and allows B a lead of 7 meters. The race ends in dead heat. What is
the length of the race?

A. 10 meters
B. 15 meters
C. 25 meters
D. 35 meters
E. 45 meters

10. A and B start swimming simultaneously from opposite ends, X and Y respectively of a
swimming pool of length 200 meters. A crosses B, reaches the opposite end Y and turns back
immediately and again meets B at a distance of 120 meters from X. Find the ratio of the speeds
of A and B if B has not yet reached the opposite end even once.

A. 7:2
B. 4:1
C. 6:4
D. 5:3
E. 9:5

11. A train X starts from Meerut at 4 p.m. and reaches Ghaziabad at 5 p.m.while another train Y
starts from Ghaziabad at 4 p.m. and reaches Meerut at 5:30 p.m. The two trains will cross each
other at

A. 4:36 p.m.
B. 4:42 p.m.
C. 4:48 p.m.
D. 4:50 p.m.
E. 4:52 p.m.
Set Theory

1. According to a survey, 7 percent of teenagers have not used a computer to play games, 11
percent have not used a computer to write reports, and 95 percent have used a computer for at
least one of the above purposes. What percent of the teenagers in the survey have used a
computer both to play games and to write reports?

A. 13%
B. 56%
C. 77%
D. 87%
E. 91%

2.

In a poll, 200 subscribers to Financial Magazine X indicated which of five specific companies
they own stock in. The results are shown in the table above. If 15 of the 200 own stock in both
IBM and AT&T, how many of those polled own stock in neither of the two companies ?

A. 63
B. 93
C. 107
D. 122
E. 137
3. In a certain region, the number of children who have been vaccinated against rubella is twice
the number who have been vaccinated against mumps. The number who have been vaccinated
against both is twice the number who have been vaccinated only against mumps. If 5,000 have
been vaccinated against both, how many have been vaccinated only against rubella?

A. 2,500
B. 7,500
C. 10,000
D. 15,000
E. 17,500

4. In the city of San Durango, 60 people own cats, dogs, or rabbits. If 30 people owned cats, 40
owned dogs, 10 owned rabbits, and 12 owned exactly two of the three types of pet, how many
people owned all three?

A. 2
B. 4
C. 8
D.12
E. 32

5. In school V, all students must take at least one of Mathematics, Physics, or Biology. None of
the students took 3 classes. The number of students who took Mathematics class is 18, the
number of students who took Physics class is 23, and the number of students who took Biology
class is 20. 3 students took both Mathematics class and Physics class, 8 students took both
Physics class and Biology class, and 6 students took both Biology class and Mathematics class.
What is the total number of students in school V?

A. 11
B. 22
C. 33
D. 44
E. 55
6. Of the 400 members at a health club, 260 use the weight room and 300 use the pool. If at
least 60 of the members do not use either, then the number of members using both the weight
room and the pool must be between:

A. 40 to 100
B. 80 to 140
C. 160 to 260
D. 220 to 260
E. 220 to 300

7. A certain firm has 28 lawyers in three states. These lawyers have licenses either in only one
state or all three states. If 10 have license in state X, 11 have license in Y, 13 have license in Z.
How many lawyers have licenses only in state Z ?

A. 3
B. 6
C. 10
D. 11
E. 13

8. Of all the students in a certain dormitory, 1/2 are first-year students and the rest are
second-year students. If 4/5 of the first-year students have not declared a major and if the
fraction of second-year students who have declared a major is 3 times the fraction of first-year
students who have declared a major, what fraction of all the students in the dormitory are
second-year students who have not declared a major?

1
A. 15
1
B. 5
4
C. 15
1
D. 3
2
E. 5
9. Among 100 students in a class, 57 like football and 68 like basketball. The maximum possible
number of students who like both football and basketball is represented by 'a', and the
minimum possible number of students who like both football and basketball is represented by
'b'. What is the value of 'a' minus 'b'?

A. 12
B. 19
C. 23
D. 28
E. 32
Word Problems

1. Last year a state senate consisting of only Republican and Democrat members had 20 more
Republican members than Democrat members. This year the senate has the same number of
members as last year, but it has 2 fewer Republican members than last year. If this year the
number of Republican members is 2/3 the number of senate members, how many members
does the senate have this year?

A. 33
B. 36
C. 42
D. 45
E. 48

2. A company wants to buy computers and printers for a new branch office, and the number of
computers can be at most 3 times the number of printers. Computers cost $1,500 each, and
printers cost $300 each. What is the greatest number of computers that the company can buy if
it has a total of $9,100 to spend on computers and printers?

A. 2
B. 3
C. 4
D. 5
E. 6
3. Abby, Brandon, Cedric, and Deirdre are planning to attend a concert. If their friend Kim also
goes with them, they will get a group discount on their tickets. If the total price of the tickets
with the group discount is the same as the total price of the tickets without the group discount,
how much is the discount?

A. 5%
B. 10%
C. 20%
D. 25%
E. 33%

4. Last month a certain music club offered a discount to preferred customers. After the first
compact disc purchased, preferred customers paid $3.99 for each additional compact disc
purchased. If a preferred customer purchased a total of 6 compact discs and paid $15.95 for the
first compact disc, then the dollar amount that the customer paid for the 6 compact discs is
equivalent to which of the following?

A. 5(4.00) + 15.90
B. 5(4.00) + 15.95
C. 5(4.00) + 16.00
D. 5(4.00 - 0.01) + 15.90
E. 5(4.00 - 0.05) + 15.95

5. The ratio between the present ages of Aakash and Bipul is 5 : 4. The ratio between Aakash’s
age 10 years hence and Bipul’s age 10 years ago is 5 : 3. What is the ratio between Aakash’s age
5 years ago and Bipul’s age 5 years hence ?

A. 13 : 20
B. 21 : 12
C. 12 : 21
D. 25 : 23
E. 25 : 21
6. Darcy lives at a distance of 1.5 miles from work. She can walk to work at a constant rate of 3
miles per hour, or she can ride the train to work at a constant rate of 20 miles per hour. If she
rides the train, there is an additional x minutes spent walking to the nearest train station,
waiting for the train, and walking from the final train station to her work. If it takes Darcy a total
of 15 more minutes to commute to work by walking than it takes her to commute to work by
riding the train, what is the value of x?

A. 10.5
B. 15
C. 25.5
D. 30
E. 60

7. The total cost of a vacation was divided equally among 3 people. If the total cost of the
vacation had been divided equally among 4 people, the cost per person would have been $50
less. What was the total cost of the vacation?

A. $200
B. $300
C. $400
D. $500
E. $600

8. During a certain two-week period, 70 percent of the movies rented from a video store were
comedies, and of the remaining movies rented, there were 5 times as many dramas as action
movies. If no other movies were rented during that two-week period and there were A action
movies rented, then how many comedies, in terms of A, were rented during that two-week
period?

𝐴
A. 14
5𝐴
B. 7
7𝐴
C. 5
D. 14A
E. 35A
9. Exchange Rates in a Particular Year

$1 = 5.3 francs
$1 = 1.6 marks

An American dealer bought a table in Germany for 480 marks and sold the same table in France
for 2,385 francs. What was the dealer's gross profit on the two transactions in dollars?

A. $0
B. $50
C. $100
D. $150
E. $200

10. A certain movie star's salary for each film she makes consists of a fixed amount, along with a
percentage of the gross revenue the film generates. In her last two roles, the star made $32
million on a film that grossed $100 million, and $24 million on a film that grossed $60 million. If
the star wants to make at least $40 million on her next film, what is the minimum amount of
gross revenue the film must generate?

A. $110 million
B. $120 million
C. $130 million
D. $140 million
E. $150 million
ANSWER KEYS FOR REFERENCE

TIME AND WORK

QUESTION ANSWER QUESTION ANSWER

1 C 6 B

2 D 7 D

3 B 8 D

4 A 9 D

5 D 10 D

TIME SPEED DISTANCE

QUESTION ANSWER QUESTION ANSWER

1 C 7 D

2 D 8 E

3 E 9 D

4 E 10 A

5 E 11 A

6 E
SET THEORY

QUESTION ANSWER QUESTION ANSWER

1 D 6 D

2 E 7 C

3 C 8 B

4 B 9 E

5 D

WORD PROBLEMS

QUESTION ANSWER QUESTION ANSWER

1 E 6 A

2 D 7 E

3 C 8 D

4 A 9 D

5 D 10 D
Statistics Extra Questions

Averages

1. The average (arithmetic mean) of 7 numbers in a certain list is 12. The average of the 4
smallest numbers in this list is 8, while the average of the 4 greatest numbers in this list is 20.
How much greater is the sum of the 3 greatest numbers in the list than the sum of the 3
smallest numbers in the list ?

A. 4
B. 14
C. 28
D. 48
E. 52

2. A group consists of both men and women. The average (arithmetic mean) height of the
women is 66 inches, and the average (arithmetic mean) height of the men is 72 inches. If the
average (arithmetic mean) height of all the people in the group is 70 inches, what is the ratio of
women to men in the group ?

A. 1:1
B. 1:2
C. 2:1
D. 2:3
E. 3:2
3. A certain list consists of 21 different numbers. If n is in the list and n is 4 times the
average(arithmetic mean) of the other 20 numbers in the list, then n is what fraction of the sum
of the 21 numbers in the list ?

1
A. 20
1
B. 6
1
C. 5
4
D. 21
5
E. 21

4. In a certain archery contest, archers are awarded a different number of points for each arrow,
depending upon the accuracy of the shot. If an arrow hits the bull’s-eye, 10 points are awarded.
The results for one archer are shown in the table, but the values do not include her bull’s-eyes.
If the archer’s average score per arrow is 6.0, how many of her attempts were bull’s-eyes ?

Points Number of Arrows

0 1

1 1

3 3

5 6

7 8

9 2

A. 0
B. 1
C. 2
D. 3
E. 4
5. The average (arithmetic mean) monthly income of four workers is $2,200. After one worker’s
income increases by 25 percent the new average income is $2,400. What was the original
income of the worker whose monthly income increased ?

A. $800
B. $1,200
C. $2,400
D. $3,200
E. It cannot be determined from the information given

6. An exam is given in a certain class. The average (arithmetic mean) of the highest score and
the lowest score is equal to x. If the average score for the entire class is equal to y and there are
z students in the class, where z > 5, then in terms of x, y, and z, what is the average score for the
class excluding the highest and lowest scorers?

(𝑧𝑦 – 2𝑥)
A. 𝑧
(𝑧𝑦 – 2)
B. 𝑧
(𝑧𝑥 – 𝑦)
C. (𝑧 – 2)
(𝑧𝑦 – 2𝑥)
D. (𝑧 −2)
(𝑧𝑦 – 𝑥)
E. (𝑧 + 2)

7. The median height of 10 people is 70, the average is 72.5, and the range is 12. If a person
with a height of 74 is added, which number will definitely change?

A. Mean
B. Median
C. Range
D. Mean & Range
E. None of the above
8. At a restaurant, glasses are stored in two different-sized boxes. One box contains 12 glasses,
and the other contains 16 glasses. If the average number of glasses per box is 15, and there are
16 more of the larger boxes, what is the total number of glasses at the restaurant ?
(Assume that all boxes are filled to capacity.)

A. 96
B. 240
C. 256
D. 384
E. 480

9. Fiber X cereal is 55% fiber. Fiber Max cereal is 70% fiber. Sheldon combines an amount of the
two cereals in a single bowl of mixed cereal that is 65% fiber. If the bowl contains a total of 12
ounces of cereal, how much of the cereal, in ounces, is Fiber X ?

A. 3
B. 4
C. 6
D. 8
E. 9

10. A certain car averages 25 miles per gallon of gasoline when driven in the city and 40 miles
per gallon when driven on the highway. According to these rates, which of the following is
closest to the number of miles per gallon that the car averages when it is driven 10 miles in the
city and then 50 miles on the highway?

A. 28
B. 30
C. 33
D. 36
E. 38
11. At a certain fruit stand, the price of each apple is 40 cents and the price of each orange is 60
cents. Mary selects a total of 10 apples and oranges from the fruit stand, and the average
(arithmetic mean) price of the 10 pieces of fruit is 56 cents. How many oranges must Mary put
back so that the average price of the pieces of fruit that she keeps is 52 cents ?

A. 1
B. 2
C. 3
D. 4
E. 5

12. Ashok purchased two qualities of grains at the rate of Rs 100 per quintal and Rs 160 per
quintal. In 50 quintals of the second quality, how much grain of the first quality should be mixed
so that by selling the resulting mixture at Rs 195 per quintal, he gains a profit of 30% ?

A. 10 quintals
B. 14 quintals
C. 20 quintals
D. 25 quintals
E. None of these

13. Kiran lends Rs 1000 on simple interest to Harsh for a period of 5 years. She lends a part of
the amount at 2% annual interest and the rest at 8% annual interest and receives Rs 300 as the
amount of interest. How much money (in Rs) did she lend at a 2% interest rate ?

A. 333.33
B. 666.67
C. 400
D. 500
E. None of the above
14. A man purchased a table and a chair for Rs 2000. He sold the table at a profit of 20% and the
chair at a profit of 40%. In this way, his total profit was 25%.Find the cost price (in Rs) of the
table.

A. 1500
B. 900
C. 1000
D. 800
E. None of the above
Median

1. Three positive numbers x, y, and z have the following relationships y = x + 2 and z = y + 2.


When the median of x, y, and z is subtracted from the product of the smallest number and the
median, the result is 0. What is the value of the largest number ?

A. –2
B. 3.14
C. 5
D. 8
E. 10.5

2. In a certain lottery drawing, five balls are selected from a tumbler in which each ball is
printed with a different two-digit positive integer. If the average (arithmetic mean) of the five
numbers drawn is 56 and the median is 60, what is the greatest value that the lowest number
selected could be ?

A. 43
B. 48
C. 51
D. 53
E. 56

3. A set of 5 numbers has an average of 50. The largest element in the set is 5 greater than 3
times the smallest element in the set. If the median of the set equals the mean, what is the
largest possible value in the set ?

A. 85
B. 86
C. 88
D. 91
E. 92
4. For the set of positive, distinct integers {v, w, x, y, z}, the median is 10. What is the minimum
value of v + w + x + y + z ?

A. 25
B. 32
C. 36
D. 40
E. 50

5. Which of the following cannot be the median of five positive integers a, b, c, d, and e ?

A. a
B. d + e
C. b + c + d
(𝑏 + 𝑐 + 𝑑)
D. 3
(𝑎 + 𝑏 + 𝑐 + 𝑑 + 𝑒)
E. 5

6 : $10, $11, $11, $12, $12, $14, $16, $17, $21, $22

The various prices that a certain product was sold for at retailers in City X is shown above. How
many prices were greater than the median price but less than the mean price?

A. None
B. One
C. Two
D. Three
E. Four
7. Positive integers from 1 to 45, inclusive are placed in 5 groups of 9 each. What is the highest
possible average of the medians of these 5 groups ?

A. 25
B. 31
C. 15
D. 26
E. 23

8. An analysis of the monthly incentives received by 5 salesmen : The mean and median of the
incentives is $7000. The only mode among the observations is $12,000. Incentives paid to each
salesman were in full thousands. What is the difference between the highest and the lowest
incentive received by the 5 salesmen in the month ?

A. $4000
B. $13,000
C. $9000
D. $5000
E. $11,000

9. The median annual household income in a certain community of 21 households is $50,000. If


the mean annual income of a household increases by 10% per year over the next 2 years, what
will the median income in the community be in 2 years?

A. $50,000
B. $60,000
C. $60,500
D. $65,000
E. Cannot be determined
10. { 150, 200, 250, n }
Which of the following could be the median of the 4 integers listed above?

I. 175
II. 215
III. 235

A. I only
B. II only
C. I and II only
D. II and III only
E. I, II, and III
Range

1. The range of the salary in January is $240 and the range of the salary in February is $320.
What is the minimal range of the salary for the two months combined ?

A. 80
B. 240
C. 320
D. 400
E. 480

2. 10 distinct integers are arranged in ascending order. The range of the first 7 is 25 and that of
the last 7 is 30. What is the maximum range of the 10 numbers?

A. 25
B. 27
C. 30
D. 52
E. 55

3. List K consists of 12 consecutive integers. If -4 is the least integer in list K, what is the range of
the positive integers in list K?

A. 5
B. 6
C. 7
D. 11
E. 12
2
4. S is the set that contains numbers of the form (𝑘 − 𝑛) , where k and n are integers such
that 4 ≤ k < 7 < n ≤ 12. What is the range of the numbers in S ?

A. 20
B. 30
C. 40
D. 50
E. 60

2 2
5. If Set M consists of the root(s) of the equation 2 - 𝑥 = (𝑥 − 2) , What is the range of Set
M?

A. 0
1
B.
2
C. 1
D. 2
E. 2

6. Set R contains 7 distinct positive integers {8, 6, 14, 1, 12, n, 7 }. The greatest possible value of
n should be how much greater than the least possible value of n, if the range of all the elements
present in S is not more than 21?

A. 12
B. 14
C. 15
D. 20
E. 21
7. Positive integers a, b, c, d and e are such that a < b < c < d < e. If the average (arithmetic
mean) of the five numbers is 6 and d - b = 3, then what is the greatest possible range of the five
numbers?

A. 12
B. 17
C. 18
D. 19
E. 20

8. The average (arithmetic mean) of a list of 8 different positive integers is 23. Which of the
following is the greatest possible range of this list of numbers?

A. 149
B. 155
C. 156
D. 168
E. 183
Standard Deviation

1. Set S is a set of distinct positive integers. The standard deviation of Set S must increase if
which of the following were to occur ?

I. The smallest number is increased to become equal to the median.


II. The smallest number is increased to become larger than the current largest number.
III. The largest number is doubled.

A. I only
B. II only
C. III only
D. I and III only
E. I, II and III

2. A list of 50 data points has an average (arithmetic mean) of 20 and a standard deviation of 10.
Which of the following pairs of data, when added to the list, must result in a list of 52 data
points with standard deviation less than 10 ?

A. 0 and 10
B. 0 and 20
C. 10 and 30
D. 20 and 20
E. 30 and 40

3. If a certain sample of data has a mean of 24.0 and the value 31.0 is more than 2.5 standard
deviations from the mean, which of the following could be the standard deviation of the
sample ?

A. 3.75
B. 3.50
C. 3.25
D. 3.00
E. 2.75
4. { 2 , 3 , 3 , 4 , 6 , 8 , 12 , 15 , 18 , 21 , 24 , 28 }

The list above shows the weight, in grams, of 12 plants grown in an experimental greenhouse. If
the standard deviation of the weights is 9, how many of the 12 plants are more than 1.5
standard deviations above the mean ?

A. 0
B. 1
C. 2
D. 3
E. 4

5. 500 records have a distribution that is symmetric about the mean 10. If 68 percent of the
distribution lies within one standard deviation of the mean, and the deviation is 2, then at least
how many of the records are less than 12 ?

A. 160
B. 320
C. 340
D. 420
E. 460

6. A set of 12 test scores has an average of 500 and a standard deviation of 50. Which of the
following sets of additional test scores, when combined with the original set of 12 test scores,
must result in a combined data set with a standard deviation less than 50?

A. 6 test scores with average of 450 and standard deviation of 50.


B. 6 test scores with an average of 500 and standard deviation of 25.
C. 6 test scores with average of 550 and standard deviation of 25.
D. 12 test scores with average of 450 and standard deviation of 25.
E. 2 test scores with average of 550 and standard deviation of 50.
7. Which of the following sets has a standard deviation greater than the standard deviation of
Set A = {10, 12, 14, 16, 18, 20, 22, 24, 26, 28} ?

I. Set B, which consists of the first 10 positive integers


II. Set C, which consists of the first 10 positive odd numbers
III. Set D, which consists of the first 10 prime numbers

A. Set B only
B. Set C only
C. Set D only
D. Sets C and D only
E. Sets B, C and D

8. Which of the following sets must have the same standard deviation as Set { a,b,c } ?
2
A. { ab , 𝑏 , cb }
B. { 2a, b + a , c + b }
C. { 0 , b + a , c - a }
D. { ab , bc , ac }
E. { ab + c , a(1 + b) , b(1 + a) }
ANSWER KEYS FOR REFERENCE

AVERAGES

Question Answer Question Answer

1 D 9 B

2 B 10 D

3 B 11 E

4 D 12 A

5 D 13 A

6 D 14 A

7 A

8 E

MEDIAN

Question Answer Question Answer

1 C 6 B

2 B 7 B

3 E 8 E

4 C 9 E

5 C 10 C
RANGE

Question Answer Question Answer

1 C 5 A

2 D 6 D

3 B 7 C

4 E 8 B

STANDARD DEVIATION

Question Answer Question Answer

1 C 5 D

2 D 6 B

3 E 7 C

4 B 8 E
Numbers Extra Questions

Odd-Even/Primes

1. w, x, y, and z are consecutive odd integers such that w < x < y < z. Which of the following
statements must be true? Indicate all such statements.
1. wxyz is odd
2. w + x + y + z is odd
3. w + z = x + y

A. 1 Only
B. 2 Only
C. 3 Only
D. 1 and 3
E. 1 and 2

2. If x is odd, which one of the following must be odd EXCEPT ?


A. x2 + 4x + 6
B. x3 + 5x + 3
C. x4 + 6x + 7
D. x5 + 7x + 1
E. x6 + 8x + 4

3. a and b are two positive even integers such that a > b. Which of the following represents the
number of odd integers less than a + 2 and greater than b - 2?

(𝑎 + 1 − 𝑏 − 1)
A. 2
(𝑎 − 𝑏 − 2)
B. 2
(𝑎 + 2 − 𝑏 − 2)
C. 2
(𝑎 + 2 − 𝑏 + 2)
D. 2
(𝑎 − 𝑏)
E. 2
+1
3 3 2
4. 𝑠 𝑡 = 𝑣
If s and t are both primes, how many positive divisors of v are greater than 1, if v is an integer?

A. Two
B. Three
C. Five
D. Six
E. Eight

𝑛
5. If n is an integer such that 15 < n < 250, for how many possible values of n is 5
the square of
a prime number ?

A. Two
B. Three
C. Four
D. Five
E. Six
HCF, LCM, Factors, Multiples

1. Company H distributed $4,000 and 180 pencils evenly among its employees, with each
employee getting an equal integer number of dollars and an equal integer number of pencils.
What is the greatest number of employees that could work for company H?

A. 9
B. 10
C. 20
D. 40
E. 180

2. Ramon wants to cut a rectangular board into identical square pieces. If the board is 18 inches
by 30 inches, what is the least number of square pieces he can cut without wasting any of the
board ?

A. 4
B. 6
C. 9
D. 12
E. 15

3. How many integers are there between 1 and 1000, inclusive, that are not divisible by either
11 or 35?

A. 884
B. 890
C. 892
D. 910
E. 945
4. If r is divisible by 10 and s is divisible by 9, rs must have at least how many factors ?

A. Two
B. Four
C. Twelve
D. Fourteen
E. Sixteen

3
5. If n is an integer and 𝑛 is divisible by 24, what is the largest number that must be a factor of
n?

A. 1
B. 2
C. 6
D. 8
E. 12

6. If a = 16b and b is a prime number greater than 2, how many positive distinct factors does a
have ?

A. 4
B. 5
C. 6
D. 8
E. 10
7. If a and b are integers such that a > b > 1, which of the following cannot be a multiple of
either a or b ?

A. a – 1
B. b + 1
C. b – 1
D. a + b
E. ab
Divisibility

1. If k is a multiple of 24 but not a multiple of 16 , which of the following cannot be an integer ?


𝑘
A. 8
𝑘
B. 9
𝑘
C. 32
𝑘
D. 36
𝑘
E. 81

2. What is the number of integers that are not divisible by 7 in the range of integers from 3 to
500 ?
A. 70
B. 71
C. 426
D. 427
E. 428

3. If 3x(52) is divided by 35(53), the quotient terminates with one decimal digit. If x > 0, which of
the following statements must be true?

A. x is even
B. x is odd
C. x < 5
D. x ≥ 5
E. x = 5
4. If M is the product of all positive integers greater than 59 and less than 71, then what is the
𝑀
greatest integer n for which 𝑛 is an integer?
6
A. 5
B. 7
C. 9
D. 11
E. 13

2
𝑡
5. If t is divisible by 12, what is the least possible integer value of a for which 𝑎 might not be an
2
integer ?

A. 2
B. 3
C. 4
D. 5
E. 6

6. If 8A5146B is divisible by 88, then what is the value of BA ?

A.81
B.64
C.15
D.12
E.3
7. Find the sum of the digits of the smallest 6-digit number divisible by 120.

A.3
B.12
C.6
D.9
E.7
Dividend=Div×Quo+Rem

1. When m + n is divided by 9, the remainder is 1. If m is divisible by 9, which of the following


could equal the sum of the digits of n?

A. 34
B. 35
C. 36
D. 37
E. 38

2. When the positive integer x is divided by 6 , the remainder is 4. Each of the following could
also be an integer EXCEPT

𝑥
A. 2
𝑥
B. 3
𝑥
C. 7
𝑥
D. 11
𝑥
E. 17

3. A positive integer N leaves a remainder of 4 when divided by 33. How many different
remainders can N have, when N is divided by 55 ?

A. 6
B. 5
C. 4
D. 3
E. 2
𝑠
4. If s and t are positive integers such that 𝑡
= 64.12 , which of the following could be the
remainder when s is divided by t ?

A. 2
B. 4
C. 8
D. 20
E. 45

5. A number when divided by a divisor leaves a remainder of 24. When twice the original
number is divided by the same divisor, the remainder is 11. What is the value of the divisor?

A. 12
B. 13
C. 35
D. 37
E. 59
Power of Primes in Factorial

1. What is the highest power of 111 which divides


30! × 31! × 32! × 33! × 34! × 35! × 36! × 37! × 38! × 39! × 40! ?

A. 3
B. 4
C. 5
D. 6
E. 7

2. How many positive factors of the number 28×36×54×105 are multiples of 120 ?

A. 540
B. 594
C. 660
D. 668
E. 792

3. For every positive integer x, f(x) represents the greatest prime factor of x!, and g(x)
represents the smallest prime factor of 2x+1. What is (g(f(12)) ?

A. 2
B. 3
C. 5
D. 7
E. 11
4. What is the highest integer power of 30 that can divide 31!?
A. 1
B. 7
C. 14
D. 15
E. 26

𝑦 2
5. If 6 is a factor of (10!) , What is the greatest possible value of y ?

A. 2
B. 4
C. 6
D. 8
E. 16
Units digit Cyclicity

1. Calculate the units digit of the following expression:


1!1 + 2!2+ 3!3 + 4!4+ 5!5…..+10!10

A.1
B.3
C.5
D.7
E.9

2. The last digit of 1212 + 1313– 1414×1515 =


A. 0
B. 1
C. 5
D. 8
E. 9

96
3. What is the remainder when 4 is divided by 6?

A. 0
B. 1
C. 2
D. 3
E. 4
28 47 19
4. What is the units digit of the product 32 × 33 × 37 ?

A. 0
B. 2
C. 4
D. 6
E. 8
Rounding Off

1. [x] denotes the greatest integer less than or equal to X. If X is a positive integer and

𝑥 𝑥
[ 5 ] − [ 7 ] = 1. If the minimum value of x is ‘a’ and the maximum value is ‘b’, then a + b = ?

A. 40
B. 35
C. 34
D. 33
E. 32

2. [x] denotes the largest integer less than or equal to x. If [x] = -1, which of the following must
be true ?

A. x = 1
B. −2 ≤ x <−1
C. −2 < x ≤ −1
D. −1 ≤ x< 0
E. −1< x ≤ 0

3. If [X] denotes the greatest integer less than or equal to x, what is the value of
[√1]+[√2]+[√3]+...+[√36] ?

A. 100
B. 111
C. 121
D. 126
E. 131
4. If X is the hundredths digit in the decimal 0.1X and if Y is the thousandths digit in the decimal
0.02Y, where X and Y are nonzero digits, which of the following is closest to the greatest possible
0.1𝑋
value of 0.02𝑌
?

A. 4
B. 5
C. 6
D. 9
E. 10

5. When x is rounded to the nearest tenth the result is 1.5, and when y is rounded to the
nearest tenth the result is 7.5. If z = y − x, which of the following inequalities gives all of the
possible values of z ?

A. 5 < z < 7
B. 5.5 < z < 6.5
C. 5.9 < z < 6.0
D. 5.9 < z < 6.1
E. 5.95 < z < 6.05
Factor Theorem

1. Check whether 2y + 1 is a factor of the polynomial 4y3 + 4y2 – y – 1 using the factor theorem.

2. Use the factor theorem to check whether y + 1 is a factor of the polynomial 3y4 + y3 – y2 + 3y +
2, or not.
ANSWER KEYS FOR REFERENCE

Odd-Even/Primes

QUESTION ANSWER

1 D

2 C

3 D

4 B

5 C

HCF, LCM, Factors, Multiples

QUESTION ANSWER

1 C

2 E

3 A

4 C

5 C

6 E

7 C
Divisibility

QUESTION ANSWER

1 C

2 D

3 D

4 A

5 D

6 B

7 D

Dividend=Div×Quo+Rem

QUESTION ANSWER

1 D

2 B

3 B

4 E

5 D
Power of Primes in Factorial

QUESTION ANSWER

1 B

2 B

3 B

4 B

5 D

Units digit Cyclicity

QUESTION ANSWER

1 D

2 B

3 E

4 D

Rounding Off

QUESTION ANSWER

1 C

2 D

3 E

4 D

5 D
Algebra Extra Questions

Functions
1. Which of the following functions satisfies f(a+b) = f(a) × f(b) for all positive numbers a and b ?

A. f(x) = x+1
B. f(x) = x2+1
C. f(x) = √x
1
D. f(x) = 𝑥
E. f(x) = 2x

2. The functions f(x) and g(x) are defined by f(x)= x2–1 and g(x)=1–2x. Given that f(g(k))= 3,
which of the following could be the value of k ?

1
A. 2
√3
B. 2
C. 1
3
D. 2
E. -1

3. For which of the following functions does f(x) = f(2−x)?

A. f(x) = x+2
B. f(x) = 2x−x2
C. f(x) = 2−x
D. f(x) = (2−x)2
E. f(x) = x2
4. For every even positive integer m, f(m) represents the product of all even integers from 2 to
m, inclusive. For example, f(12) = 2 x 4 x 6 x 8 x 10 x 12. What is the greatest prime factor of
f(24) ?

A. 23
B. 19
C. 17
D. 13
E. 11

5. The function f(m) is defined for all positive integers m as the product of m + 4, m+5, and m +
6. If n is a positive integer, then f(n) must be divisible by which one of the following numbers?

A. 4
B. 5
C. 6
D. 7
E. 11

6. m and n are positive integers and n > m. Function $ is defined as m $ n = sum of all integers
greater than m but less than n.
What is the value of (107 $ 119)-(109 $ 120) ?

A. -123
B. -2
C. 45
D. 95
E. 98
Arithmetic Progression

1. In an arithmetic progression, the 10th term is 11 and the 11th term is 10.How many
consecutive terms(starting from the first term) of the arithmetic progression should be
considered so as to make their sum equal to zero?
A.33
B.41
C.37
D.39
E.42

2. The sum of nine consecutive odd numbers of Set A is 621. What is the sum of a different set
of six consecutive even numbers whose lowest number is 15 more than the lowest number of
Set A?

A. 498
B. 488
C. 486
D. 478
E. 468
3. If the sum of the first 51 terms of arithmetic progression is zero, then which of the following
must be true ? (An arithmetic progression is a sequence of numbers such that the difference
between the consecutive terms is constant)

I. 51st largest term is zero


II. 26th largest term is zero
III. All terms are non negative

A. I
B. II
C. III
D. I and II
E. None of the above

4. A club consists of members whose ages are in arithmetic progression (an arithmetic
progression is a sequence of numbers such that the difference between the consecutive terms
is constant). The common difference being 3 months. If the youngest member of the club is just
7 years old and the sum of the ages of all the members is 250, then number of members in the
club are:

A. 18
B. 20
C. 22
D. 24
E. 25
5. The nth term (𝑡𝑛) of a certain sequence is defined as 𝑡𝑛 = 𝑡𝑛−1 + 4 . If t1 = −7 , then 𝑡71 =
A. 273
B. 277
C. 281
D. 283
E. 287

6. The sum of three numbers in an arithmetic progression is 36. The sum of the squares of the
three numbers is 464. Find the smallest number ?

A.8
B.10
C.12
D.16
E.13

7. Dave drove a total of 2250 miles over a period of 15 days. Each day after the first he
increased the number of miles he drove in a single day by one mile. What is the difference
between the average (arithmetic mean) number of miles he drove per day and the median
number of miles driven over the course of the 15 days?

A. 0
B. 1
C. 5
D. 15
E. 25
Geometric Progression

1. At the start of an experiment, a certain population consisted of 3 animals. At the end of each
month after the start of the experiment, the population size was double its size at the beginning
of that month. Which of the following represents the population size at the end of 10 months?

A. 23
B. 32
C. 2(310)
D. 3(210)
E. 3(102)

2. The turnover of an organization named 'Coverage' was $640 crore per annum in 2001. In the
same year another organization named 'Miles', established just a few years back, had a turnover
of $40 crore per annum. The turnover of 'Miles' increased by 300% every year whereas
Coverage's turnover increased by 700% every two years. After how many years will both
companies have the same turnover?

A. 4 years
B. 6 years
C. 8 years
D. 10 years
E. 12 years
3.
v, w, x, y, z
A geometric sequence is a sequence in which each term after the first is equal to the product of
the preceding term and a constant. If the list of numbers shown above is a geometric sequence,
which of the following must also be a geometric sequence?

I. 2v, 2w,2x,2y,2z
II. v+2, w+2, x+2, y+2, z+2
III. √v, √w, √x, √y, √z

A. I only
B. II only
C. III only
D. I and II
E. I and III

4. 5 and 15 are the first two terms in a geometric sequence. What is the absolute difference
between the 11th term and the 13th term?

A. 3×52
B. 5× 313 - 5× 311
C. 513
D. 40 × 310
E. 312 - 310
1 1 1 1
5. If r = 1 + 3
+ 9
+ 27
and S =1 + 3
r , then S exceeds r by

1
A. 3
1
B. 6
1
C. 9
1
D. 27
1
E. 81

6. x, y, z are three positive numbers in a geometric progression such that x < y < z, while 5x, 16y,
and 12z are in an arithmetic progression. What is the common ratio of the geometric
progression ?

3
A. 6
3
B. 2
5
C. 2
1
D. 6
3
E. 8

7. The numbers –2, x, –32 are the first three terms in a geometric progression. Which of the
following could be the sixth term in the progression?

A. –4,096
B. –2,048
C. –1,024
D. 512
E. 1,024
User-Defined Sequence

1. The first two terms of a sequence are -3 and 2. Subsequent odd-numbered terms are given by
adding 1 to the previous term, and subsequent even-numbered terms are given by multiplying
the previous term by -1. What is the sum of the first 147 terms?

A. -3
B. -2
C. -1
D. 1
E. 2

2. A sequence is given by the rule 𝑎𝑛=|𝑎𝑛−2|−|𝑎𝑛−1| for all n ≥ 3, where a1= 0 and a2 = 3. What
is 𝑎99 ?

A. -3
B. 0
C. 3
D. 201
E. 303

3. Find the sum of the first 20 terms of this series which begins this way: -12 + 22 - 32 + 42 - 52 + 62

A. 210
B. 330
C. 519
D. 720
E. 190
4. Each number Sn in a sequence can be expressed as a function of the preceding number (Sn–1)
2
as follows: Sn= 3
×Sn-1 – 4. Which of the following equations correctly expresses the value of Sn
in this sequence in terms of Sn+2 ?

9
A. Sn= 4
×Sn+2 + 18
4
B. Sn= 9
×Sn+2 + 15
9
C. Sn= 4
×Sn+2 + 15
4
D. Sn= 9
×Sn+2 − 8
2
E. Sn= 3
×Sn+2 − 8

5. The sequence a1, a2 , … , an is such that an= 2an−1− x for all positive integers n ≥ 2 and for a
certain number x. If a5 = 99 and a3 = 27, what is the value of x ?

A. 3
B. 9
C. 18
D. 36
E. 45

6. If each term in the sum a1 + a2 + a3 +... + an is either 2 or 22 and the sum equals 100, which of
the following could be equal to n?

A. 38
B. 39
C. 40
D. 41
E. 42
7. What is the difference between the fourth and third terms of the sequence defined by
an = 3n − n2 ?

A.18
B. 23
C. 47
D. 65
E. 83

𝑛
8. A sequence is defined as follows: an= 𝑛+1
. If n is a positive integer, then how many of the first
100 terms of this sequence are less than 0.891 ?

A. 7
B. 8
C. 9
D. 10
E. 12

9. In a certain sequence, Sn represents the sum of all terms from t1 to tn , inclusive (for
n≥1) . If Sn= 3n2, what is the value of t7 ?

A. 36
B. 39
C. 45
D. 49
E. 108
Linear Equation

­1. 5 friends - Ajax, Betty, Cody, Donna and Elsie - decide to contribute to a charity. Ajax
1 1
contributes 6
th as much as all the others combined, Betty contributes 5
th as much as all the
1
others combined, Cody contributes 4
th as much as all the others combined and Donna
1
contributes 3
rd as much as all the others combined. If the total contribution of the friends is
$42,000 , then how much did Elsie pay ?

A $8100
B $9100
C $10000
D $10100
E $11000

2. Thabo owns exactly 140 books, and each book is either paperback fiction, paperback
nonfiction, or hardcover nonfiction. If he owns 20 more paperback nonfiction books than
hardcover nonfiction books, and twice as many paperback fiction books as paperback nonfiction
books, how many hardcover nonfiction books does Thabo own?

A.10
B.20
C.30
D.40
E.50

3. If A+B = C, A+D = B, 2C = 3D and A, B, C, and D are positive integers, what is the least possible
value of B?

A. 1
B. 4
C. 5
D. 6
E. 10
4. Alice has 3 times the number of stamps that Doris does and Jane has 7 stamps more than
Doris does. If Alice, Doris, and Jane each increase their number of stamps by 4, which of the
following must be true after each person increases her number of stamps?

I.Alice has more stamps than Jane.


II.Jane has 3 more stamps than Doris.
III.The sum of the numbers of stamps that Alice and Doris have is a multiple of 4.

A.None
B.III only
C.I and III only
D.II and III only
E.I, II, and III

5. Three years from now, Dathan will be three times as old as Ellen and Ellen will be six years
younger than Famke. If Dathan's age is three years less than twice Famke's age, how old is
Famke ?

A. 9
B. 15
C. 21
D. 27
E. 33
Quadratic Equation

1. x2 + bx + 72 = 0 has two distinct positive integer roots. How many values are possible for b?

A. 3
B. 6
C. 8
D. 12
E. 24

2. The temperature inside a certain industrial machine at time t seconds after startup, for
2𝑡+1 𝑡+2
0 < t < 10, is given by h(t) = 4 - 4 degrees Celsius. How many seconds after startup is the
temperature inside the machine equal to 128 degrees Celsius?

3
A. 2
B. 2
5
C. 2
D. 3
7
E. 2

3. Which of the following equations has a root in common with x2− 6x + 5=0 ?
A. x2+1=0
B. x2−x−2=0
C. 2x2−2=0
D. x2−2x−3=0
E. x2−10x−5=0
4. What are the values of K for which the equation (3K+1)x2 + 2(K+1)x + 1 = 0 has equal roots?

A. 0 or 1
B. 0 or 2
C. 0 or 3
D. 1 or 2
E. 1 or 3

5. Which of the following equations has 3+2√3 as one of its roots?

A. x2 + 6x + 3 = 0
B. x2 − 6x + 3 = 0
C. x2 + 6x − 3 = 0
D. x2− 6x − 3 = 0
E. x2− 4√3x − 3 = 0

6. If q is one root of the equation x2 + 18x + 11c = 0, where –11 is the other root and c is a
constant, then q2-c2 =

A. 98
B. 72
C. 49
D. 0
E. It can't be determined from the information given
ANSWER KEYS FOR REFERENCE

Functions

QUESTION ANSWER QUESTION ANSWER

1 E 4 E

2 D 5 C

3 B 6 E

Arithmetic Progression

QUESTION ANSWER QUESTION ANSWER

1 B 5 A

2 C 6 A

3 B 7 A

4 E

Geometric Progression

QUESTION ANSWER QUESTION ANSWER

1 D 5 E

2 C 6 C

3 A 7 B

4 D
User Defined Sequence

QUESTION ANSWER QUESTION ANSWER

1 E 6 C

2 A 7 C

3 A 8 B

4 C 9 B

5 A

Linear Equations

QUESTION ANSWER QUESTION ANSWER

1 D 4 B

2 B 5 A

3 C

Quadratic Equations

QUESTION ANSWER QUESTION ANSWER

1 B 4 A

2 A 5 D

3 C 6 D
Coordinate Geometry + Graphs Extra Questions

1.

In the rectangular coordinate system shown above, which quadrant, if any, contains no point
(x,y) that satisfies the inequality 2x − 3y ≤ −6 ?

A. None
B. I
C. II
D. III
E. IV

2. A student draws 35 lines in the XY plane, none of which are vertical and 15 of which have a
positive slope. Of those with positive slope, 1/3 also have a positive Y-intercept. If 23 lines have
a Y-intercept less than or equal to zero, how many lines contain no point in Quadrant I ?

A. 0
B. 7
C.13
D.20
E. 35
3. In the coordinate plane, Line H has a positive slope, and Line J never passes through the first
quadrant. Which of the following could be true?

I. line J is perpendicular to line H


II. line J is parallel to line H
III. line J intersects line H on the x-axis

A. I only
B. II only
C. I and III only
D. II and III only
E. I, II, and III

1
4. A line in the xy-plane passes through the origin and has a slope of 7
. Which of the following
points lies on the line ?

A. (0, 7)
B. (1, 7)
C. (7, 7)
D. (14, 2)
E. (7, 14)

5. In the coordinate plane, Line J passes through the origin as well as points (a, 2) and (8, a).
Which of the following could be the slope of Line J ?

1
A. 4
1
B. 2
C. 1
D. 2
E. 4
6. In the xy-plane , line n passes through the origin and has slope 4. If points (1, c) and (d, 2) are
𝑐
on line n , what is the value of 𝑑
?

A. 0.25
B. 0.5
C. 2
D. 4
E. 8

7. The y-intercepts, P and Q, of two perpendicular lines intersecting at the point A(6, 8) have a
1
sum of zero. What is the area of triangle APQ ? (Area of the triangle = 2
×Base× Height)

A. 45
B. 48
C. 54
D. 60
E. 72

8.If each of the two lines p and q are parallel to line r , which of the following MUST be correct ?

I. Lines p, q and r lie in the same plane.


II. Lines p and q are parallel to one another.
III. Line p is the same as line q.

A. Only I
B. Only II
C. Only III
D. Only I and II
E. Only II and III
9. If line m is perpendicular to line L and line L passes through points (6, 5) and (6, 8) , what is
the slope of line m ?

A. -1
B. 0
C. 1
D. the slope is undefined
E. the slope cannot be determined from the information given

10. Line K is tangent to the circle with center (0, 0) at (-3, y) such that its y-intercept is positive.
If the radius of the circle is 5, what is the slope of Line K?

−4
A. 3
−3
B. 4
3
C. 4
1
D. 2
E. Cannot be determined

11. Which of the following is the equation of the line in the xy-plane that has slope 0 ?

I. x = 2
II. y = 3
III. x + y = 0

A. I only
B. II only
C. III only
D. I and II only
E. II and III only
12.

In the figure above, lines k, m and p have slopes r, s and t respectively. Which of the following is
a correct ordering of these slopes ?

A. r < s < t
B. r < t < s
C. s < r < t
D. s < t < r
E. t < s < r

13. If line k passes through the points (48, 33) and (31, 22), what is the x-intercept of line k?

A. -2
B. -3
C. -1
D. 1
E. 2
14. If a line passes through the point (3, 5) and has a positive slope, then which of the following
must be true about the y-intercept (c) of the line ?

I. c < 0
II. c = 0
III. c > 0

A. I only
B. II only
C. I & II only
D. I, II & III
E. None of the above

15. Line D passes through point (-2,5), and the product of its x-intercept and y-intercept is
positive. Which of the following points could be on line D?

A. (5,10)
B. (-3,2)
C. (-1,7)
D. (-1,2)
E. (-3,1)

4
16. The equation of line n is y = 3
x - 100. What is the smallest possible distance in the xy-plane
from the point with coordinates (0, 0) to any point on line n?

A. 48
B. 50
C. 60
D. 75
E. 100
17. In the xy-coordinate system, the distance between the point (0,0) and point P is
√40. Which of the following could be the coordinates of point P?

A. (4,7)
B. (4,10)
C. (5,6)
D. (6,2)
E. (20,20)

18. In the XY-coordinate system, line k passes through points (-5m, 0) and (0, 2m). Which of the
following is a possible equation of line k ?

5
A. y = − 2
x + 2m
2
B. y = 5
x - 5m
5
C. y = 2 x + 2m
2
D. y = 5 x + 2m
2
E. y = − 5 x - 5m

19. What is the equation of the line parallel to the line 2x - 7y =20 and making an intercept of 7
on the x axis ?

A. 2x - 7y =7
B. x - y = 7
C. 2x - y = 7
D. x - 7y = 14
E. 2x -7y =14
20. What is the equation of the line that is the perpendicular bisector of the line segment
connecting points (4,–2) and (–3,5) in the xy plane ?

3
A. y = –x + 2
1
B. y = x + 2
3
C. y = 2
x–1
D. y = –x + 2
E. y = x + 1

21. Line k is in the rectangular coordinate system. If the x-intercept of k is -2, and the y-intercept
is 3, which of the following is an equation of line k ?

A. -3x + 2y = 6
B. 3x + 2y = -6
C. 3x - 2y = 6
D. 2x - 3y = 6
E. -2x - 3y = 6

22. The standard equation of a circle is (x−h)2+(y−k)2= r2 , where the point (h,k) is the center of
the circle and r represents the length of the radius. If a particular circle can be represented by
the equation x2 + y2 − 8x + 2y = −1, what is the radius of that circle ?

A. -1
B. 1
C. 2
D. 4
E. 8
23. The coordinates of points A and C are (0, -3) and (3, 3), respectively. If point B lies on line AC
between points A and C, and if AB = 2BC, which of the following represents the coordinates of
point B ?

A. (1, -√5)
B. (1, -1)
C. (2, 1)
D. (1.5, 0)
E. (√5, √5)

24.

Which of the following could be the equation of line m ?

A. 6y + 6x = 7
B. 3y = -4x - 3
C. 5y + 10 = -4x
D. y = 2
E. x = -2
25. Which of the following points (x, y) is NOT on the graph of y < 2x ?

A. (-3, -7)
B. (3, 3)
C. (2, -9)
D. (2, 2)
E. (2, 5)

26. If a circle circumscribes a triangle bounded by the x-axis, y-axis and the line 3y - 4x = -24,
what is the radius of the circle ?
A. 2.5
B. 3
C. 4
D. 5
E. 10

27. A parabola has one x-intercept at (– 4, 0). If the vertex is at (2, 5), find the other x-intercept.

A. 5
B. 6
C. 7
D. 8
E. 9
28. On the x-y coordinate plane there is a parabola, y = x(6-x). Which of the following points is in
the region that surrounds this parabola and x-axis ?

I. (1, 4)
II. (3, 6)
III. (5, 4)

A. I only
B. II only
C. III only
D. I & II
E. I, II & III

29.

A graph of the function g(x) is shown above. g(x) is defined by which of the following equations?

A. g(x) = (x+1)2 − 2
B. g(x) = (x−1)2 − 2
C. g(x) = (x+1)2 + 2
D. g(x) = (x−1)2 + 2
E. g(x) = (x−2)2 + 1
30. Which of the following points are on the graphs of both the equation y = x + 2 and the
equation y = x2 in the xy-plane?

I. (-1,1)
II. (0, 0)
III. (2,4)

A. I only
B. II only
C. III only
D. I and III only
E. I, II and III

31.

In the rectangular coordinate system above, the area of triangular region PQR is
1
( Area of Triangle = 2
×base×height )

A. 12.5
B. 14
C. 10√2
D. 16
E. 25
32. What is the area of the region bounded by the lines y = |x| - 2 and y = 2 - |x| ?

A. 2
B. 6
C. 8
D. 12
E. 16

𝑥
33. What is the area of the triangle formed by the intersection of lines y =2x – 2, y = − 2
+8,
and y = 0 ?

A. 20
B. 30
C. 40
D. 45
E. 60
34.

In the rectangular coordinate system above, triangle ABC has an area of 27 square units. Vertex
A is at point (0, 9), vertex B is at point (0, y), and vertex C is at point (9, y). What is the value of y
?

A. 1
B. 2
C. 3
D. 4
E. 6
35.

Point P is on the line y = 2x+a. The x-coordinate of P is 4. The area of the shaded quadrilateral is
24. What is the value of a ?
1
[ Area of Trapezium = 2
× (Sum of parallel sides) × height ]

A. 1
4
B. 3
C. 2
7
D. 6
E. 3
36.

In the rectangular coordinate system above, the line y = x is the perpendicular bisector of
segment AB (not shown), and the x-axis is the perpendicular bisector of segment BC (not
shown). If the coordinates of point A are (2,3), what are the coordinates of point C ?

A. (-3,-2)
B. (-3,2)
C. (2,-3)
D. (3,-2)
E. (2,3)

18
(1+2 )
37. Which of the following is closest to 4 10 ?
(2 +2 )

A. 25
B. 26
C. 27
D. 28
E. 29

1
38. Which of the following is closest to 1.001
?

A. 0.997
B. 0.998
C. 0.999
D. 1.000
E. 1.001
39. AB + CD = AAA , where AB and CD are two-digit numbers and AAA is a three digit number ;
A, B, C, and D are distinct positive integers. What is the value of C ?

A. 1
B. 3
C. 7
D. 9
E. Cannot be determined

40.

In the correctly worked addition problem shown, P, Q, R, and S are digits. If Q = 2P, which of the
following could be the value of S ?

A. 3
B. 2
C. 1
D. -1
E. -2
ANSWER KEYS FOR REFERENCE

QUESTION ANSWER

1 E

2 C

3 C

4 D

5 B

6 E

7 D

8 B

9 B

10 C

11 B

12 C

13 B

14 E

15 D

16 C

17 D

18 D

19 E

20 E

21 A

22 D
23 C

24 A

25 E

26 D

27 D

28 E

29 B

30 D

31 A

32 C

33 D

34 C

35 C

36 D

MISCELLANEOUS

37 D

38 C

39 D

40 A
PnC , Probability Extra Questions

PERMUTATIONS AND COMBINATIONS

1. There’s a party in town and Mike can’t decide what to wear. However, he has decided to
follow the following:
a. He will wear tie and shoes
b. He may or may not wear socks.
Mike has 3 ties, 4 pairs of shoes and 5 pairs of socks. In how many ways can Mike dress for the
party?

A. 12
B. 60
C. 72
D. 144
E. 288

2. In her class of 10 girls and 8 boys, the teacher has to select either a girl OR a boy. In how
many ways can she make her selection?
A. 12
B. 18
C. 40
D. 80
E. 160
3. How many 3 digits even numbers are possible using the digits 0,3,1,6,7,9 if repetition of digits
is allowed ?

A. 20
B. 30
C. 40
D. 60
E. 120

4. Of the three-digit integers greater than 660, how many have two digits that are equal to each
other and the remaining digit different from the other two?

A. 47
B. 60
C. 92
D. 95
E. 96

5. A teacher wishes to seat 5 boys and 3 girls in a row at a bench. In how many ways can the
teacher seat the boys and the girls so that all the girls do not sit together?

A. 720
B. 2160
C. 4032
D. 18000
E. 36000
6. In how many ways 4 boys and 3 girls can be seated in a row so that they are alternate ?

A. 12
B. 24
C. 72
D. 96
E. 144

7. A certain company places a six-symbol code on each of their products. The first two symbols
are one of the letters A–E and the last four symbols are digits. If repeats are allowed on both
letters and numbers, how many such codes are possible?

A. 250000
B. 350000
C. 450000
D. 550000
E. 600000

8. If all the letters of the word RATE are taken and arranged in alphabetical order as in a
dictionary , then what is the rank of the word TEAR ?

A. 20
B. 23
C. 22
D. 21
E. 19
9. In how many ways can 6 people be seated around a circular table if 2 particular people are
always separated ? (2 seating arrangements are considered different only when the positions of
the people are different relative to each other.)

A. 24
B. 48
C. 72
D. 96
E. 120

10. In how many ways can a party of 4 men and 4 women be seated at a circular table so that
no two men are adjacent? (Two sitting arrangements are considered different only when the
positions of the people are different relative to each other.)

A. 24
B. 48
C. 72
D. 144
E. 288

11. If a committee of 3 men and 3 women is to be selected from a group of 7 men and 7
women, how many different committees are possible choices ?

A. 36
B. 48
C. 242
D. 352
7!
E. ( 3! )2
12. A committee of four is to be chosen from seven employees for a special project at ACME
Corporation. Two of the seven employees are unwilling to work with each other. How many
committees are possible if the two employees do not work together ?

A. 15
B. 20
C. 25
D. 35
E. 50

13. A committee needs to be formed of 3 women and 2 men from a group of 6 women and 5
men. Among the chosen members, one needs to be selected for the post of the President, and
one for the post of Secretary. In how many ways can the President and Secretary be chosen
from this committee formed ?

A. 200
B. 400
C. 600
D. 800
E. 4000

14. What is the total number of ways in which Kosha can distribute 9 distinct gifts among her 8
distinct girlfriends such that each of them gets at least one gift ?

A. 2592*8!
B. 144*8!
C. 72*8!
D. 96*7!
E. 72*7!
15. The number of ways in which 5 different books can be distributed among 10 people if each
person can get at most one book is

A. 252
5
B. 10
10
C. 5
D. 10C5 * 5!
E. 10C5 * 10!

16. In how many ways can 9 identical balls be distributed among four baskets such that each
basket gets at least one ball ?

A. 35
B. 56
C. 63
D. 70
E. 126
17. How many triangles can be inscribed in the heptagon pictured, where the three vertices of
the triangle are also vertices of the heptagon ?

A. 9
B. 21
C. 35
D. 140
E. 210

18. If 4 points are indicated on a line and 5 points are indicated on another line that is parallel to
the first line, how many triangles can be formed whose vertices are among the 9 points ?

A. 20
B. 30
C. 40
D. 70
E. 90
19. What is the number of ways in which a person can move from point A to B walking along the
grids but not moving upwards or left of the grid or even retracing any grid ?

A. 45
B. 210
C. 35
D. 70
E. 25

20. What is the number of the shortest routes from X to Y through Z ?

A. 8
B. 12
C. 16
D. 18
E. 20
21. Halle, Julia and Drew have 5 donuts to share. If one of them can be given any number of
donuts from 0 to 5, in how many different ways can the donuts be distributed ?

A. 19
B. 20
C. 21
D. 23
E. 37

22. Find the number of positive integer solutions to a + b + c = 10, where none of
a, b or c is 0.

A. 36
B. 72
C. 144
D. 288
E. 576

23. In how many ways can 7 identical balls be placed into four boxes P,Q,R,S such that the two
boxes P and Q have at least one ball each ?

A.84
B.70
C.120
D.56
E.54
PROBABILITY

1.If John throws a coin until a series of three consecutive heads or three consecutive tails
appears, what is the probability that the game will end on the fourth throw ?

1
A. 16
1
B. 8
3
C. 16
1
D. 4
3
E. 8

2. A gambler rolls three fair six-sided dice. What is the probability that two of the dice show the
same number, but the third shows a different number?

1
A. 8
5
B. 18
1
C. 3
5
D. 12
5
E. 6
3. If two fair, six-sided dice are rolled, what is the probability that the sum of the numbers will
be 5 ?

1
A. 36
1
B. 18
1
C. 9
1
D. 6
1
E. 4

4. A bag contains 5 red balls and 4 white balls. Three balls are drawn without replacement.
What is the probability of drawing at least 2 red balls?

5
A. 42
5
B. 18
17
C. 42
25
D. 42
13
E. 18

5. A bag contains 3 red, 4 black and 2 white balls. What is the probability of drawing a red and a
white ball in two successive draws, each ball NOT being put back after it is drawn ?

2
A. 27
1
B. 6
1
C. 3
4
D. 27
2
E. 9
6. In a certain game, you pick a card from a standard deck of 52 cards. If the card is a heart, you
win. If the card is not a heart, the person replaces the card to the deck, reshuffles, and draws
again. The person keeps repeating that process until he picks a heart, and the point is to
measure how many draws it takes before the person picks a heart and wins. What is the
probability that there will be at least three draws involved in a win, i.e. someone picking her
first heart on the third draw or later?

1
A. 2
9
B. 16
11
C. 16
13
D. 16
15
E. 16

7. From a pack of 52 cards, two are drawn one by one without replacement. What is the
probability that both of them are jacks ?

1
A. 221
11
B. 850
25
C. 204
5
D. 24
5
E. 21

8. If events E and F are independent and each has a probability p of occurring, which of the
following represents the probability that E will occur but F will not occur?

2
A. 𝑝
2
B. 1 - 𝑝
2
C. 𝑝 - 1
2
D. 𝑝 - p
2
E. p - 𝑝
9. The probability of event p occurring is 0.4. The probability of event q occurring is 0.25. If
events p and q are independent, what is the probability that at most 1 of the events occur ?

A. 0.1
B. 0.35
C. 0.325
D. 0.9
E. 1.15

10. If a certain coin is flipped, it has probability 0.5 of landing on heads and probability 0.5 of
landing on tails. If the coin is flipped 4 times, what is the probability that it will land on tails at
least once?

3
A. 8
1
B. 4
1
C. 2
5
D. 16
15
E. 16

11. When tossed, a certain coin has equal probability of landing on either side. If the coin is
tossed 3 times, what is the probability that it will land on the same side each time?

1
A. 8
1
B. 4
1
C. 3
3
D. 8
1
E. 2
12. In a class, 60% of students play soccer or basketball. If 10% play both sports and 60% do not
play soccer, what is the probability that a student chosen at random from the class plays neither
soccer nor basketball?

A. 0.3
B. 0.4
C. 0.5
D. 0.6
E. 0.7

13. A fair die is rolled once and a fair coin is flipped once. What is the probability that either the
die will land on 3 or that the coin will land on heads?

1
A. 2
5
B. 12
7
C. 12
2
D. 3
3
E. 4

14. The probability of P hitting the target is 0.8 and that of Q hitting the target is 0.9 . If both of
them fire independently in a shooting competition, what is the probability that the target will
get hit ?

A. 0.72
B. 0.98
C. 0.7
D. 0.8
E. 0.28
15. Three snipers shoot a certain target. Their probabilities of hitting the target are 0.9, 0.7, and
0.5 respectively. What is the probability that exactly one sniper hit the target ?

A. 0.015
B. 0.185
C. 0.485
D. 0.515
E. 0.985

16. The probability of rain is 1/2 for every day next week. What is the chance that it rains on at
least one day during the workweek (Monday through Friday)?

1
A. 2
31
B. 32
63
C. 64
127
D. 128
5
E. 2

17. At an archery tournament, eight of the twenty competitors were left-handed and the
remainder were right-handed. Due to the weather conditions it is more difficult for left-handers
to hit the target. Practice sessions indicated that the probability of a left-hander hitting the
target was 0.7, while for right-handers it was 0.9. If an archer hits the target, what is the
probability that the person was right-handed ?

41
A. 50
27
B. 41
3359
C. 9500
37
D. 100
7
E. 25
ANSWER KEYS FOR REFERENCE

PERMUTATIONS AND COMBINATIONS

QUESTION ANSWER

1 C

2 B

3 D

4 D

5 E

6 E

7 A

8 D

9 C

10 D

11 D

12 C

13 E

14 C

15 D

16 B

17 C

18 D

19 C

20 B
21 C

22 A

23 D

PROBABILITY

QUESTION ANSWER

1 B

2 D

3 C

4 D

5 B

6 B

7 A

8 E

9 D

10 E

11 B

12 B

13 C

14 B

15 B

16 B

17 B
Inequality Modulus Extra Questions

Inequalities

1. If x2 − 9 < 0, which of the following is true?

A. x < −3
B. x > 3
C. x > 9
D. x < −3 or x > 3
E. −3 < x < 3

2. If (|x| - 2)(x + 5) < 0, then which of the following must be true?

A. x > 2
B. x < 2
C. -2 < x < 2
D. -5 < x < 2
E. x < -5

3. Which of the following is a value of x for which x−9 – x−7 > 0?

A. -2
B. -1
1
C. − 2
D. 1
E. 2
4.

Which of the following inequalities specifies the shaded region ?

2
A. 𝑥 + 1 < 3
B. |2x - 3| < 5
C. |x + 1| > -1
D. x - 2 < 2
E. |x - 1| < 4

5. If -13 < 7a + 1 < 29 and 19 < 2 - b < 23, what is the maximum possible integer value of a + b?

A. -23
B. -18
C. -14
D. -13
E. -12

6 .If x3 > y2 > z, which of the statements could be true?

I. x < y < z
II. x < z < y
III. y < x < z

A. I only
B. III only
C. I and II only
D. II and III only
E. I, II and III
7. If (x + 3)2 = 225, which of the following could be the value of x - 1 ?

A. 13
B. 12
C. -12
D. -16
E. -19

(𝑥+2)(𝑥+3)
8. How many of the integers that satisfy the inequality (𝑥−2)
≥ 0 are less than 5 ?
A. 1
B. 2
C. 3
D. 4
E. 5

9. If x < y , 2x = A, and 2y = B, then

A. A = B
B. A < B
C. A > B
D. A < x
E. B < y

(𝑥+𝑘)
10. If x, y, and k are positive and x is less than y, then (𝑦+𝑘)
is

A.1
B.greater than x/y
C.equal to x/y
D.less than x/y
E.less than x/y or greater than x/y, depending on the value of k
11.
5𝑝 + 3𝑞 ≤ 17,000
7𝑝 + 9𝑞 ≤ 43,000

A manufacturer wants to make p screws and q nails from a shipment of raw material. Material
constraints require p and q to satisfy the inequalities given above. What is the maximum total
number of screws and nails that can be created, given the material constraints?

A. 4,000
B. 5,000
C. 6,000
D. 7,000
E. 8,000­

12. If x ≥ 8 and y ≤ 3, then it must be true that

A. x + y ≥ 5
B. x + y ≤ 11
C. x − y ≥ 5
D. x − y ≤ 5
E. x − y ≤ 11

13. If 3x + 7y = 8x − 18y and y > 7, then which of the following must be true?

7
A. x < 5
B. x < 5
C. x > 18
D. x < 35
E. x > 36
14. If 1 < x < 7 and 2 < y < 9, what is the largest integer value of x + y?

A. 4
B. 13
C. 14
D. 15
E. 16

15. If a > b and c < a , which of the following must be true ?

I. a > b > c
II. 2a > b + c
III. a2 > bc

A. I only
B. II only
C. III only
D. II and III only
E. I, II, and III

16. Which of the following inequalities is true?

1 1
A. 11
< 0.08 < 9
1 1
B. 10 < 0.11 < 8
1 1
C. 7 < 0.17 < 6
1 1
D. 5 < 0.26 < 4
1 1
E. 3 < 0.08 < 2
17. For real numbers x and y,

−5 ≤ x − y ≤ 7
−9 ≤ x + y ≤ 6

What is the greatest possible value of x2−y2 ?

A. 35
B. 42
C. 45
D. 54
E. 63

18. ­At a certain store, all notebooks have the same price and all pencils have the same price.
The price of four notebooks and three pencils is more than $12 and less than $13. The price of
two notebooks and five pencils is more than $8 and less than $9. If a notebook costs x more
than a pencil, which of the following could be the value of x?

A. $0.60
B. $1.05
C. $1.30
D. $2.20
E. $2.70

3 5
19. Which of the following inequalities, if true, is sufficient alone to show that 𝑥 < 𝑥 ?

A. −1< x < 0
B. x > 1
C. |x|< −1
D. |x|> 1
E. x < −1
20. If x < y < 0, which of the following inequalities must be true ?

A. y + 1 < x
B. y - 1 < x
C. xy2 < x
D. xy < y2
E. xy < x2
Modulus

1.

­The number line below is equivalent to which of the following inequalities?

A. |x| ≤ 5
B. |x| ≥ 6
C. |3x – 1| ≤ 14
D. |4x + 2| ≤ 22
E. |10x – 6| ≤ 54­­

2. When |x|>1 and |x-3|>1, which of the following is contained in the scope of x ?

A. -0.5
B. 0
C. 0.5
D. 1.5
E. 2.5
3. Which of the following inequalities represents the entire solution set for x if |x| > x ?

A. x ≥ 0
B. x > 0
C. x ≤ 0
D. x < 0
E. x < −1

4. If a + b > 0 and ab < 0, which of the following must be true ?

I. a < 0
II. b > 0
III. |b|>|a|

A. I only
B. I and II only
C. I and III only
D. I, II and III
E. None of these

5. Find the number of real solutions of the equation x2 - 2|x| - 8 = 0.

A. 0
B. 1
C. 2
D. 3
E. 4
6. If x > 0 and 3x + 4|y| = 33, then how many integer pairs of (x, y) are there ?

A. 6
B. 5
C. 4
D. 3
E. 2

7. If |x| + |y| = – x – y and xy does not equal 0, which of the following must be true ?

A. x+y>0
B. x+y<0
C. x–y>0
D. x–y<0
E. x2 – y2 > 0

8. If |12x−5|>|7−6x| , which of the following CANNOT be the product of two possible values of
x?

A. -12
B. -7/5
C. -2/9
D. 4/9
E. 17
9. If y = |2 + x| - |2 – x| and |2x – 15| < 2, how many integer values can y take?

A. 0
B. 1
C. 2
D. 4
E. Cannot be determined

10. Which of the following inequalities is equal to |2x-|x||<6?

A. -2 < x < 0
B. 0 < x < 6
C. -2 < x < 6
D. -2 < x < 2
E. -6 < x < 6

11. The equation |2x - 3| = x - 5 has how many solutions for x ?

A. 0
B. 1
C. 2
D. 3
E. 4­

12. The sum of all solutions for x in the equation x2 – 8x + 21 = |x – 4|+ 5 is equal to:

A. –7
B. 7
C. 10
D. 12
E. 14
13. If x and y are integers and |x - y| = 12, what is the minimum possible value of xy ?

A. -12
B. -18
C. -24
D. -36
E. -48

14.

The absolute value of (x + a) will be the greatest if x is the coordinate of which of the five points
P, Q, R, S and T on the number line above?

A. P
B. Q
C. R
D. S
E. T

15. If -8 ≤ a ≤ 1 and -5 ≤ b ≤ 10. Find the sum of minimum and maximum value of a - |b|.

A. -19
B. -23
C. -18
D. -22
E. -17
16. Which of the following inequalities has a solution set that, when graphed on a number line,
is a single, finite line segment ?

A. x ≥ 4
B. x2 ≥ 4
C. x3≥ 64
D. |x|≥ 4
E. |x|≤ 4

2
17. If (3𝑥 + 3) = 900, which of the following could be the value of x -1 ?

A. -12
B. -11
C. -8
D. 9
E. 11

2
18. What is the product of all the solutions of (𝑥 + 2) = |x + 2| ?

A. -6
B. -2
C. 2
D. 6
E. 12
19. If |a+b|=|a-b|, then a×b must be equal to

A. 1
B. -1
C. 0
D. 2
E. -2

20. If |z|/w = 1, which of the following must be true ?

A. z = -w
B. z = w
C. z2 = w2
D. z2 = w3
E. z3 = w3
ANSWER KEYS FOR REFERENCE

INEQUALITIES

QUESTION ANSWER

1 E

2 B

3 A

4 B

5 C

6 E

7 E

8 D

9 B

10 B

11 B

12 C

13 C

14 D

15 B

16 B

17 C

18 D

19 E

20 E
MODULUS

QUESTION ANSWER

1 D

2 D

3 D

4 D

5 C

6 B

7 B

8 C

9 B

10 C

11 A

12 D

13 D

14 E

15 E

16 E

17 A

18 A

19 C

20 C
EXTRA QUESTION SOLUTIONS
Arithmetic Part 1 Extra Question Solutions

PERCENTAGES

1. In a basket of fruits, 60% are mangoes and the remaining 40% are apples. 25% of the apples
are green and the rest 75% are red. Of the mangoes, 80% are red and the rest of the mangoes
are green. What percentage of the green fruits are mangoes?

A. 63.63%
B. 54.54%
C. 40%
D. 30%
E. 20%

SOLUTION :

Please pay attention to the base of the percentages given. For 60% and 40%, the base is the
total number of fruits. 25% and 75% are percentages of apples whereas 80% and 20% are
percentages of mangoes. And in the question asked, the base has to be the total number of
green fruits!

If the total number of fruits is 100, 60 are mangoes and 40 are apples.
Green Apples = 25% of 40 = 10
Green Mangoes = 20% of 60 = 12
Total number of green fruits = 10 + 12 = 22 of which 12 are mangoes

12 6
Therefore, the required percentage = 22
= 11
= 54.54%
ANSWER B
2. In an exam, a student scored 50% of the maximum marks and yet failed by 12 marks. If he
had scored 10% more than what he scored, he would have just managed to get the pass
percentage. What is the maximum mark of the paper?

A. 100
B. 120
C. 180
D. 200
E. 240

SOLUTION :

Max marks = M
Students score = 50%M
Passing Marks = 50%M + 12
According to the question , If he had scored 10% more than what he scored, he would have just
managed to get the pass percentage.

=> 50%M (1 + 10%) = 50%M + 12


=> 5% M = 12
=> M = 240

Alternate Solution :

As per the given question, 10% of the (marks he scored) is equal to 12 marks as he failed by 12
marks. Thus, marks scored by the student is 120. This value is 50% of the maximum marks,
suggesting that maximum marks is 240.

ANSWER E
3. Mira's expenditure and savings are in the ratio 3: 2. Her income increases by 10%. Her
expenditure increases by 12%. By what percentage does her savings increase?

A. 2%
B. 4%
C. 5%
D. 7%
E. 10%

SOLUTION :

Let Mira's expenditure be 3x and savings be 2x. Hence, her income = 5x.
Her new expenditure and income after the increase are 3x(1.12)= 3.36x and 5x(1.1)=5.5x
respectively.

Hence, her new savings = (5.5x - 3.36x) = 2.14x.

0.14𝑥
So, increase in the savings = ( 2𝑥
) x 100 = 7

ANSWER D
4. Of the total amount received by Karan, 20% was spent on purchases and 5% of the remaining
on transportation. If he is left with Rs 1520 , the initial amount was?

A. 4500
B. 3000
C. 2500
D. 2000
E. 1800

SOLUTION :

Let 100 be the initial amount with Kiran. 20 is spent on purchases. Hence, we are left with 80.
Five percent of the remaining is spent on transport => 5% of 80 is 4. Hence, the remaining is 76.
We are given that the remaining is 1520.
Here, 76 corresponds to 1520.
100 × 1520
hence, 100 corresponds to : 76
= 2000.

ANSWER D
5. If prices decrease by 25%, by what percentage can consumption increase without affecting
the expenditure?

A. 25%
B. 20%
C. 15%
D. 33.33%
E. 40%

SOLUTION :

We know Expenditure = Price * Consumption


Assume the price to be 4. The new price would be 3.
This means we can assume the value of the Expenditure to be a multiple of 4 and 3 i.e 12.
Hence, Consumption increases from 3 to 4.
Increase of 1 over a base of 3 i.e 33.33%

ANSWER D
6. Since the price of mangoes decreased by 25%, I can purchase 4 more mangoes for Rs. 60.
What is the original price of one mango?

A. 5
B. 10
C. 12
D. 15
E. 20

SOLUTION :

Expenditure = Price * Consumption


We know that the Expenditure is 60.
Consumption of Mango be ‘C’
Old Price of Mango = M
New Price of Mango = 0.75M
=> M*C = 0.75M * (C + 4)
=> 4C = 3C +12
=> C = 12

Expenditure = Price * Consumption = 60 = M*C = M * 12 => M = 5

ANSWER A
7. If the area of the circle is increased by 21%, then what is the percentage increase in the
circumference of the circle?

A. 5%
B. 10%
C. 15%
D. 18%
E. 25%

SOLUTION :

Let r and r' be the old and new radii of the circle.
Let C and C' be the old and new radii of the circle.

Initial Area = 100


Final Area = 121
𝐹𝑖𝑛𝑎𝑙 𝑟' 2 121 𝑟' 11
𝐼𝑛𝑖𝑡𝑖𝑎𝑙
=( 𝑟
) = 100 => 𝑟
= 10

𝐶' 2π𝑟' 11
𝐶
= 2π𝑟
= 10

There’s an increase of 1 over a base of 10 i.e 10% increase

ANSWER B
8. The radius of a circle has increased by 20%. By what percentage does the circumference and
the area increase ?

A. 10%,10%
B. 10%,20%
C. 20%,20%
D. 40%,40%
E. 20%,44%

SOLUTION :

Circumference of a circle = 2π r.
Since r increases by 20%, the circumference also increases by 20%.
r = 5 , r’ = 6
𝐶' 6
𝐶
= 5

Circumference increases by 1 over a base of 5 i.e a 20% increase.

2
Area of a circle = π𝑟 .
r = 5 , r’ = 6
2 2
=> 𝑟 = 25 , 𝑟' = 36

𝐴' 36
𝐴
= 25

Area increases by 11 over a base of 25 i.e a 44% increase.

ANSWER E
9. Two numbers are respectively 19% and 70% more than the third number. What percentage is
the first number out of the second ?

A. 50%
B. 60%
C. 70%
D. 80%
E. 90%

SOLUTION :

Let the third number be 100.

Then, the first number is 100 + 19 = 119 and the second number is 170.
𝑓𝑖𝑟𝑠𝑡 119
𝑠𝑒𝑐𝑜𝑛𝑑
= 170
= 70%.

ANSWER C
10. Salaries of A, B and C are in the ratio 1:2:3. Salary of B and C together is Rs. 6,000. By what
percentage is the salary of C more than that of A ?

A. 100
B. 150
C. 180
D. 200
E. 250

SOLUTION :

Let A = x , B = 2x and C = 3x
Then, 2x+ 3x = 6000 ⇒ x = 1200
So, A = 1200 and C = 3600
2400
Hence, Salary of C is 2400 more than A i. e 1200
= 200% more

Alternative method:

If salary of A = x, then salary of C = 3x, hence, salary of C is 2x more than A.


2𝑥
So, C is 𝑥
× 100 = 200.

ANSWER D
11. In a physical measurement, by mistake, Shyam gave his height as 25% more than normal. In
the interview panel, he clarified that his actual height was 5 feet 5 inches. Find the percentage
correction made by the candidate from his stated height to his actual height.

A. 20%
B. 30%
C. 40%
D. 50%
E. 60%

SOLUTION :

Actual height = 5 feet 5 inches = 5 × 12 inches + 5 inches = 65 inches


125
Stated height = 100
× 65 inches = 81.25 inches

81.25 − 65 16.25
Required percentage error = 81.25
x 100 = 81.25
x 100 = 20

ANSWER A
12. A number is mistakenly divided by 2 instead of being multiplied by 2. Find the percentage
change in the result due to this mistake.

A. 100%
B. 125%
C. 200%
D. 75%
E. 150%

SOLUTION :

Let's say the number is 2.


Desired Number = 4
Mistaken Number = 1
The base will be the desired number
4−1 3
% change will be = 4
× 100 = 4
× 100 = 75

ANSWER D
PROFIT LOSS DISCOUNT

1. A man sells 2 items for Rs. 4,000 each, neither gaining nor losing in the deal. If he sells one
cow at a gain of 25%, then the other cow must have sold at a loss of

A. 16.66%
B. 18.22%
C. 25%
D. 30%
E. 20%

SOLUTION :

He sells one cow at a gain of 25% , if CP = 4 , SP = 5


CP : SP = 4 : 5 = x : 4000 [Let actual cost of one cow be x]
On solving , we get x = 3200

Given, Total S.P = Total C.P


=> 8000 = 3200 + CP of the other cow
=> CP of the other cow = 4800
1
So, there’s a loss of 800 over a base of 4800 i . e 6
=> 16.66%

ANSWER A
2. A salesman sold one-fourth of his stock at 50% profit and the remaining at 80% profit. Find
the net profit % he made on his total stock.

A. 72.5%
B. 80%
C. 50%
D. 65%
E. 40%

SOLUTION :

Let's say the salesman invested in 100 stocks worth Rs. 1 each.
So, Stocks worth Rs. 25 was sold at a 50% profit and thus he would have received Rs. 37.5
(50% of 25 = 12.5)
For the remaining stock worth Rs. 75, he would have received Rs. 75 (1 + 80%) = 75 (1.8)
Rs. 135.
Thus he receives a total of Rs. 172.5 (135 + 37.5) on an investment of Rs. 100, implying a profit
of 72.5%

ANSWER A
3. Aditya purchases toffees at Rs. 10 per dozen and sells them at Rs. 12 for every 10 toffees.
Find the gain or loss percentage.

A. 20%
B. 44%
C. 50%
D. 60%
E. 75%

SOLUTION :

10
Method 1: Cost price of 1 toffee = Rs. 12
[12 toffees for Rs 10 ]
12
Selling price of 1 toffee = Rs. 10
[10 toffees for Rs 12 ]
𝑆𝑃 144
𝐶𝑃
= 100
So, profit percentage = 44%. (Profit of 44 over a base of 100)

Method 2: Assume that he buys 120 toffees.


{ Assuming a multiple of 12 and 10 as he buys 12 toffees and sells 10 toffees }
Aditya spends Rs. 100. [ for 120 toffees.]
Revenue generated is Rs. 144. [ for 120 toffees.]

Hence, Profit Percentage = 44

ANSWER B
4. An off-season discount of x % is being offered at the discount store. An additional 12.5%
discount is given if the value of purchase is more than Rs. 500. After the discounts, a person
pays Rs. 525 for a jean whose list price is Rs. 750. Find the value of x.

A. 5
B. 10
C. 20
D. 25
E. 30

SOLUTION :

On 750 , we got a discount of x% . Then on the resulting value , we got an additional 12.5%
discount as the value of purchase was more than Rs. 500. Now, the final value is 525.

=> 750 (1 - x%) ( 1 - 12.5%) = 525


7
=> 750 (1 - x%) 8
= 525
=> (1 - x%) = 80%
=> x% = 20%
=> x = 20

ANSWER C
5. Inspite of giving a discount of 10%, a shopkeeper manages to make a profit of 8%. By what
percentage does the shopkeeper mark- up his goods ?

A. 5%
B. 10%
C. 15%
D. 20%
E. 30%

SOLUTION :

We need to find a relationship between the MP and CP so as to get the mark-up percentage.
SP = MP (1 - 10%) - - - - - - (2)
SP = CP (1 + 8%) - - - - - - - (3)
Equating (2) and (3) , we get
90%MP = 108%CP
=> MP = 1.2 CP
𝑀𝑃 6
=> 𝐶𝑃
= 5

Mark up of 1 over a base of 5 i.e a 20% markup.

ANSWER D
6. A shopkeeper gives 2 items free with every 3 items purchased. In effect, what discount
percentage is being offered ? [ Assume that all the items are identical ]

A. 20%
B. 25%
C. 40%
D. 50%
E. 66.66%

SOLUTION :

2
Please realize that the discount is not 3
or 66.66%.
Why ?
Discount percent is expressed as a percentage of marked or list price.
The list price of the 5 items will be 5x and the selling price will be 3x.
2
Thus , discount percent is 5
or 40%.

ANSWER C
7. A man bought 80 kg of rice for Rs. 88 and sold it at a loss of as much money as he received for
20 kg. At what price did he sell it ?

A. 60 paise
B. 70 paise
C. 75 paise
D. 80 paise
E. 88 paise

SOLUTION :

C.P. of 80 kg - S.P. of 80 kg = S.P. of 20 kg (GIVEN)


C.P. of 80 kg = S.P. of 80 kg + S.P. of 20 kg
C.P. of 80 kg = S.P. of 100 kg
=> 88 = S.P. of 100 kg
=> SP of 1 kg = Rs 0.88 or 88 paise

ANSWER E
8. A person earns 15% on an investment but loses 10% on another investment. If the ratio of
the two investments be 3: 5 respectively, what is the gain or loss on the two investments taken
together ?

A. 0.0625%
B. 0.625%
C. 6.25%
D. 62.5%
E. None of the above

SOLUTION :

Let the investments be 3x and 5x.


Then, the total investment = 8x

Total Revenue = (115% of 3x + 90% of 5x) = (3.45x + 4.5x) = 7.95x


0.05𝑥
Loss = 8𝑥
× 100 = 0.625

ANSWER B
SI CI

1. Suhit borrowed a sum of $6300 from Vikas at the rate of 14% per annum simple interest
for 3 years . He then added some more money to the borrowed sum and lent it to Mohit at the
rate of 16% of simple interest for the same time. If Suhit gained $618 in the whole transaction,
then what sum did he lend to Mohit?

A. $6000
B. $6800
C. $7200
D. $7500
E. $7800

SOLUTION :

The interest that Suhit has to pay Vikas after 3 years


=> 3 × 14% × 6300
The interest that Suhit will get when he lent the money to Mohit
=> 3 × 16% × 6300 + 3 × 16% × y [ y is the ‘some more money’ added by Suhit ]

So, we can say ‘The interest that Suhit will get’ - ‘The interest that Suhit has to pay’ = 618

On simplifying , we get
=> 3× 2% × 6300 + 3 × 16% × y = 618
=> 378 + 0.48y = 618
=> 63 + 0.08y = 103
=> y = 500

What sum did he lend to Mohit ? => 6300 + 500 => 6800

ANSWER B
2. If a sum of money invested under simple interest amounts to $3,200 in 4 years and $3,800 in
6 years , what is the rate at which the sum of money was invested ?

A. 10%
B. 12%
C. 15%
D. 20%
E. 24%

SOLUTION :

We know that the Simple Interest remains the same for all years if the rate is constant.
Since it is simple interest , we can look at the difference to get two years of interest.
Sum invested amounts to 3200 in 4 years and 3800 in 6 years would mean that the interest for
the 2 years is 600. So, the interest is 300 for 1 year.

Backtrack to 4 years before $3200, we started out with $2000 (3200 - 300×4) and we received
300
$300 a year, so the interest rate is 2000
= 15% .
( Formula Wise : SI = 300 , P = 2000 , n = 1, you can find R )

ANSWER C
3. Simple interest on a certain sum for 3 years at 8% per annum is equal to half the compound
interest on $4000 for 2 years at 10% per annum. What is the sum placed on simple interest?

A. $840
B. $1250
C. $1550
D. $1750
E. $1950

SOLUTION :

Let’s find the compound interest first. The interest for the first year of a $4000 investment at
10% per annum is 4000 x 0.1 = $400. The interest for the second year is (4000 + 400) x 0.1 =
$440 (due to compounding interest). Therefore, for the $4000 investment, the total interest for
the two years is 400 + 440 = $840. Half that value is $420.

We can now calculate the amount placed in simple interest.


We can let p = the principal for 3 years at 8% per annum in simple interest and create the
equation:

n x p x R% = Interest
3 x p x 8% = 420
On solving , we get
p = 1750

ANSWER D
4. A sum of $3310 is to be paid back in 3 equal installments. How much is each installment if the
interest is compounded annually at 10% per annum ? (CALCULATION INTENSIVE ; GOOD FOR
PRACTICE)

A. 1311
B. 1331
C. 1103
D. 2331
E. 2311

SOLUTION :

The approach to solve this question is:

Assume each installment = X


Initial value = 3310

End of 1st year, value = 3310(1.1) AND we pay the first installment, so remaining value =
3310(1.1) - X

End of 2nd year, value = [ 3310(1.1) - X ] (1.1) AND we pay the second installment, so remaining
value = [ 3310(1.1) - X ] (1.1) - X

End of 3rd year, value = { [ 3310(1.1) - X ] (1.1) - X } (1.1)

** KEY is the value at the end of 3rd year = X (the last installment)
=> { [ 3310(1.1) - X ] (1.1) - X } (1.1) = X

Take all the variables to the right and numbers to the left and solve.
10 21
=> 3310(1.1)(1.1) = X ( 11
+ 10
)

On solving , we get

=> X = 1331

ANSWER B
5. Samantha purchased a five-year municipal bond that pays simple interest for $600. After the
bond matured five years later, she had received a total of $720 in principal plus interest
payments. What is the interest rate of Samantha's bond?

A. 4%
B. 5%
C. 10%
D. 12%
E. 20%

SOLUTION :

Purchase price = 600


Final price received = 720

Interest value = 720 - 600 = 120 for 5 years

So , interest will be 24 for 1 year. Principal = 600


SI = n P R%
=> 24 = 600 (R%)
1
=> R% = 25
= 4%

ANSWER A
6. What amount, in dollars, invested for one year at an interest rate of 2 percent compounded
semiannually would produce the same final balance at the end of the year as $10,000 invested
for one year at an interest rate of 4 percent compounded quarterly?

A. $10,000
B. $10,201
C. $10,500
D. $10,801
E. $20,000

SOLUTION :

Let the amount be x dollars

2% 2 4% 4
x (1 + 2
) = 10000 (1 + 4
)
2 4
x (1. 01) = 10000 (1, 01)
2
x = 10000(1. 01) = $10,201

ANSWER B
7. The simple interest for 10 years is Rs. 6,000. The compound interest for 2 years is Rs. 1,400.
Find the rate of interest and the principal.

A. 30%, 1,800
1
B. 33 3 %, 1,600
1
C. 33 3 %, 1,800
D. 35%, 1,500
2
E. 66 3 %, 1,600

SOLUTION :

We know that SI remains the same for all years.


If SI is Rs.6000 for 10 years , it will be Rs.600 for every year.
We also know that SI for the 1st year is equal to CI for the 1st year, which is 600.

So , CI for the 2nd year = 800 as CI for 2 years is 1400.


CI grows every year with the same rate as that of rate of interest.
How much interest grew from 1st year to 2nd year?
From Rs 600 to Rs. 800, an increase of Rs 200 over a base of Rs. 600, which is a 33.33% increase.

Now, we know that R = 33.33 or , SI for 1 year = 600 , n = 1 year


1𝑃
SI = 𝑛𝑃𝑅% => 600 = 3
=> P = 1800

ANSWER C
8. An amount of X dollars is invested at R% simple annual interest and yields a total interest of
$100 in 2 years. In terms of X, what dollar amount invested at the same rate of interest will yield
a total interest of $400 in 4 years?

𝑋
A. 2
B. X
3𝑋
C. 2
D. 2X
5𝑋
E. 2

SOLUTION :

100
if 100$ is in 2 years, then 2
= 50$ per year.
SI = 𝑛𝑃𝑅% => 50 = xR% - - - (1)
400
If 400$ in 4 years then 4
=100$ per year.
Let the amount invested be y$.
SI = 𝑛𝑃𝑅% => 100 = yR% - - - (2)

Dividing Equation (1) by Equation (2) , we get


1 𝑥
=> 2
= 𝑦
=> y = 2x

ANSWER D
Ratios and Mixtures

1. From a 100 ml can full of milk, 10 ml is removed and 20 ml water is added. Next 20 ml of the
solution is removed and 30 ml water is added. What is the quantity of milk in the solution now?

800
A. 11
ml
810
B. 11
ml
820
C. 11
ml
830
D. 11
ml
840
E. 11
ml

SOLUTION :

100 ml milk , 10 ml is removed => 100 - 10 = 90 ml milk


20 ml water is added => 90ml(milk) + 20 ml(water) = 110ml solution ( M : W = 9 : 2)

20 ml of the solution is removed from 110ml solution ( M : W = 9 : 2)


In this 20 ml of solution , M and W will be removed in the same ratio (9:2)
20
So 9k + 2k = 20 => 11k = 20 => k = 11
180
Milk removed = 9k = 11
ml
40
Water removed = 2k = 11 ml
We are now left with 90 ml of solution
180
Milk left = 90 - 11
40
Water left = 20 - 11
Now, 30 ml of water is added.So, the final volume is 120 ml.

180 810
Milk left = 90 - 11
= 11 ml
40 510
Final Water = 20 - 11 + 30 = 11
ml
810
Hence, In the 120ml solution, the quantity of milk is 11

ANSWER B
2. There are three vessels of equal capacity which are full . Vessel A contains Milk and water in
the ratio 4:3 ,Vessel B contains Milk and water in the ratio 2:1 and Vessel C contains Milk and
water in the ratio 3:2 .If the mixture in all the three vessels is mixed up ,what will be the ratio of
milk and water ?

A. 9:6
B. 3:2
C. 118:126
D. 193:122
E. 201:132

SOLUTION :

The three vessels are of equal capacity . The milk : water ratio is given for each as 4:3, 2:1 and
3:2. The reason we cannot just add the milk parts (4+2+3) and water parts (3+1+2) together is
that they represent different fractions of the whole volume i.e. in the first vessel, milk is 4 parts
out of a total of 7 parts while in the second vessel, milk is 2 parts out of a total of 3 parts .

If we make the total number of parts in each ratio equal, then we can just add the 'milk parts'
together and all the 'water parts' together.

Using Ratios:
4:3 (Total 7 parts)
2:1 (Total 3 parts)
3:2 (Total 5 parts)
How can we make the total number of parts equal in the 3 cases ? By taking the LCM.
LCM of 7,3, and 5 is 105.

4 × 15 : 3 × 15 = 60:45 (Total parts 105)


2 × 35 : 1 × 35 = 70:35 (Total parts 105)
3 × 21 : 2 × 21 = 63:42 (Total parts 105)

When we mix the three solutions, the total number of parts of milk is 60+70+63 = 193
the total number of parts of water = 45+35+42 = 122

The required ratio = 193:122

ANSWER D
3. As a result of a chemical reaction between compounds A and B , 1/4 of each compound was
transformed into an equal amount of the other compound, while the remaining 3/4 of each
compound was not transformed. If the amount of B was initially 4 times the amount of A , what
was the ratio of the amount of A to the amount of B after the reaction?

A. 1 to 3
B. 3 to 4
C. 4 to 7
D. 7 to 13
E. 7 to 16

SOLUTION :

Before reaction
Say A = 20 units , B = 80 units

During reaction
One-fourth of A transformed into B (A => B) = 5 units
One-fourth of B transformed into A (B => A) = 20 units

Remaining A = 15 units , Remaining B = 60 units

After reaction
A = (Remaining A) + (B => A) = 15 + 20 = 35
B = (Remaining B) + (A => B) = 60 + 5 = 65
Required Ratio = 35 : 65 = 7 : 13

ANSWER D
4. In Smithtown, the ratio of right-handed people to left-handed people is 3 to 1 and the ratio of
men to women is 3 to 2. If the number of right-handed men is maximized , then what percent of
all the people in Smithtown are left-handed women?

A. 50%
B. 40%
C. 25%
D. 20%
E. 10%

SOLUTION :

As the total is a multiple of 4 and 5 (see the ratios and the total in the table) , assume the value
of total to be 20.So, a = 5 , b = 4.
We need to maximize Right handed men , so we can say the left handed men will be minimum
i.e 0. So, Right handed men = 12. This gives Right handed women as 3 and Left handed women
as 5 respectively.

Right handed Left handed

Men 12 0 3b = 12

Women 3 5 2b = 8

3a = 15 1a = 5 Total = 4a = 5b = 20

5
% of Left handed women out of the total = 20
= 25%
ANSWER C
5. Rice of two different varieties A and B are mixed together, by a seller, in the ratio of 4: 7.
Upon selling the mixture at the price of $17 per kg, the seller incurred a loss of 15%. If the cost
price of B is $22 per kg, what is the cost price per kg of A?

A. 15
B. 16.5
C. 17.75
D. 18
E. 19

SOLUTION :

Upon selling the mixture at the price of $17 per kg, the seller incurred a loss of 15%.
Let Cost price of the mixture be $x per kg.
=> SP = CP (1 -15%)
=> 17 = x (0.85)
=> x = 20

Given, the cost price of B is $22 per kg. Let the cost price of A be $P per kg.
Let, variety A be 4y kg and B be 7y kg in quantity, then,

***Rice of 4y kg having a cost price of $P per kg is mixed with Rice of 7y kg having a cost price of
$22 per kg and the overall cost of the mixture becomes $20 per kg.***

=> 4y × P + 7y × 22 = 20 (7y+4y)
=> 4P = 20 × 11 - 154
66
=> P = 4
=> P = 16.5

ANSWER B
6. The ratio of x to y to z is 3 : 4 : 12. If the ratio is changed so that the ratio of x to z is doubled
and the ratio of y to z is halved, which of the following is the new ratio?

A. 6 : 2 : 24
B. 6 : 1 : 24
C. 6 : 1 : 12
D. 3 : 1 : 6
E. 3 : 8 : 6

SOLUTION :

x : z = 3 : 12 = 1 : 4
This ratio is doubled , so the new ratio will be x : z = 2 : 4
y : z = 4 : 12 = 1 : 3
This ratio is halved , so the new ratio will be y : z = 1 : 6

In order to make the value of z the same in both the new ratios , multiply the first by 3 and the
second by 2.
Hence , x : z = 6 : 12 and y : z = 2 : 12
So , x : y : z = 6 : 2 : 12 = 3 : 1 : 6

ANSWER D
7. At a certain university, the ratio of the number of teaching assistants to the number of
students in any course must always be greater than 3:80. At this university , what is the
maximum number of students possible in a course that has 5 teaching assistants?

A. 130
B. 131
C. 132
D. 133
E. 134

SOLUTION :

3
Assistants : Students => 3 : 80 => 80
We need to maximize the Students

𝐴𝑠𝑠𝑖𝑠𝑡𝑎𝑛𝑡𝑠 3
GIVEN : 𝑆𝑡𝑢𝑑𝑒𝑛𝑡𝑠
> 80

5 3
=> 𝑆𝑡𝑢𝑑𝑒𝑛𝑡𝑠
> 80

400
=> Students < 3
=> Students < 133.33

Hence, maximum number of students = 133

ANSWER D
8. If A : B = 3 : 4 , B : C = 8 : 9 , C : D = 15 : 16 , find A: B: C: D.

A. 10 : 20 : 30 : 48
B. 20 : 40 : 45 : 48
C. 30 : 40 : 45 : 64
D. 30 : 40 : 45 : 48
E. None of the above

SOLUTION :

A : B = 3 : 4 , B : C = 8 : 9 , C : D = 15 : 16
Multiplying A : B by 2 , we get A : B = 6 : 8

So , A : B : C = 6 : 8 : 9

As C is 9 in the above ratio and 15 in the ratio C : D , we need to multiply the above ratio by 5
and C : D by 3 so that the value of C becomes equal in both the ratios.
Hence, A : B : C = 30 : 40 : 45 , C : D = 45 : 48

Hence, A : B : C : D = 30 : 40 : 45 : 48

ANSWER D.
9. A precious stone worth Rs. 6,800 is accidentally dropped and breaks into 3 pieces. The
weights of these 3 pieces are in the ratio 5:7:8. The value of the stone is proportional to the
square of its weight. Calculate the loss in the values, if any , incurred because of the breakage.

A. Rs 2344
B. Rs 3144
C. Rs 3554
D. Rs. 4454
E. Rs 5544

SOLUTION :

Let the weight of 3 pieces be 5g, 7g, and 8g.


So the total weight of the unbroken stone : 5 + 7 + 8 = 20g
2
The price of the stone is directly proportional to its 𝑤𝑒𝑖𝑔ℎ𝑡 .
2
So, 6800 = k x 20 hence k = 17
2 2 2
So the values of the broken pieces are k x 5 , k x 7 , k x 8 , i.e. Rs. 425, Rs. 833 and Rs. 1,088
respectively.
Hence the total value of broken pieces = (425 + 833 + 1088) = Rs. 2,346.

So the total loss = 6800 - 2346 = Rs. 4,454.

ANSWER D
10. In two alloys the ratios of copper to tin are 3 : 4 and 1 : 6 respectively. If 7 kg of the first
alloy and 21 kg of the second alloy are mixed together to form a new alloy, then what will be the
ratio of copper to tin in the new alloy ?

A. 11 : 3
B. 3:8
C. 3 : 11
D. 3 : 14
E. 11 : 14

SOLUTION :

1st Alloy , C : T = 3k : 4k , Total = 7 = 7k => k = 1


Copper = 3kg , Tin = 4kg

2nd Alloy , C : T = 1m : 6m , Total = 21 = 7m => m = 3


Copper = 3kg , Tin = 18kg

When these alloys are mixed together,


Copper = 6kg , Tin = 22 kg
Ratio of Copper : Tin = 3 : 11

ANSWER C
11. 5 liters of 30% alcohol solution is mixed with 10 liters of 45% alcohol solution. What is the
concentration of the resultant solution ?

A. 35%
B. 38%
C. 40%
D. 42%
E. 44%

SOLUTION :

Say when these solutions are mixed together , we get 15 liters of x% alcohol solution.

This means that Alcohol in 1st solution + Alcohol in the 2nd solution is equal to the Alcohol in
the resultant solution.

=> 30% of 5 + 45% of 10 = x% of 15


=> 1.5 + 4.5 = x(0.15)
=> x = 40

ANSWER C
12. How many kilograms of salt costing Rs. 42 per kilogram must a man mix with 25 kilograms
of salt costing Rs. 24 per kilogram, such that on selling the mixture at Rs. 40 per kilogram, there
is a gain of 25% on the outlay ?

A. 10 kg
B. 12 kg
C. 15 kg
D. 18 kg
E. 20 kg

SOLUTION :

Selling Price of the mixture per kg = Cost Price of the mixture per kg (1 + 25%)
5
=> 40 = CP ( 4 )
=> CP = 32

Say x kg of salt costing Rs. 42 per kg is mixed with 25 kg of salt costing Rs. 24 per kg which gives
rise to x + 25 kg of salt costing Rs 32 per kg.

=> 42x + 24*25 = 32*(x + 25)


=> 10x = 200
=> x = 20

ANSWER E
Arithmetic Part 2 Extra Question Solutions

Time Work

1. Pump A, pumping water at a constant rate, can fill a certain swimming pool in 6 hours. Pump
B, pumping water at a constant rate, can fill the same pool in 4 hours. If both pumps begin
filling the pool simultaneously when the pool is empty and pump B breaks down 1 hour after
they begin filling the pool, how many hours will it take pump A alone to finish filling the pool ?

A. 1.4 hrs
B. 2.3 hrs
C. 3.5 hrs
D. 4 hrs
E. 4.8 hrs

SOLUTION :

Let's assume that the pool holds a capacity of 24 units of water.

Rate of Pump A = 4 units/hour


Rate of Pump B = 6 units/hour
Combined Rate = (6+4) = 10 units/hour

In the first hour, when both the pumps work together, the amount of water pumped = 10 units

Units of water remaining to be pumped = 24−10 = 14

Time taken by Pump A alone to finish filling the pool = 14/4 = 7/2 = 3.5 hours

ANSWER C
2. Working alone, Printers X, Y, and Z can do a certain printing job, consisting of a large number
of pages, in 12, 15, and 18 hours, respectively. What is the ratio of the time it takes Printer X to
do the job, working alone at its rate, to the time it takes Printers Y and Z to do the job, working
together at their individual rates?

4
A. 11
1
B. 2
15
C. 22
22
D. 15
11
E. 4

SOLUTION :

If you want to avoid fractions, assume the total number of pages to be 180.

X does 180/12 = 15 pages/hr


Y does 180/15 = 12 pages/hr
Z does 180/18 = 10 pages/hr

Y and Z together do 12+10 = 22 pages/hr

Ratio of speed of X : speed of Y and Z combined = 15:22


Ratio of time taken by X : time taken by Y and Z combined = 22:15
(time taken varies inversely with speed when distance or work is constant)

ANSWER D­
3. In order to complete a reading assignment on time, Terry planned to read 90 pages per day.
However, she read only 75 pages per day at first, leaving 690 pages to be read during the last 6
days before the assignment was to be completed. How many days in all did Terry have to
complete the assignment on time?

A. 15
B. 16
C. 25
D. 40
E. 46

SOLUTION :

We are given that Terry’s initial plan was to read 90 pages per day for “T” days. We can multiply
and get 90T for the total work that was supposed to be done for the entire reading assignment.
We are next told that she actually started out reading 75 pages per day until she had 6 days left
to finish the project. Since T represents the total number of days we can say she read 75 pages
per day for (T–6) days. Finally, when we multiply, we see that her work done on the T–6 days
was 75T–450.

To finish this problem we need to set up one final equation. We are told that she must read 690
pages in the last 6 days to complete the assignment. From the chart we see that 90T denotes a
completed assignment. Thus we can say:
75T – 450 + 690 = 90T
75T + 240 = 90T
15T = 240
T = 16

ANSWER B
4. A team of 10 workers can paint the wall in 8 days. After working 2 days, x additional workers
joined the team and together, they finished the job in 4 days. What is the value of x ?
(Rate of each worker remains the same)

A. 5
B. 6
C. 12
D. 4
E. 9

SOLUTION :

Let the rate of each worker is 1 unit per day . So , Total work = 8×10 = 80 units

Work done in 2 days by 10 workers = 20 units

Work left = 80-20 = 60 units

Additional x workers added to complete left work in 4 days.

=> (10 + x) × 4 = 60

=> 4x = 60-40

=> 4x = 20

=> x = 5

ANSWER A
5. If 20 diggers, each working at the same constant rate, can dig a 100-meter long trench in hard
soil in 3 days, how many diggers are needed to dig a 180-meter long trench in soft soil in the
same 3 days, given that a digger is 20% more productive in soft soil than in hard soil?

A. 25
B. 28
C. 29
D. 30
E. 32

SOLUTION :

Let the rate of each digger is M.


time × rate = work
3 × (20 M) = 100
5
M= 3

How many diggers are needed to dig a 180-meter long trench in soft soil in the same 3 days,
given that a digger is 20% more productive in soft soil than in hard soil ?

Say we need y diggers.

time × rate = work

3 × (y M)(1 + 20%) = 100

6
3y×M× 5 = 100

5
Putting M = 3
gives y = 30

ANSWER D
6. A cistern is filled by an inlet valve in 5 hours. However because of a leak, it takes 30 minutes
more to fill up. In how much time will the leak empty the filled cistern independently ?

A. 50 hrs
B. 55 hrs
C. 60 hrs
D. 65 hrs
E. 70 hrs

SOLUTION :

Looking at the times provided , we can assume the work to be 55 units.

Hence, Rate of inlet comes out to be 11 units/hr


(Rate of Inlet + Leak) = 10
=> 11 units/hr + Rate of the Leak = 10
=> Rate of the Leak = 10 - 11 = -1 units / hr

time rate work

Inlet 5 hrs 11 55 units

Inlet + Leak 5.5 hrs 10 (+11 - 1) 55 units

Leak y -1 55 units

y = 55 hrs.

Hence , The leak will empty the filled cistern independently in 55 hrs

ANSWER B
7. A force of 60 men has food for 28 days. 8 days later , reinforcements arrive leaving the
number of days the food would last to 15 days. What was the strength of the reinforcement ?

A. 5
B. 10
C. 16
D. 20
E. 24

SOLUTION :

Originally the food would have lasted for 28 days.


After 8 days, the food would have lasted for 20 days.
Let the reinforcement number be x.
The food(Work) that would have been consumed by 60 men in 20 days, was consumed by
(60 + x) men in 15 days.

Let each man consume 1 unit per day.


60 men consume 60 units per day.
(60 + x) men consume (60 + x) units per day.

Work remains constant.

time 1 × rate 1 = time 2 × rate 2


20 × 60 = 15 × (60 + x)
x = 20.

The strength of the reinforcement was 20.

ANSWER D
8. A and B can separately do a piece of work in 20 and 15 days respectively. They worked
together for 6 days, after which B was replaced by C. If the work was finished in the next 4 days,
then find the number of days in which C alone could do the work.

A. 20
B. 30
C. 35
D. 40
E. 45

SOLUTION :

Assume the work to be a multiple of 20 and 15 i.e 60 units.

time rate work

A 20 3 60

B 15 4 60

A+B 6 (3 + 4) 42

A+C 4 (3 + x) Remaining work = 18

time * rate = Work => 4 * (3 + x) = 18 => x = 1.5 units/day

𝑊𝑜𝑟𝑘 60
time taken by C working alone = 𝑟𝑎𝑡𝑒
= 1.5
= 40 days

ANSWER D
9. Three men can complete a piece of work in 6 days. Two days after they started, 3 more men
joined them. How many days will they take to complete the remaining work ?
( Assume they all have equal efficiency )

A. 0.25
B. 0.50
C. 1
D. 2
E. 3

SOLUTION :

Rate of each men be M units per day

time rate work

Three men 6 3M 18M

2 days working 2 3M 6M

Six men y 6M Remaining work =


12M

time × rate = work

y(6M) = 12M => y = 2 days

ANSWER D
10. 2 men and 3 boys can do a piece of work in 10 days, while 3 men and 2 boys can do the
same work in 8 days. In how many days can 2 men and 1 boy do the work ?

A. 10
B. 11
C. 12
D. 12.5
E. 15

SOLUTION :

Let the total work be LCM(10, 8) = 40 units.

Let work done per day by 1 man = x units

Let work done per day by 1 boy = y units

2 men and 3 boys can do a piece of work in 10 days (Rate will be 4 units/day)
So, 2x + 3y = 4 [ We are equating the rates here ]

3 men and 2 boys can do the same work in 8 days (Rate will be 5 units/day)
So, 3x + 2y = 5 [ We are equating the rates here ]

7 2
On solving, we get, x = 5
and y = 5

7 2 16
Rate of (2 men + 1 boy) = 2 × 5
+1 × 5
= 5
units

𝑊𝑜𝑟𝑘 40 200 25
Number of days taken to do the work = 𝑟𝑎𝑡𝑒
= 16 = 16
= 2
days
5

ANSWER D
Time Speed Distance

1. A predator is chasing its prey. The predator takes 4 leaps for every 6 leaps of the prey and the
predator covers as much distance in 2 leaps as 3 leaps of the prey. What is the ratio of the speed
of the predator to that of the prey ?

A. 11 : 9
B. 10 : 9
C. 1:1
D. 9 : 10
E. 9 : 11

SOLUTION :

Assuming one leap of the predator is x units and one leap of the prey is y units, then:
In 4 leaps, the predator covers the distance of 4x units.
In 6 leaps, the prey covers the distance of 6y units.
Note that the predator and the prey are traveling these distances in the same amount of time.

Since it is also given that the predator covers as much distance in 2 leaps as the prey does in 3
leaps, we get 2x = 3y.
Substituting 2x = 3y in the above, we get 6y = 2(3y) = 4x.
Thus, in the same time period, both the predator and the prey cover the same distance of 4x
units.
As time = constant , Ratio of distances is same as the ratio of speeds
i. e 4x : 6y => 4x : 4x => 1 : 1

ANSWER C
2. Airplane A flew against a headwind a distance of 900 miles at an average speed of (s - 50)
miles per hour. Airplane B flew the same route in the opposite direction with a tailwind and
traveled the same distance at an average speed of (s + 50) miles per hour. If Airplane A’s trip
took 1.5 hours longer than Airplane B’s trip, how many hours did Airplane B’s trip take?

A. 1.5
B. 2
C. 2.5
D. 3
E. 3.5

SOLUTION :

Airplane A’s trip took 1.5 hours longer than Airplane B’s trip

900 900
=> 𝑠−50
= 1.5 + 𝑠+50
900 900
=> 𝑠−50
- 𝑠+50
= 1.5
Divide the equation by 900.
1 1 1
=> 𝑠−50
- 𝑠+50
= 600
2
=> 𝑠 - 2500 = 60000
2
=> 𝑠 = 62500 => s = 250

How many hours did Airplane B’s trip take ?

900 900
Time taken = 250 + 50
= 300
= 3 hrs

ANSWER D
3. If a man walks at the rate of 5 kmph, he misses a train by 7 minutes. However, if he walks at
the rate of 6 kmph, he reaches the station 5 minutes before the departure of the train. Find the
distance to the station.

A. 2 km
B. 3 km
C. 4 km
D. 5 km
E. 6 km

SOLUTION :

distance = constant
Ratio of speeds = s1 : s2 = 5 : 6
Ratio of times = t1 : t2 = 6 : 5 (Based on the ratio , the time difference is 1)

In the first case , the man misses the train by 7 min i.e he take 7 min more than the actual time
t. Say he takes t + 7 min

In the second case , the man reaches 5 min early i.e he take 5 min less than the actual time t.
Say he takes t - 5 min

What’s the actual time difference ? => (t + 7) - (t - 5) => 12 min

Ratio of times = t1 : t2 = 6 : 5 (Based on the ratio , the time difference is 1)

When the time difference is 1 , t2 is 5.


When the time difference is 12 , t2 will be 5×12 = 60 min = 1 hr.
So , distance = Speed 2 × time 2 = 6 × 1 = 6 km

ANSWER E
4. A car traveled 75% of the way from town A to town B by traveling at T hours at an average
speed of V mph. The car travels at an average speed of S mph for the remaining part of the trip.
Which of the following expressions represents the average speed for the entire trip ?

A. 0.75V + 0.25S
B. 0.75T + 0.25S
𝑉𝑇
C. (3𝑆)

4𝑉𝑇
D. (𝑇+𝑆)
3

4𝑉𝑆
E. (3𝑆+𝑉)

SOLUTION :

Let the total distance be d miles


The distance traveled at V mph is TV. ( Time taken is T for 75% of the stretch.)
Since this is 75% of the total distance, 0.75d = TV => d = 4VT/3

Furthermore, the remaining distance = 0.25d = VT/3 miles are traveled at a speed of S mph, so
𝑟𝑒𝑚𝑎𝑖𝑛𝑖𝑛𝑔 𝑑𝑖𝑠𝑡𝑎𝑛𝑐𝑒
time taken = 𝑠𝑝𝑒𝑒𝑑
= (VT/3)/S = VT/(3S) hours are spent on traveling at S mph.

𝑡𝑜𝑡𝑎𝑙 𝑑𝑖𝑠𝑡𝑎𝑛𝑐𝑒
Since average speed = 𝑡𝑜𝑡𝑎𝑙 𝑡𝑖𝑚𝑒
, we have:

Average speed = (4VT/3) / (T + VT/(3S))

Average speed = (4VTS) / (3ST + VT)

4𝑉𝑆
Average speed = 3𝑆 + 𝑉

ANSWER E
5. Sarah is traveling from City A to City B. The distance between these two cities is 185 miles,
and she planned to arrive at City B in 2 hours and 30 minutes. Her average speed for the first 1
hour and 40 minutes was 75 miles per hour, but then she realized that she should slow down to
drive more safely. If she still wanted to arrive at her destination on time, at what speed should
she drive ?

A. 60.0 miles per hour


B. 64.0 miles per hour
C. 68.0 miles per hour
D. 70.5 miles per hour
E. 72.0 miles per hour

SOLUTION :

5
Sarah wants to cover 185 miles in 2 hours and 30 minutes i.e 2
hours.

5 5
In 1 hour and 40 minutes, or 3
hours, at 75 miles per hour, Sarah would have covered 3
× 75 =
125 miles.

5 5 5
To cover the remaining 185 - 125 = 60 miles, in exactly ( 2 hours - 3
hours) = 6
hours, she
𝑅𝑒𝑚𝑎𝑖𝑛𝑖𝑛𝑔 𝑑𝑖𝑠𝑡𝑎𝑛𝑐𝑒 60
must average = 𝑅𝑒𝑚𝑎𝑖𝑛𝑖𝑛𝑔 𝑡𝑖𝑚𝑒
= 5 = 72 miles per hour.
6

ANSWER E
6. A turtle climbed to the top of a plateau at a rate of 4 miles an hour, crossed the plateau at a
2
rate of x miles per hour, and descended the other side of the plateau at a rate of 𝑥 miles per
hour. If each portion of the journey was equal in distance, what was the turtle's average speed
for the entire trip, in terms of x ?

2𝑥
A. 𝑥+2

2
3
B. (𝑥 + 2)

2
C. (𝑥 + 2)

2
4𝑥
D. 2
(𝑥+2)

2
12𝑥
E. 2
(𝑥+2)

SOLUTION :

If equal distances are given , we can set the distance of each portion as 1 mile for convenience.

2
1 1 1 𝑥 + 4𝑥 + 4
Then the total distance is 3 miles, the time taken in total was 4
+ 𝑥
+ 2 = 2
𝑥 𝑥

2
𝑡𝑜𝑡𝑎𝑙 𝑑𝑖𝑠𝑡𝑎𝑛𝑐𝑒 3 12𝑥
Then the average speed is = 𝑡𝑜𝑡𝑎𝑙 𝑡𝑖𝑚𝑒
= 2
𝑥 + 4𝑥 + 4
= 2
2
(𝑥+2)
𝑥

ANSWER E
7. Amitabh sees Uma standing at a distance of 200 m from his position. He increases his speed
by 50% and hence takes 20 s now to reach her. What was his original speed (in km/hr)?

A. 10
B. 12
C. 16
D. 24
E. 30

SOLUTION :

Distance = Constant

Speed Initial : Speed Final = 2 : 3

Time Initial : Time Final = 3 : 2

2 corresponds to 20 secs , so 3 corresponds to 30 secs.


We know distance = 200m , Time Initial = 30 secs

𝐷𝑖𝑠𝑡𝑎𝑛𝑐𝑒 200 20 18
Original speed = 𝑡𝑖𝑚𝑒 𝑖𝑛𝑖𝑡𝑖𝑎𝑙
= 30
m/s or 3
× 5
= 24 km/hr.

ANSWER D
5
8. A train met with an accident 150 km from station A. It completed the remaining journey at 6
of the previous speed and reached 15 min late at station B. Had the accident taken place 30 km
further, it would have been only 7 min late. Find the distance between the two stations A and B.

A. 170 km
B. 180 km
C. 192.25 km
D. 200.50 km
E. 206.25 km

SOLUTION :

Equation Method: Let distance between site of accident and station B be x km and let the usual
speed of the train be s kmph.

150 + 𝑥
Usual time taken from Station A to Station B = 𝑠

150 𝑥
But because of accident time taken = 𝑠
+ 5
6
𝑠

180 𝑥−30
Had accident occurred 30 km further, time taken = 𝑠
+ 5
6
𝑠

(Time taken due to accident at 150 km) - (Usual time taken) = 15 min [GIVEN]
150 𝑥 150 + 𝑥 15
We know that ( 𝑠
+ 5 ) -( 𝑠
)= 60
………………………. (i)
6
𝑠

and

(Time taken due to accident at 180 km ) - (Usual time taken) = 15 min [GIVEN]

180 𝑥−30 150 + 𝑥 7


( 𝑠
+ 5 )-( 𝑠
)= 60
………………………. (ii)
6
𝑠

Subtracting (ii) from (i), we have ,

30 30 8 36 30 8
5 - 𝑠
= 60
⇒ 𝑠
- 𝑠
= 60
⇒ s = 45 kmph
6
𝑠
Substituting s in either (i) or (ii), x can be found out. x comes out to be 56.25km.
Hence, distance between the two stations A and B = 150 + 56.25 = 206.25 km

This problem is a classic case of why students find TSD difficult, as it sometimes involves
cumbersome equations.

HINT FOR ALTERNATE SOLUTION : This question can be solved a bit easily using the application
of distance = constant. (Ratio of times will be inverse of Ratio of speeds)

ANSWER E
9. A runs 25% faster than B and allows B a lead of 7 meters. The race ends in dead heat. What is
the length of the race?

A. 10 meters
B. 15 meters
C. 25 meters
D. 35 meters
E. 45 meters

SOLUTION :

Speed of A : Speed of B = 5 : 4

This question is an application of time = constant


The ratio their speeds is 5:4 ; therefore the ratio of distance covered by them will also be 5:4
Length of the race equals the distance traveled by A.
When the difference between distances is 1 , A traveled 5.
When the difference between distances is 7 meters , A will travel 5×7 meters = 35 meters.

ANSWER D
10. A and B start swimming simultaneously from opposite ends, X and Y respectively of a
swimming pool of length 200 meters. A crosses B, reaches the opposite end Y and turns back
immediately and again meets B at a distance of 120 meters from X. Find the ratio of the speeds
of A and B if B has not yet reached the opposite end even once.

A. 7:2
B. 4:1
C. 6:4
D. 5:3
E. 9:5

SOLUTION :

Since A and B start simultaneously, when they meet, they would have swum for equal amounts
of time and thus the ratio of speeds is equal to the ratio of the distances.

When they meet after A reaches Y and turns back , A has swum 200 + (200 - 120) = 280 meters
and B has swum 80 meters and thus the ratio of the distances is 280 : 80 i.e. 7:2.

Hence , Ratio of Speeds = 7 : 2

ANSWER A
11. A train X starts from Meerut at 4 p.m. and reaches Ghaziabad at 5 p.m.while another train Y
starts from Ghaziabad at 4 p.m. and reaches Meerut at 5:30 p.m. The two trains will cross each
other at

A. 4:36 p.m.
B. 4:42 p.m.
C. 4:48 p.m.
D. 4:50 p.m.
E. 4:52 p.m.

SOLUTION :

Assume the distance to be 150 km.

X takes 1 hr to cover the distance that Y covers in 1.5 hrs.

Speed of X = 150 km/hr

Speed of Y = 100 km/hr

As both the trains are traveling from opposite ends and at the same time ,

150 3
The meeting time = 150 + 100
= 5
hr = 36 min

Hence , The two trains will cross each other at 4 : 36 pm.

ANSWER A
Set Theory

1. According to a survey, 7 percent of teenagers have not used a computer to play games, 11
percent have not used a computer to write reports, and 95 percent have used a computer for at
least one of the above purposes. What percent of the teenagers in the survey have used a
computer both to play games and to write reports?

A. 13%
B. 56%
C. 77%
D. 87%
E. 91%

SOLUTION :

Please convert all the “COMPLEMENTARY” information.

Given: 7 percent of teenagers have NOT used a computer to play games.


So, The percentage of teenagers who have used a computer to play games is 93%

Since 11 percent have NOT used a computer to write reports,


the percentage of teenagers who have used a computer to write reports is 89%

It has been given that teenagers who have used the computer for at least one of
the purposes = 95%
(making the number of teenagers not using computers for either of the purposes / none = 5%)

(Total) = (Play games) + (Write Reports) - (Both) + (None)


100% = 93% + 89% - x + 5%
x = 87%

Therefore, the percentage of teenagers who use the computer for both purposes is 87%

ANSWER D
2.

In a poll, 200 subscribers to Financial Magazine X indicated which of five specific companies
they own stock in. The results are shown in the table above. If 15 of the 200 own stock in both
IBM and AT&T, how many of those polled own stock in neither of the companies ?

A. 63
B. 93
C. 107
D. 122
E. 137

SOLUTION :

(IBM) = 48
(AT&T) = 30
Both = 15
At least 1 = (IBM OR AT&T ) = 48 + 30 - 15 = 63
We know that Total = none + At least 1
=> 200 = none + 63
=> none = 137

ANSWER E
3. In a certain region, the number of children who have been vaccinated against rubella is twice
the number who have been vaccinated against mumps. The number who have been vaccinated
against both is twice the number who have been vaccinated only against mumps. If 5,000 have
been vaccinated against both, how many have been vaccinated only against rubella?

A. 2,500
B. 7,500
C. 10,000
D. 15,000
E. 17,500

SOLUTION :

Use a single variable as far as possible.

For more simplicity , draw a venn diagram and solve.

Number vaccinated ONLY against mumps = x

Number vaccinated against BOTH = 2x = 5000 (so x = 2500)

Then, number vaccinated against mumps (including both) = x + 2x = 3x

Number vaccinated against rubella = 2×3x = 6x (GIVEN)

Then, number vaccinated against only rubella = Rubella - BOTH = 6x - 2x = 4x = 4(2500) = 10000

ANSWER C
4. In the city of San Durango, 60 people own cats, dogs, or rabbits. If 30 people owned cats, 40
owned dogs, 10 owned rabbits, and 12 owned exactly two of the three types of pet, how many
people owned all three?

A. 2
B. 4
C. 8
D.12
E. 32

SOLUTION :
none = 0
cats = C
dogs = D
Rabbits = R

Total = C + D + R - (CandD + DandR + CandR) + (CandDandR) - - - - - (1)

Exactly 2 = (CandD + DandR + CandR) - 3(CandDandR)


12 = (CandD + DandR + CandR) - 3(CandDandR) - - - - - (2)

Taking the value of (CandD + DandR + CandR) from Equation (2) and putting in Equation (1), we
get Total = C + D + R - (12 + 3(CandDandR)) + (CandDandR)

60 = 30 + 40 + 10 - 12 - 2(CandDandR)

=> (CandDandR) = 4

ANSWER B
5. In school V, all students must take at least one of Mathematics, Physics, or Biology. None of
the students took 3 classes. The number of students who took Mathematics class is 18, the
number of students who took Physics class is 23, and the number of students who took Biology
class is 20. 3 students took both Mathematics class and Physics class, 8 students took both
Physics class and Biology class, and 6 students took both Biology class and Mathematics class.
What is the total number of students in school V?

A. 11
B. 22
C. 33
D. 44
E. 55

SOLUTION :

none = 0
Mathematics = M
Physics = P
Biology = B

Total = none + M + P + B - (MandP) -(PandB) - (MandB) + (MandPandB)

Total = 0 + 18 + 23 + 20 - (3) - (8) - (6) + (0) = 61 - 17 = 44

ANSWER D
6. Of the 400 members at a health club, 260 use the weight room and 300 use the pool. If at
least 60 of the members do not use either, then the number of members using both the weight
room and the pool must be between:

A. 40 to 100
B. 80 to 140
C. 160 to 260
D. 220 to 260
E. 220 to 300

SOLUTION :

The maximum value of both will be 260.


(As intersection can’t be more than 260 as 260 members use the weight room)

Let’s find the minimum value of both.

Total = none + Weight Room + Pool - (both)


400 = none + 560 - (both)
(both) = none + 160
It is given that none ≥ 60

So minimum value of both


(both) = 60 + 160 = 220

ANSWER D
7. A certain firm has 28 lawyers in three states. These lawyers have licenses either in only one
state or all three states. If 10 have license in state X, 11 have license in Y, 13 have license in Z.
How many lawyers have licenses only in state Z ?

A. 3
B. 6
C. 10
D. 11
E. 13

SOLUTION :

X=a+d
Y=b+d
Z=c+d

X + Y + Z = a + b + c + 3d
10 + 11 + 13 = a + b + c + 3d
34 = a + b + c + 3d - - - - - - (1)

We can write one more equation with respect to the total.


Total = 28 = a + b + c + d - - - - - (2) [ We know none = 0 ]

Equation (1) - (2) gives d = 3.


Z = c + d => 13 = c + 3 => c (Only Z) = 10.

ANSWER C
8. Of all the students in a certain dormitory, 1/2 are first-year students and the rest are
second-year students. If 4/5 of the first-year students have not declared a major and if the
fraction of second-year students who have declared a major is 3 times the fraction of first-year
students who have declared a major, what fraction of all the students in the dormitory are
second-year students who have not declared a major?

1
A. 15
1
B. 5
4
C. 15
1
D. 3
2
E. 5

SOLUTION :

Let's pick 50 as the number of first year students and 50 as the number of 2nd year students.

First Year Second Year

Major 10 30

Not Major 40 20

50 50 Total = 100

We know that 4/5 of the 1st years have not declared a major so that gives us 40% of 50=40, so
10 have declared a major.

We also know that the number of 2nd year students who have declared a major = 3 times the
number of 1st year students who have declared a major. This gives us 3×10 = 30

The question asks for a fraction of 2nd years who have not declared a major over the total
students. 30 have declared a major out of the 2nd year so 20 have not.

20
Since we have 100 students (50+50), the answer is 100
or 20%.
ANSWER B
9. Among 100 students in a class, 57 like football and 68 like basketball. The maximum possible
number of students who like both football and basketball is represented by 'a', and the
minimum possible number of students who like both football and basketball is represented by
'b'. What is the value of 'a' minus 'b'?

A. 12
B. 19
C. 23
D. 28
E. 32

SOLUTION :

Total = F + B - Both + None


100 = 57 + 68 - Both + None
Both = 25 + None
Both will be minimum when None = 0.
Min value of Both = 25
Maximum value of Both = 57 (As 57 like football and intersection can’t be more than this value)

Difference = a - b = 57 - 25 = 32

ANSWER E
Word Problems

1. Last year a state senate consisting of only Republican and Democrat members had 20 more
Republican members than Democrat members. This year the senate has the same number of
members as last year, but it has 2 fewer Republican members than last year. If this year the
number of Republican members is 2/3 the number of senate members, how many members
does the senate have this year?

A. 33
B. 36
C. 42
D. 45
E. 48

SOLUTION :

Let D = the number of Democrats LAST YEAR


Let R = the number of Republicans LAST YEAR

R = D + 20 - - - - - (1)

This year the senate has the same number of members as last year, but it has 2 fewer
Republican members than last year.
So, R - 2 = the number of Republicans THIS YEAR

Since the Senate still has the same number of members as last year, there must be 2 additional
Democrats this year (to account for the fact there are 2 fewer Republicans).
So, D + 2 = the number of Democrats THIS YEAR

If this year the number of Republican members is 2/3 the number of senate members
2
Important : R - 2 = 3 (R + D) => R = 2D + 6 - - - - (2)
Solving Equation (1) and (2) gives D = 14 => R = 34
The total number of Senators THIS YEAR = R + D = 14 + 34 = 48

ANSWER E
2. A company wants to buy computers and printers for a new branch office, and the number of
computers can be at most 3 times the number of printers. Computers cost $1,500 each, and
printers cost $300 each. What is the greatest number of computers that the company can buy if
it has a total of $9,100 to spend on computers and printers?

A. 2
B. 3
C. 4
D. 5
E. 6

SOLUTION :

The number of computers can be at most 3 times the number of printers


Let C = the number of computers purchased
Let P = the number of printers purchased
So, we can write: C ≤ 3P

Computers cost $1,500 each, and printers cost $300 each.


So, the total cost of purchasing C computers and P printers = 1500C + 300P

What is the greatest number of computers that the company can buy if it has a total of $9,100
to spend on computers and printers ?

This tells us that the total cost cannot exceed $9100


So, we can write, 1500C + 300P ≤ 9100

=> 15C + 3P ≤ 91

We now have the following system of inequalities:


C - 3P ≤ 0
15C + 3P ≤ 91

Adding both the inequalities , we get 16C ≤ 91 => C ≤ 5.xx


Since C must be a positive integer, the greatest possible value of C is 5
ANSWER D
3. Abby, Brandon, Cedric, and Deirdre are planning to attend a concert. If their friend Kim also
goes with them, they will get a group discount on their tickets. If the total price of the tickets
with the group discount is the same as the total price of the tickets without the group discount,
how much is the discount?

A. 5%
B. 10%
C. 20%
D. 25%
E. 33%

SOLUTION :

ASSIGN SOME "NICE" VALUES

Let's say the price per ticket = $1


So, the TOTAL cost (i.e., without a discount) for Abby, Brandon, Cedric, and Deirdre will be $4

The total price of the tickets with the group discount is the same as the total price of the tickets
without the group discount
When Kim comes too, the TOTAL discounted price will be $4

If tickets are $1 each, then the TOTAL cost (without discount) for the five people SHOULD be $5

So, what's the discount (expressed as a percent)?


There’s a discount of 1 over a base of 5 i.e 20%

ANSWER C
4. Last month a certain music club offered a discount to preferred customers. After the first
compact disc purchased, preferred customers paid $3.99 for each additional compact disc
purchased. If a preferred customer purchased a total of 6 compact discs and paid $15.95 for the
first compact disc, then the dollar amount that the customer paid for the 6 compact discs is
equivalent to which of the following?

A. 5(4.00) + 15.90
B. 5(4.00) + 15.95
C. 5(4.00) + 16.00
D. 5(4.00 - 0.01) + 15.90
E. 5(4.00 - 0.05) + 15.95

SOLUTION :

The first disc is for $15.95 and the 5 subsequent ones for $3.99 each.

Total = 15.95 + 5(3.99)


= 15.95 + 5(4.00-0.01)
= 15.95 + 5(4.00)-0.05
= 15.90 + 5(4.00)

ANSWER A
5. The ratio between the present ages of Aakash and Bipul is 5 : 4. The ratio between Aakash’s
age 10 years hence and Bipul’s age 10 years ago is 5 : 3. What is the ratio between Aakash’s age
5 years ago and Bipul’s age 5 years hence ?

A. 13 : 20
B. 21 : 12
C. 12 : 21
D. 25 : 23
E. 25 : 21

SOLUTION :

𝐴 5𝑥
𝐵
= 4𝑥

5𝑥+10 5
4𝑥−10
= 3
⇒ x = 16

A = 80 year and B = 64 year

5 years ago, Age of A = 75


After 5 years, Age of B = 69

=> 25 : 23.

ANSWER D
6. Darcy lives at a distance of 1.5 miles from work. She can walk to work at a constant rate of 3
miles per hour, or she can ride the train to work at a constant rate of 20 miles per hour. If she
rides the train, there is an additional x minutes spent walking to the nearest train station,
waiting for the train, and walking from the final train station to her work. If it takes Darcy a total
of 15 more minutes to commute to work by walking than it takes her to commute to work by
riding the train, what is the value of x?

A. 10.5
B. 15
C. 25.5
D. 30
E. 60

SOLUTION :
"Darcy lives 1.5 miles from work. She can walk to work at a constant rate of 3 miles per hour," -
1
When she walks, she takes 2
hr or 30 mins to reach work.

"or she can ride the train to work at a constant rate of 20 miles per hour." - When she takes the
1.5
train, she takes ( 20
) × 60 mins = 4.5 mins to reach work.

"If she rides the train, there is an additional x minutes spent walking to the nearest train
station, waiting for the train, and walking from the final train station to her work." - Time taken
to reach work by train = 4.5 + x

"If it takes Darcy a total of 15 more minutes to commute to work by walking than it takes her to
commute to work by riding the train," - it takes her 30 mins to walk so it takes her 15 mins to go
by train.

"what is the value of x? "

15 = 4.5 + x
x = 10.5

ANSWER A
7. The total cost of a vacation was divided equally among 3 people. If the total cost of the
vacation had been divided equally among 4 people, the cost per person would have been $50
less. What was the total cost of the vacation?

A. $200
B. $300
C. $400
D. $500
E. $600

SOLUTION :

Let T = total cost of vacation


𝑇 𝑇
So, 3
= cost per person with THREE people and 4
= cost per person with FOUR people

If the total cost of the vacation had been divided equally among 4 people, the cost per person
would have been $50 less.

In other words (cost per person with FOUR people) = (cost per person with THREE people) - $50
𝑇 𝑇
We can write: 4
= 3
- 50

At this point, we can EITHER test the answer choices OR solve the equation.
Let's solve the equation.

𝑇 𝑇
Take: 4
= 3
- 50
12 is the LCM of 3 and 4, so multiply both sides by 12 to get: 3T = 4T - 600

=> T = 600

ANSWER E
8. During a certain two-week period, 70 percent of the movies rented from a video store were
comedies, and of the remaining movies rented, there were 5 times as many dramas as action
movies. If no other movies were rented during that two-week period and there were A action
movies rented, then how many comedies, in terms of A, were rented during that two-week
period?

𝐴
A. 14
5𝐴
B. 7
7𝐴
C. 5
D. 14A
E. 35A

SOLUTION :

Total movies = 100.


Comedies = 70.

Remaining Movies = Action + Drama = 30.

Since there were 5 times as many dramas as action movies, then Action + 5 × Action = 30
⇒ Action = A = 5.

Comedies = 70 = 14A.

ANSWER D.
9. Exchange Rates in a Particular Year

$1 = 5.3 francs
$1 = 1.6 marks

An American dealer bought a table in Germany for 480 marks and sold the same table in France
for 2,385 francs. What was the dealer's gross profit on the two transactions in dollars?

A. $0
B. $50
C. $100
D. $150
E. $200

SOLUTION :

480
Cost price = 480 marks = $( 1.6
) = $300

2385
Selling price = 2385 francs = $( 5.3
) = $450

Profit = Selling price -Cost price = $450- $300 = $150

ANSWER D
10. A certain movie star's salary for each film she makes consists of a fixed amount, along with a
percentage of the gross revenue the film generates. In her last two roles, the star made $32
million on a film that grossed $100 million, and $24 million on a film that grossed $60 million. If
the star wants to make at least $40 million on her next film, what is the minimum amount of
gross revenue the film must generate?

A. $110 million
B. $120 million
C. $130 million
D. $140 million
E. $150 million

SOLUTION :

Let x be fixed income


Let y be the % of variable income.

x + (y%) of 100 = 32 - - - - (1)

x + (y%) of 60 = 24 - - - - - -(2)

On Solving , {Equation (1) - (2)}

We get y = 20. Putting the value of y in Equation (1) will give x = 12

If the star wants to make at least $40 million on her next film :

12 + 20%(Gross Revenue) > = 40

(Gross Revenue) > = 140

Hence, the minimum amount of gross revenue the film must generate = 140.

ANSWER D
Statistics Extra Question Solutions

Averages

1. The average (arithmetic mean) of 7 numbers in a certain list is 12. The average of the 4
smallest numbers in this list is 8, while the average of the 4 greatest numbers in this list is 20.
How much greater is the sum of the 3 greatest numbers in the list than the sum of the 3
smallest numbers in the list ?

A. 4
B. 14
C. 28
D. 48
E. 52

SOLUTION :
Let the 7 numbers in ascending order be a,b,c,d,....,g.
Sum of 7 numbers = a + b + c + d + . . . . . .. . .+ g = 7×12 = 84 - - - - - - - (1)
Sum of the 4 smallest numbers = a + b + c + d = 4×8 = 32 - - - - - - - - - - (2)
Sum of the 4 greatest numbers = d + e + f + g = 4×20 = 80 - - - - - - - - - (3)

Equation (2)+(3)
a + b + c + 2d + e + f + g = 112

Equation (2)+(3) - Equation (1) gives


d = 28

Sum of the 3 greatest numbers = e + f + g = 80 - d => e + f + g = 80 - 28 = 52


So , e + f + g = 52

Sum of the 3 smallest numbers = a + b + c = 32 - d = 32 - 28 = 4


So, a + b + c = 4

Sum of the 3 greatest numbers - Sum of the 3 smallest numbers = 48

ANSWER D
2. A group consists of both men and women. The average (arithmetic mean) height of the
women is 66 inches, and the average (arithmetic mean) height of the men is 72 inches. If the
average (arithmetic mean) height of all the people in the group is 70 inches, what is the ratio of
women to men in the group ?

A. 1:1
B. 1:2
C. 2:1
D. 2:3
E. 3:2

SOLUTION :
As all the average info is provided , we can solve this question by alligation.

Women : Men = 1 : 2

ANSWER B
3. A certain list consists of 21 different numbers. If n is in the list and n is 4 times the
average(arithmetic mean) of the other 20 numbers in the list, then n is what fraction of the sum
of the 21 numbers in the list?
1
A. 20
1
B. 6
1
C. 5
4
D. 21
5
E. 21

SOLUTION :

Let the numbers be n , n1 , n2 , n3 , . . . . . . . . . .n20

If n is in the list and n is 4 times the average(arithmetic mean) of the other 20 numbers in the
list

𝑛1 + 𝑛2 + . . . . . . + 𝑛20
=> n = 4 ( 20
)
=> 𝑛1 + 𝑛2 + . . . . . . + 𝑛20 = 5n

n is what fraction of the sum of the 21 numbers in the list ?

𝑛 𝑛 1
=> 𝑛 + (𝑛1 + 𝑛2 + . . . . . . + 𝑛20)
= 𝑛 + 5𝑛
= 6

ANSWER B
4. In a certain archery contest, archers are awarded a different number of points for each arrow,
depending upon the accuracy of the shot. If an arrow hits the bull’s-eye, 10 points are awarded.
The results for one archer are shown in the table, but the values do not include her bull’s-eyes.
If the archer’s average score per arrow is 6.0, how many of her attempts were bull’s-eyes ?

Points Number of Arrows

0 1

1 1

3 3

5 6

7 8

9 2

A. 0
B. 1
C. 2
D. 3
E. 4

SOLUTION :
𝑆𝑢𝑚
Average = 𝑛
If the archer’s average score per arrow is 6.0
1×0 + 1×1 + 3×3 + 5×6 + 7×8 + 9×2 + 10×𝐵𝑢𝑙𝑙'𝑠 𝐸𝑦𝑒
=> 6 = 1 + 1 + 3 + 6 + 8 + 2 + 𝐵𝑢𝑙𝑙'𝑠 𝐸𝑦𝑒
114 + 10×𝐵𝑢𝑙𝑙'𝑠 𝐸𝑦𝑒
=> 6 = 21 + 𝐵𝑢𝑙𝑙'𝑠 𝐸𝑦𝑒

=> 126 + 6 Bull’s Eye = 114 + 10 Bull’s Eye


=> 12 = 4 Bull’s Eye
=> Bull’s Eye = 3

ANSWER D
5. The average (arithmetic mean) monthly income of four workers is $2,200. After one worker’s
income increases by 25 percent the new average income is $2,400. What was the original
income of the worker whose monthly income increased ?

A. $800
B. $1,200
C. $2,400
D. $3,200
E. It cannot be determined from the information given

SOLUTION :
Hint: Let the income of the 4 workers be x1,x2,x3,x4.
Let the income x4 increase by 25%.
Thus, Sum of all samples = number of samples × Average
Initially, x1+ x2 + x3 + x4 = 8800 - - - - - (1)

And later, x1+ x2 + x3 + 1.25 x4 = 9600 - - - - - (2)

Equation (2) - (1),

0.25 x4 = 800

Thus, x4 = 3200.

ANSWER D
6. An exam is given in a certain class. The average (arithmetic mean) of the highest score and
the lowest score is equal to x. If the average score for the entire class is equal to y and there are
z students in the class, where z > 5, then in terms of x, y, and z, what is the average score for the
class excluding the highest and lowest scorers?

(𝑧𝑦 – 2𝑥)
A. 𝑧
(𝑧𝑦 – 2)
B. 𝑧
(𝑧𝑥 – 𝑦)
C. (𝑧 – 2)
(𝑧𝑦 – 2𝑥)
D. (𝑧 −2)
(𝑧𝑦 – 𝑥)
E. (𝑧 + 2)

SOLUTION :

The average (arithmetic mean) of the highest score and the lowest score is equal to x
=> Sum of the highest and lowest scores is 2x.

If the average score for the entire class is equal to y and there are z students in the class :
Sum of all samples = number of samples × Average

=> Highest + Lowest + Sum of the scores of (z-2) students = z × y


=> 2x + (z - 2)×Average of the rest = zy
=> (z - 2)×Average of the rest = zy - 2x
𝑧𝑦 − 2𝑥
=> Average of the rest = (𝑧 − 2)

ANSWER D
7. The median height of 10 people is 70, the average is 72.5, and the range is 12. If a person
with a height of 74 is added, which number will definitely change?

A. Mean
B. Median
C. Range
D. Mean & Range
E. None of the above

SOLUTION :
𝑁𝑒𝑤 𝑆𝑢𝑚 774
If a person with a height of 74 is added , then Mean = 𝑛+1
= 11
= 70.XXX
Mean will definitely change.
The median might not change when the 5th and 6th term is the same
The range might not change if newly added term '74' is not the smallest or the largest term of
the set

ANSWER A
8. At a restaurant, glasses are stored in two different-sized boxes. One box contains 12 glasses,
and the other contains 16 glasses. If the average number of glasses per box is 15, and there are
16 more of the larger boxes, what is the total number of glasses at the restaurant ?
(Assume that all boxes are filled to capacity.)

A. 96
B. 240
C. 256
D. 384
E. 480

SOLUTION :
We have all the average info , so we can use alligation here.

Small : Large : Difference = 1 : 3 : 2


Difference in the actual question is 16 , so we need to multiply the ratio by 8.
Hence, Small Boxes = 8 , Large Boxes = 16
=> Total number of glasses = 8 ×12 + 24×16 = 480

ANSWER E
9. Fiber X cereal is 55% fiber. Fiber Max cereal is 70% fiber. Sheldon combines an amount of the
two cereals in a single bowl of mixed cereal that is 65% fiber. If the bowl contains a total of 12
ounces of cereal, how much of the cereal, in ounces, is Fiber X ?

A. 3
B. 4
C. 6
D. 8
E. 9

SOLUTION :
We have all the average info , so we can use alligation here.

Normal : Max : Total = 1 : 2 : 3


Total weight of the mixture given is 12 , so multiply the ratio by 4 to get the actual weight of the
Normal Cereal (Fibre X) = 4

ANSWER B
10. A certain car averages 25 miles per gallon of gasoline when driven in the city and 40 miles
per gallon when driven on the highway. According to these rates, which of the following is
closest to the number of miles per gallon that the car averages when it is driven 10 miles in the
city and then 50 miles on the highway?

A. 28
B. 30
C. 33
D. 36
E. 38

SOLUTION :
A certain car averages 25 miles per gallon of gasoline when driven in the city :
1 gallon -> 25 miles , So for 1 miles -> 1/25 gallon will be used.

A certain car averages 40 miles per gallon of gasoline when driven on the highway :
1 gallon -> 40 miles , So for 1 miles -> 1/40 gallon will be used.

The average is of the form gallon / mile , so samples will be in miles.

Sum of all samples = number of samples × Average


=> (1/25)×10 + (1/40)×50 = 60 × Average
=> 0.4 + 1.25 = 60 × Average
=> 1.65 = 60 × Average
165
=> Average = 6000
gallon / mile

165 6000
1 mile -> 6000
gallon ; 1 gallon -> 165
miles -> 36.36 miles

36 miles/gallon

ANSWER D
11. At a certain fruit stand, the price of each apple is 40 cents and the price of each orange is 60
cents. Mary selects a total of 10 apples and oranges from the fruit stand, and the average
(arithmetic mean) price of the 10 pieces of fruit is 56 cents. How many oranges must Mary put
back so that the average price of the pieces of fruit that she keeps is 52 cents ?

A. 1
B. 2
C. 3
D. 4
E. 5

SOLUTION :
We have all the average info , so we can use alligation here.

Total Fruits is 10 , so we need to multiply the ratio by 2.


Apples = 2 , Oranges = 8
Finally , Apple : Orange : Total = 2 : 3 : 5
As we know that the number of apples are not changing (it is 2 initially)
We have apples = 2 , Oranges = 3

This means that there’s a change of 5 Oranges.

How many oranges must Mary put back so that the average price of the pieces of fruit that she
keeps is 52 cents ?
=> 5 Oranges.

ANSWER E
12. Ashok purchased two qualities of grains at the rate of Rs 100 per quintal and Rs 160 per
quintal. In 50 quintals of the second quality, how much grain of the first quality should be mixed
so that by selling the resulting mixture at Rs 195 per quintal, he gains a profit of 30% ?

A. 10 quintals
B. 14 quintals
C. 20 quintals
D. 25 quintals
E. None of these

SOLUTION :

SP of the mixture = CP of the mixture (1 + 30%)


195 = 1.3 CP
CP = 150

As we have all the average information , we can use alligation here.

Weight 1 : Weight 2 = 10 : 50
Hence , 10 quintals of the first quality will be taken.

ANSWER A
13. Kiran lends Rs 1000 on simple interest to Harsh for a period of 5 years. She lends a part of
the amount at 2% annual interest and the rest at 8% annual interest and receives Rs 300 as the
amount of interest. How much money (in Rs) did she lend at a 2% interest rate ?

A. 333.33
B. 666.67
C. 400
D. 500
E. None of the above

SOLUTION :
This question is another application of alligation . Two amounts separately give certain interest
and they are equivalent to a combined amount of Rs 1000.

300 is the interest earned in 5 years , So Rs 60 will be the interest in 1 year.


60
We get 60 over 1000 for 1 year. Hence , the rate is 1000
i.e 6%

Now , we know all the averages involved.

1000
As we know the Total Sum is 1000 , we need to multiply the ratio by 3
1000
So, Sum 1 = 3
= 333.33

ANSWER A
14. A man purchased a table and a chair for Rs 2000. He sold the table at a profit of 20% and the
chair at a profit of 40%. In this way, his total profit was 25%.Find the cost price (in Rs) of the
table.

A. 1500
B. 900
C. 1000
D. 800
E. None of the above

SOLUTION :
This question is another application of alligation . Profit % on two items is given and the total
profit % information is also provided.

Total Cost is 2000 , so we need to multiply the ratio by 500.


Hence, Cost Price of the Table = 3×500 = 1500

ANSWER A
Median

1. Three positive numbers x, y, and z have the following relationships y = x + 2 and z = y + 2.


When the median of x, y, and z is subtracted from the product of the smallest number and the
median, the result is 0. What is the value of the largest number ?

A. –2
B. 3.14
C. 5
D. 8
E. 10.5

SOLUTION :
y is 2 more than x and z is 2 more than y. So the three numbers are:
x, x+2, x+4

Their median = x+2


Product of smallest number and median = x(x+2)

Given: x(x+2) - (x+2) = 0


Taking (x+2) common, (x+2) (x-1) = 0
So either x = -2 or x = 1

But we are given that these are positive numbers. So x=1


Largest number = x + 4 = 1 + 4 = 5

ANSWER C
2. In a certain lottery drawing, five balls are selected from a tumbler in which each ball is
printed with a different two-digit positive integer. If the average (arithmetic mean) of the five
numbers drawn is 56 and the median is 60, what is the greatest value that the lowest number
selected could be ?

A. 43
B. 48
C. 51
D. 53
E. 56

SOLUTION :
Say the five numbers drawn are a, b, c, d, e, where: a < b < c < d < e
(each ball is printed with a different two-digit positive integer).

The average (arithmetic mean) of the five numbers drawn is 56 and the median is 60:
median = c = 60 and
Sum of all samples = number of samples × Average
a + b + 60 + d + e = 56× 5 = 280.

a + b + d + e = 220.

What is the greatest value that the lowest number selected could be? The lowest number is a.
To maximize a, we need to minimize other numbers. The lowest values of d and e are 61 and 62.

a + b + 61 + 62 = 220.
a + b = 97.

If we divide 97 in two equal parts , a and b would have been 48.5 each
(since a < b) => a = 48 and b = 49

ANSWER B
3. A set of 5 numbers has an average of 50. The largest element in the set is 5 greater than 3
times the smallest element in the set. If the median of the set equals the mean, what is the
largest possible value in the set ?

A. 85
B. 86
C. 88
D. 91
E. 92

SOLUTION :
Let the 5 numbers be A,B,C,D,E.
Sum of all samples = number of samples × Average
A + B + C + D + E = 250
C = 50 (median of the set equals the mean)
What is the largest possible value in the set ?
In order to get the maximum value , minimize the other values.

=> A + A + 50 + 50 + 5 + 3A = 250
=> 105 + 5A = 250
=> 5A = 145
=> A = 29
E = 5 + 3A = 5 + 3(29) = 5 + 87 = 92

Alternate way is to go via options.

ANSWER E
4. For the set of positive, distinct integers {v, w, x, y, z}, the median is 10. What is the minimum
value of v + w + x + y + z ?

A. 25
B. 32
C. 36
D. 40
E. 50

SOLUTION :

Let v < w < x < y < z


x = 10 (median)
What is the minimum value of v + w + x + y + z ?
y and z should be as close to the median as possible i.e y = 11 , z = 12
v , w should be as minimum as possible i.e v = 1 , w = 2
So , v + w + x + y + z = 1 + 2 + 10 + 11 + 12 = 36

ANSWER C
5. Which of the following cannot be the median of five positive integers a, b, c, d, and e ?

A. a
B. d + e
C. b + c + d
(𝑏 + 𝑐 + 𝑑)
D. 3
(𝑎 + 𝑏 + 𝑐 + 𝑑 + 𝑒)
E. 5

SOLUTION :

Note that you are not given that the numbers are arranged in ascending order. They could be
any way when arranged in ascending order. Keeping this in mind, try to find easy ways of getting
the median.

A. a
a could be the middle number
a=3, {b,c,d,e}={1,2,4,5}

B. d + e
Middle number could be other than d and e and d and e could be the numbers smaller than the
middle numbers e.g.
{ d,e,a,b,c } = { 1,1,2,2,2 }
=> Median is 2 which is 1 + 1 ( d + e )

D. (b + c + d) / 3
Avg of 3 numbers can certainly be the median -> say all numbers are equal: 2, 2, 2, 2, 2
-> median is 2 -> Avg of any 3 numbers
E. (a + b + c + d + e) / 5
Avg of all 5 numbers can certainly be the median -> say all numbers are equal : 2, 2, 2, 2, 2
-> median is 2 -> Avg of all 5 numbers

C. b + c + d
Now, you don't need to worry about (C) and you can just mark it but note why it cannot be the
median.
Out of 5 positive numbers, the median will be the middle number. b , c or d cannot be the
middle number. So either a or e will be the median. Two numbers will be smaller than (or equal
to) the median and two numbers will be greater (or equal to). So even if you pick both numbers
which are smaller than the median, you will still need to pick one which is equal or greater. So
the sum of 3 numbers will certainly be greater than the median.

ANSWER C
6 : $10, $11, $11, $12, $12, $14, $16, $17, $21, $22

The various prices that a certain product was sold for at retailers in City X is shown above. How
many prices were greater than the median price but less than the mean price?

A. None
B. One
C. Two
D. Three
E. Four

SOLUTION :

The mean price is:

10 + 22 + 24 + 14 + 16 + 17 + 21 + 22 146
10
= 10
= 14.6

12 + 14
The median is 2
= 13

So, there is only 1 price greater than the median but less than the mean, which is $14.

ANSWER B
7. Positive integers from 1 to 45, inclusive are placed in 5 groups of 9 each. What is the highest
possible average of the medians of these 5 groups ?

A. 25
B. 31
C. 15
D. 26
E. 23

SOLUTION :
We are dividing the numbers into sets of 9. So, we will have 5 sets.
The median of each set is the middle number when the 9 numbers in the set are arranged in
ascending order.So, the median is the 5th number when the numbers in a set are written in
ascending order.Therefore, we will have 4 numbers that are greater than the median in each set.

We need to maximize the median in each group in order to maximize the average of all the
medians.
The highest possible median is 41 as there should be 4 numbers greater than the median in a
group of 9.
So, if we have a group that has a, b, c, d, 41, 42, 43, 44, 45, the median will be 41.
In this set, it is essential not to expend any more high values on a, b, c, or d as these do not
affect the median.
For example, the median of a group that comprises 1, 2, 3, 4, 41, 42, 43, 44, 45 will be 41.

The next group can be 5, 6, 7, 8, 36, 37, 38, 39, 40. The median will be 36.
The first 4 numbers in any set are not of much consequence. Our focus is on the numbers
upward of the median in any set.
Seeing the pattern in the two sets listed above, to maximize medians in all the 5 groups, the
medians of the 5 groups will have to be 21, 26, 31, 36, and 41. (Find the medians of the rest of
the groups yourself)
The average of the highest possible medians will be the average of these 5 numbers = 31.

ANSWER B
8. An analysis of the monthly incentives received by 5 salesmen : The mean and median of the
incentives is $7000. The only mode among the observations is $12,000. Incentives paid to each
salesman were in full thousands. What is the difference between the highest and the lowest
incentive received by the 5 salesmen in the month ?

A. $4000
B. $13,000
C. $9000
D. $5000
E. $11,000

SOLUTION :
The arithmetic mean of the incentives is $7000.
The median of the incentives is also $7000.
The mode is $12,000.

Let their incentives be a, b, c, d, and e such that a ≤ b ≤ c ≤ d ≤ e


Therefore, the median of these values is 'c'.
The median incentive is $7000. So, c = $7000.
Essentially, the incentives are __ __ 7000 __ __
The arithmetic mean of the incentives is $7000.
So, the sum of their incentives a + b + c + d + e = 5 × 7000 = $35,000

There is only one mode amongst these 5 observations.


The mode is that value that appears with the maximum frequency.
Hence, $12,000 is the incentive received by the most number of salesmen.
So, the incentives are __ , __ , 7000 , 12000, 12000
(Think why can’t there be three 12,000 values)
The incentive that c has got is $7000 ; The incentive received by d and e are 12,000 each
Therefore, c + d + e = 7000 + 12,000 + 12,000 = $31,000
Hence, a + b = 35,000 - 31,000 = $4000

As there is only one mode, the incentives received by a and b have to be different.
So, a received $1000 and b received $3000.
Maximum incentive : $12,000 ; Minimum incentive : $1000
Difference between maximum and minimum incentive : $11,000

ANSWER E
9. The median annual household income in a certain community of 21 households is $50,000. If
the mean annual income of a household increases by 10% per year over the next 2 years, what
will the median income in the community be in 2 years?

A. $50,000
B. $60,000
C. $60,500
D. $65,000
E. Cannot be determined

SOLUTION :

Mean increasing by certain % doesn't ensure that the median should increase by the same %.

Median will only increase by the same % if all data points move ahead by the same %.
Increase of mean does not guarantee that all data points are increasing by the same % as
well.

ANSWER E
10. { 150, 200, 250, n }
Which of the following could be the median of the 4 integers listed above?

I. 175
II. 215
III. 235

A. I only
B. II only
C. I and II only
D. II and III only
E. I, II, and III

SOLUTION :
Note that we have 4 elements in the list, which is an even number of elements.
Then the median of this list will be an average of the two middle numbers.
With an n < 150, our median would be the average of 150 and 200, which is 175.
With n > 250, the median would be the average of 200 and 250, which is 225.

Thus, we can conclude that the median must be between 175 and 225, inclusive.
Thus option I is viable with n < 150, and option II would be possible if we let n = 230.
Option III is out of range, so we can only select I and II.

ANSWER C
Range

1. The range of the salary in January is $240 and the range of the salary in February is $320.
What is the minimal range of the salary for the two months combined?

A. 80
B. 240
C. 320
D. 400
E. 480

SOLUTION :
January
Lowest salary = x
Highest salary = x+240

February
Lowest salary = y
Highest salary = y + 320

2 Months Combined

Case 1: x < y
If x < y → x + 240 < y + 320
Lowest Value = x
Highest Value = y + 320
If this was the case, the range will be greater than 320.
Range = (y -x) + 320

Case 2: x = y
Lowest Value = x = y
Highest Value = y + 320 = x + 320
Range = 320.
Case 3: x > y
Lowest Value = y
The highest value can be either y + 320 or x + 240

If the highest value = y + 320 ; Range = 320


If the highest value = x + 240 ; Range = (x - y) + 320 ; Range will be greater than 320.

Therefore the lowest value of range from all three cases = 320

ANSWER C
2. 10 distinct integers are arranged in ascending order. The range of the first 7 is 25 and that of
the last 7 is 30. What is the maximum range of the 10 numbers?

A. 25
B. 27
C. 30
D. 52
E. 55

SOLUTION :
Given:
1st position = p
4th position = q
7th position = r
10th position = s

We know that:
r - p = 25
s - q = 30

Note: The numbers are distinct and arranged in ascending order, but they are not necessarily
consecutive.
Let’s assume the first number p to be 0 , so the 7th number r will be 25.
We are asked to maximize the range and the integers are distinct.

In order to increase the range, maximize the value of ‘s’. ‘s’ can be maximized by taking the max
value of ‘q’. From the assumed values, the maximum value of ‘q’ can be 22.
(As 7th number is 25 , 6th will be 24 , 5th will be 23 , and 4th i.e q will be 22)

Therefore, s - q = 30 => s = q + 30
=> s = 22 + 30 = 52
So, the Range = 52 - 0 = 52

ANSWER D
3. List K consists of 12 consecutive integers. If -4 is the least integer in list K, what is the range of
the positive integers in list K?

A. 5
B. 6
C. 7
D. 11
E. 12

SOLUTION :

The set is {-4, -3, -2, -1, 0, 1, 2, 3, 4, 5, 6, 7}. The question asks to find the range of the POSITIVE
integers in the list.

The range of the POSITIVE integers in the list is {largest} - {smallest} = 7 - 1 = 6.


2
4 . S is the set that contains numbers of the form (𝑘 − 𝑛) , where k and n are integers such
that 4 ≤ k < 7 < n ≤ 12. What is the range of the numbers in S ?

A. 20
B. 30
C. 40
D. 50
E. 60

SOLUTION :
2
The numbers are of the form : (𝑘 − 𝑛)

2
Minimum number in the set : when k is 6 and n is 8 : (6 − 8) : 4
2
Maximum number in the set : when k is 4 and n is 12 : (4 − 12) : 64

Range = 64 - 4 = 60

ANSWER E
2 2
5. If Set M consists of the root / roots of the equation 2 - 𝑥 = (𝑥 − 2) What is the range of
Set M ?

A. 0
1
B.
2
C. 1
D. 2
E. 2

SOLUTION :

2−x2= (x−2)2

2−x2 = x2− 4x + 4
X2−2x + 1 = 0
(x−1)2= 0
x=1
Set M contains only one element.

The range of a single element set is 0.


6. Set R contains 7 distinct positive integers { 8, 6, 14, 1, 12, n, 7 }. The greatest possible value of
n should be how much greater than the least possible value of n, if the range of all the elements
present in S is not more than 21?

A. 12
B. 14
C. 15
D. 20
E. 21

SOLUTION:

The range of a set is defined as the difference between the maximum value and the minimum
value in the set.

The maximum value of n can be 22, because if n were any larger, the range would exceed 21.

Therefore, the range with n= 22 would be 22- 1 =21, which is the maximum allowable range.

The minimum value of n can be 2, because the numbers in the set are distinct and 1 is already
present. If n were 1, it would not be distinct.

Therefore, the range with n = 2 would be 14 - 1= 13 .

The difference between the greatest possible value of n and the least possible value of n is
therefore 22-2=20.

ANSWER D.
7. Positive integers a, b, c, d and e are such that a < b < c < d < e. If the average (arithmetic
mean) of the five numbers is 6 and d - b = 3, then what is the greatest possible range of the five
numbers?

A. 12
B. 17
C. 18
D. 19
E. 20

SOLUTION :

The range of a set of numbers is defined as the difference between the maximum and minimum
values in the set, i.e., Range = e - a.
We are given that d - b = 3, which can be rewritten as d = b + 3.
The average of the five numbers is 6, which implies that the sum of the numbers is 30 (since
average = sum/number of elements), i.e , a + b + c + d + e = 30.
To maximize the range, we need to minimize a and maximize e. The minimum value a can take is
1 (since these are positive integers), so we choose a = 1.
Since the sum is fixed, we should choose the minimum possible values for b, c, and d to
maximize e.
Given that b > a, the minimum value b can take is 2.
Given that c > b, the minimum value c can take is 3.
Given that d = b + 3, we have d = 2 + 3 = 5.
Substituting these minimum values into the equation for the sum, we find the maximum value
of e:
a + b + c + d + e = 30
=> 1 + 2 + 3 + 5 + e = 30
=> e = 19

Therefore, the maximum range of the five numbers is Range(max) = e - a = 19 - 1 = 18.

ANSWER C
8. The average (arithmetic mean) of a list of 8 different positive integers is 23. Which of the
following is the greatest possible range of this list of numbers?

A. 149
B. 155
C. 156
D. 168
E. 183

SOLUTION :

Sum of 8 numbers = 8×23 = 184


In order to get the greatest possible range , The greatest number needs to be maximum and the
smallest number needs to be minimum. All the integers need to be different as well.
So, the least positive number that we can have is 1. In order to maximize the greatest number ,
we need to minimize the rest of the numbers too as Sum is constant.

So , the first 7 numbers will be 1,2,3,4,5,6,7 and their Sum will be 28.

Sum of 8 numbers = 184


=> Sum of 7 numbers + Greatest Number = 184
=> 28 + Greatest Number = 184
=> Greatest Number = 156
Range = 156 - 1 = 155
ANSWER B
Standard Deviation

1. Set S is a set of distinct positive integers. The standard deviation of Set S must increase if
which of the following were to occur?

I. The smallest number is increased to become equal to the median.


II. The smallest number is increased to become larger than the current largest number.
III. The largest number is doubled.

A. I only
B. II only
C. III only
D. I and III only
E. I, II and III

SOLUTION:

We need to check each statement to see if it would necessarily result in an increase in the
standard deviation. If we can find a case where the standard deviation does not increase, we
can eliminate that statement.

I. The smallest number is increased to become equal to the median.

Let’s consider the set {1, 2, 3}. If we increase the smallest number to equal the median, we get
the set {2, 2, 3}. This actually causes the standard deviation to decrease, so statement I is not
necessarily true. We can eliminate options A, D, and E.

II. The smallest number is increased to become larger than the current largest number. Let’s
consider the set {1, 2}. If we increase the smallest number to become larger than the current
largest number, we get the set {2, 3}. This causes the standard deviation to remain the same
(THINK WHY ??), so statement II is not necessarily true. We can eliminate option B.

By the process of elimination, the correct answer is option C, which states that the standard
deviation of set S must increase if the largest number is doubled. This is because doubling the
largest number will increase the range of the set, and a larger range generally leads to a larger
standard deviation.

ANSWER C
2. A list of 50 data points has an average (arithmetic mean) of 20 and a standard deviation of 10.
Which of the following pairs of data, when added to the list, must result in a list of 52 data
points with standard deviation less than 10 ?

A. 0 and 10
B. 0 and 20
C. 10 and 30
D. 20 and 20
E. 30 and 40

SOLUTION:

The standard deviation measures how much variation there is from the mean. A low standard
deviation indicates that the data points tend to be very close to the mean, whereas a high
standard deviation indicates that the data points are spread out over a large range of values.

When we add numbers that are far from the mean, we are stretching the set, which increases
the standard deviation. Conversely, when we add numbers that are close to the mean, we are
shrinking the set, which decreases the standard deviation.

Therefore, to decrease the standard deviation by the greatest amount, we should add two
numbers that are closest to the mean.

From the options given, the pair of numbers that are closest to the mean (20) are 20 and 20.
Adding these numbers to the list will definitely shrink the set, thus decreasing the standard
deviation.

Hence, the correct answer is option D.

ANSWER D
3. If a certain sample of data has a mean of 24.0 and the value 31.0 is more than 2.5 standard
deviations from the mean, which of the following could be the standard deviation of the
sample?

A. 3.75
B. 3.50
C. 3.25
D. 3.00
E. 2.75

SOLUTION:
Since 31.0 is more than 2.5 standard deviations from the mean (24.0), we have:
31 > 24 +2.5SD
This implies: 2.5×SD<7
Consequently: SD < 2.8
Among the given options, only option E offers a standard deviation of less than 2.8.

ANSWER E
4. Set = { 2 , 3 , 3 , 4 , 6 , 8 , 12 , 15 , 18 , 21 , 24 , 28 }

The set above shows the weight, in grams, of 12 plants grown in an experimental greenhouse. If
the standard deviation of the weights is 9, how many of the 12 plants are more than 1.5
standard deviations above the mean ?

A. 0
B. 1
C. 2
D. 3
E. 4

SOLUTION :

2 + 3 + 3 + 4 + 6 + 8 + 12 + 15 + 18 + 21 + 24 + 28 144
Mean = 12
= 12
= 12

How many of the 12 plants are more than 1.5 standard deviations above the mean ?
=> Mean + 1.5(SD) = 12 + 1.5(9) = 25.5.

How many plants have weights over 25.5 ?


=> Only 1 i.e 28

ANSWER B
5. 500 records have a distribution that is symmetric about the mean 10. If 68 percent of the
distribution lies within one standard deviation of the mean, and the deviation is 2, then at least
how many of the records are less than 12 ?

A. 160
B. 320
C. 340
D. 420
E. 460

SOLUTION :

If the distribution is symmetric, the number of values below the mean is equal to the number
above the mean. {50% to the left of mean and 50% to the right of mean}
If none of the values equal the mean exactly (a technicality we don't need to worry about), we
have 250 'below average' records, i.e. with values less than 10.
We also know that 68% of the data lie within one standard deviation i.e 34% of these will be
below average, 34% above, because the distribution is symmetric.
So from 10 to 12, we have 34% of our values, and below 10 we have another 50%, meaning 84%
will have values less than 12=> 84% of 500 is 420.

ANSWER D
6. A set of 12 test scores has an average of 500 and a standard deviation of 50. Which of the
following sets of additional test scores, when combined with the original set of 12 test scores,
must result in a combined data set with a standard deviation less than 50?

A. 6 test scores with average of 450 and standard deviation of 50.


B. 6 test scores with an average of 500 and standard deviation of 25.
C. 6 test scores with average of 550 and standard deviation of 25.
D. 12 test scores with average of 450 and standard deviation of 25.
E. 2 test scores with average of 550 and standard deviation of 50.

SOLUTION :
(CONSIDER THIS QUESTION GOOD FOR CONCEPT PRACTICE ONLY)

It is not generally true that all of the terms in a set are within one standard deviation of the
mean. However, standard deviation is a measure of the spread of the terms of a set, so we
could represent the original set of scores this way :

The oval spans ±1 standard deviation from the mean, where many of the scores will likely be.
This simplification is acceptable as long as we represent all of the other data sets the same way,
because all we want to do is compare the relative effects of the new test scores systematically.
For each of the answer choices, we can now overlay the representative ovals for the new data
on top of the oval for the original data.

It is clear that all of the data in the answer choices, when added to the original set, will result in
a likely or definite increase in the spread of the data except for answer choice (B), which
definitely concentrates the set of scores closer to the original average of 500. Thus, adding the
data in answer choice B will result in a smaller standard deviation than that found in the original
data set. The correct answer is B.

We can generalize rules for adding a single term to a set as follows:


* Adding a new term more than 1 standard deviation from the mean generally increases the
standard deviation of a set.
* Adding a new term less than 1 standard deviation from the mean generally decreases the
standard deviation of a set.

ANSWER B
7. Which of the following sets has a standard deviation greater than the standard deviation of
Set A = {10, 12, 14, 16, 18, 20, 22, 24, 26, 28} ?

I. Set B, which consists of the first 10 positive integers


II. Set C, which consists of the first 10 positive odd numbers
III. Set D, which consists of the first 10 prime numbers

A. Set B only
B. Set C only
C. Set D only
D. Sets C and D only
E. Sets B, C and D

SOLUTION :
The standard deviation is a measure of the variation of the data points from the mean, a
measure of how widespread a given set is. When the standard deviation is low, the data points
tend to be close to the mean, while a high standard deviation implies that the data is spread out
over a broader range of values.

Now, it's clear that set B = {1, 2, 3, 4, 5, 6, 7, 8, 9, 10} is less widespread than set A, so its
standard deviation is less than the standard deviation of set A. Statement I is discarded.

Set C = {1, 3, 5, 7, 9, 11, 13, 15, 17, 19} is as widespread as set A, so its standard deviation is
equal to the standard deviation of set A. An important property to note is that if we add or
subtract a constant to each term in a set, the standard deviation will not change. Since set A can
be obtained by adding 9 to each term of set C, the standard deviations of those sets are equal.
Statement II is discarded.

Set D = {2, 3, 5, 7, 11, 13, 17, 19, 23, 29} is more widespread than set A, so its standard
deviation is greater than the standard deviation of set A.

ANSWER C
8. Which of the following sets must have the same standard deviation as Set { a,b,c } ?
2
A. { ab , 𝑏 , cb }
B. { 2a, b + a , c + b }
C. { 0 , b + a , c - a }
D. { ab , bc , ac }
E. { ab + c , a(1 + b) , b(1 + a) }

SOLUTION :

Option A can be rejected as we are multiplying b to every sample which may change the
Standard Deviation.
Option B,C can be rejected as we are adding different values to the samples of the set which
may change the Standard Deviation.
Option D can be rejected as we are multiplying different values to the samples of the set which
may change the Standard Deviation.
Option E : { ab + c , ab + a , ab + b } : Subtracting ab from each sample won’t change the
Standard Deviation : { c , a , b } . Hence , Choice E must have the same standard deviation as
Set { a,b,c }

ANSWER E
Numbers Extra Question Solutions

Odd-Even/Primes

1. w, x, y, and z are consecutive odd integers such that w < x < y < z. Which of the following
statements must be true? Indicate all such statements.

1. wxyz is odd
2. w + x + y + z is odd
3. w + z = x + y

A. 1 Only
B. 2 Only
C. 3 Only
D. 1 and 3
E. 1 and 2

SOLUTION :
Given that w,x,y and z are odd integers.

1. wxyz
odd*odd*odd*odd
Product of odd integers is always odd.
True.

2. w+x+y+z
Odd+odd+odd+odd
Sum of 4 odd integers is always even.
False

3. w+z =x+y
As per the question, the consecutive odd integers are having the relation: w<x<y<z
Let us represent the odd integers as follows:
w=2a+1
x=2a+3
y= 2a+5
z= 2a+7
LHS
w+z = (2a+1)+(2a+7)=4a+8
RHS
x+y = 2a+3+2a+5 = 4a+8
So, LHS=RHS
True.

You can also put w=1, x=3 , y=5 , z=7 to solve the question

ANSWER D
2. If x is odd, which one of the following must be odd EXCEPT ?
A. x2 + 4x + 6
B. x3 + 5x + 3
C. x4 + 6x + 7
D. x5 + 7x + 1
E. x6 + 8x + 4

SOLUTION:
Given x is odd. We know that (odd)Anything is always odd.
Let us check the options now :

(A) x2+4x+6= Odd+ Even + Even = Odd

(B) x3+5x+3= Odd + Odd + Odd = Odd

(C) x4+6x+7= Odd + Even + Odd = Even

(D) x5+7x+1= Odd + Odd + Odd = Odd

(E) x6+8x+4=Odd + Even + Even = Odd

ANSWER C
3. a and b are two positive even integers such that a > b. Which of the following represents the
number of odd integers less than a + 2 and greater than b - 2?

(𝑎 + 1 − 𝑏 − 1)
A. 2
(𝑎 − 𝑏 − 2)
B. 2
(𝑎 + 2 − 𝑏 − 2)
C. 2
(𝑎 + 2 − 𝑏 + 2)
D. 2
(𝑎 − 𝑏)
E. 2
+1

SOLUTION :
Let us assume a=10 and b=2
a+2=12 and b-2=0
Odd integers less than a+2 and greater than b-2: {1,3,5,7,9,11}. Total 6 values
Let us now substitute a=10 and b=2 in the options and check which option gives us 6.

(A) (10+1-2-1)/2= 4
(B) (10-2-2)/2= 3
(C) (10+2-2-2)/2= 4
(D) (10+2-2+2)/2= 6
(E) (10-2)/2+1= 5

ANSWER D
3 3 2
4. 𝑠 𝑡 = 𝑣
If s and t are both primes, how many positive divisors of v are greater than 1, if v is an integer?

A. Two
B. Three
C. Five
D. Six
E. Eight

SOLUTION :
3 3 2
In order for the equation 𝑠 𝑡 = 𝑣 to hold, we see that s and t must be equal;, thus, we can say:
3 3 2
𝑠 𝑡 =𝑣
6 2
𝑠 =𝑣
3
𝑠 =v

To determine the total number of factors of v, we can add 1 to the exponent of 3, and thus v has
3 + 1 = 4 factors. Since one of those factors is “1”, v has 3 factors other than 1.

ANSWER B
𝑛
5. If n is an integer such that 15 < n < 250, for how many possible values of n is 5
the square of
a prime number?

A. Two
B. Three
C. Four
D. Five
E. Six

SOLUTION :
Letting p be a prime , we can create the equation:
2
n/5 = 𝑝
2
n = 5𝑝
Now, we can let p = 2, 3, 5, and so on.

If p = 2, n = 5(4) = 20.
If p = 3, n = 5(9) = 45
If p = 5, n = 5(25) = 125
If p = 7, n = 5(49) = 245

We can see that if p = 11, n will be greater than 250. Therefore, there are 4 values of n such that
15 < n < 250 and n/5 is the square of a prime number.

ANSWER C
HCF, LCM, Factors, Multiples

1. Company H distributed $4,000 and 180 pencils evenly among its employees, with each
employee getting an equal integer number of dollars and an equal integer number of pencils.
What is the greatest number of employees that could work for company H?

A. 9
B. 10
C. 20
D. 40
E. 180

SOLUTION:
Let the number of employees be x, a be the amount received by each employee and b be the
number of pencils received by each employee
So, 4000 = x*a
180 = x*b
We need to maximize x, hence we take HCF of two numbers 4000 and 180
HCF(4000,180)= 20

ANSWER C
2. Ramon wants to cut a rectangular board into identical square pieces. If the board is 18 inches
by 30 inches, what is the least number of square pieces he can cut without wasting any of the
board ?

A. 4
B. 6
C. 9
D. 12
E. 15

SOLUTION :

In order for the pieces to be minimum , the sides of the square must be maximum
HCF(30,18)=6
We get : 30 = 6 × 5 and 18 = 6 × 3

So, He needs to cut 15 square pieces.

ANSWER E
3. How many integers are there between 1 and 1000, inclusive, that are not divisible by either
11 or 35?

A. 884
B. 890
C. 892
D. 910
E. 945

SOLUTION :
Integers between 1 and 1000 that are divisible by 11: 11,22,33,......,990
We have a total of 90 integers.
Integers between 1 and 1000 that are divisible by 35: 35, 70,105,....,980
We have a total of 28 integers.
Integers divisible by both 11 and 35= 385 and 770 i.e 2 Integers

Have a look at the venn diagram:

Total = A + B - both + None


1000 = 90 + 28- 2 + None
None = 884

ANSWER A
4. If r is divisible by 10 and s is divisible by 9, rs must have at least how many factors ?

A. Two
B. Four
C. Twelve
D. Fourteen
E. Sixteen

SOLUTION :

As per the question,


r=10a
s=9b
rs= 90ab

The least value of rs will be for ab=1


rs=90

90 can be written as 2× 32× 5


Least number of factors: (1+1)*(2+1)*(1+1)=12

ANSWER C
3
5. If n is an integer and 𝑛 is divisible by 24, what is the largest number that must be a factor of
n?

A. 1
B. 2
C. 6
D. 8
E. 12

SOLUTION :
Given n is an integer
n3= 24 a
n3= 23*31 a

Now, for n3 to be an integer a must contain at least 2 powers of 3


So, n3=23*33
n3=(63)
n=6
So, 6 is the largest number that must be a factor of n

ANSWER C
6. If a = 16b and b is a prime number greater than 2, how many positive distinct factors does a
have ?

A. 4
B. 5
C. 6
D. 8
E. 10

SOLUTION :
a=16b
Given that b>2, which implies b is an odd prime.

a=24 (prime)1

A prime number can only take 1 as its power. Apart from that if it takes any other power, then b
will not be a prime number.

So,Total number of distinct factors= (4+1)*(1+1)= 5*2 = 10

ANSWER E
7. If a and b are integers such that a > b > 1, which of the following cannot be a multiple of
either a or b ?

A. a – 1
B. b + 1
C. b – 1
D. a + b
E. ab

SOLUTION :

Given a > b > 1


(A) If a = 3 and b = 2, a – 1 = 2, which is a multiple of b.

(B) If a = 3 and b = 2, b + 1 = 3, which is a multiple of a.

(C) Since a positive multiple must be greater than or equal to the number, b - 1 cannot be a
multiple of a or b, as it is smaller than both integers a and b.

(D) If a = 4 and b = 2, a + b = 6, which is a multiple of b.

(E) If a = 3 and b = 2, ab = 6, which is a multiple of both a and b.

ANSWER: C
Divisibility
1. If k is a multiple of 24 but not a multiple of 16 , which of the following cannot be an integer ?
𝑘
A. 8
𝑘
B. 9
𝑘
C. 32
𝑘
D. 36
𝑘
E. 81

SOLUTION :

𝑘
A. When k = 24 , 8
will be an integer.
𝑘
B. When k = 72 , 9
will be an integer.
𝑘
C. As k can’t contain more than 3 powers of 2 , 32
will not be an integer.
𝑘
D. When k = 72 , 36
will be an integer.
𝑘
E. When k = 8×81 , 81
will be an integer.

ANSWER C
2. What is the number of integers that are not divisible by 7 in the range of integers from 3 to
500 ?
A. 70
B. 71
C. 426
D. 427
E. 428

SOLUTION :

Total number of integers from 3 to 500 = 498


Number of multiples of 7 between 3 and 500 = 71
Number of integers in the range 3 to 500 that are not divisible by 7 = 498 - 71 = 427

ANSWER D
3. If 3x(52) is divided by 35(53), the quotient terminates with one decimal digit. If x > 0, which of
the following statements must be true?

A. x is even
B. x is odd
C. x < 5
D. x ≥ 5
E. x = 5

SOLUTION :
(𝑥−5)
3
As per the question, (3x52)/(3553) = 5
, the quotient terminates with 1 decimal digit . So , we
are sure that x ≥ 5.

If x<5, we get a non-terminating recurring decimal


Example: For x=4, (3452)/(3553)=1/(3×5)=1/15=0.0666

ANSWER D
4. If M is the product of all positive integers greater than 59 and less than 71, then what is the
𝑀
greatest integer n for which 𝑛 is an integer?
6
A. 5
B. 7
C. 9
D. 11
E. 13

SOLUTION:

We need to find the power of of 6 in 60*61*62*63*64*65*66*67*68*69*70.


6 = 2*3.
Since the power of 2 is higher in 60*61*62*63*64*65*66*67*68*69*70, than the power of 3,
then we'll have as many 6's as there are 3's.

So, basically we need to find the powers of 3 in 60*61*62*63*64*65*66*67*68*69*70

One power of 3 in 60
Two powers of 3 in 63
One power of 3 in 66
One power of 3 in 69
Total of five 3's.
Therefore the power of 3, as well as power of 6 in 60*61*62*63*64*65*66*67*68*69*70 is 5.

ANSWER A
2
𝑡
5. If t is divisible by 12, what is the least possible integer value of a for which 𝑎 might not be an
2
integer ?

A. 2
B. 3
C. 4
D. 5
E. 6

SOLUTION :
t = 12k
t = 22×31×k
t2 = 24×32×k2
t2/2a = (24×32×k2)/2a

2
𝑡
In order to find least possible integer value of a for which 𝑎 might not be an integer , K should
2
not contain any powers of 2.

For a = 5, the expression won’t give an integer value.

ANSWER D
6. If 8A5146B is divisible by 88, then what is the value of BA ?

A.81
B.64
C.15
D.12
E.3

SOLUTION:

We need to check the divisibility by 8 and 11


The number is divisible by 8, when its last 3 digits are divisible by 8
So, 46B is divisible by 8
It is possible when B=4
The number is divisible by 11 when the difference of the sum of the digits in the odd position
and sum of the digits in the even position is 0 or 11
(B+17)-(7+A)=10 + (B - A) is a multiple of 11
Only possible value of (B-A) is 1 => 4-A=1 => A=3
BA=43=64

ANSWER B
7. Find the sum of the digits of the smallest 6-digit number divisible by 120.

A.3
B.12
C.6
D.9
E.7

SOLUTION :

Smallest 6 digit number= 1,00,000


Let us divide 1,00,000 by 120 and find the remainder
1,00,000=120×833+40
If we subtract 40 from 1,00,000 we get the highest 5 digit multiple of 120
1,00,000-120= 99960
Now, The smallest 6-digit number divisible by 120 is 99960+120=1,00,080
Sum of the digits: 1+0+0+0+8+0=9

ANSWER D
Dividend=Div×Quo+Rem
1. When m + n is divided by 9, the remainder is 1. If m is divisible by 9, which of the following
could equal the sum of the digits of n?

A. 34
B. 35
C. 36
D. 37
E. 38

SOLUTION :

We know that Dividend= Divisor× Quotient + Remainder


(m+n)=9× Quotient + 1
But m=9k
So, n must leave a remainder 1 when divided by 9
Now, A number is divisible by 9 if the sum of the digits of the number is divisible by 9
Which means if the number leaves a remainder 1 when divided by 9, the sum of the digits when
divided by 9 should also leave the same remainder.

From the options, 37 when divided by 9 leaves a remainder of 1.

ANSWER D
2. When the positive integer x is divided by 6 , the remainder is 4. Each of the following could
also be an integer EXCEPT

𝑥
A. 2
𝑥
B. 3
𝑥
C. 7
𝑥
D. 11
𝑥
E. 17

SOLUTION:

x = 6q + 4
Let us check the option choices :

(A) For q=1, x=10 , x/2 can be an integer

(B) x when divided by 3 will leave a remainder 1 ; x/3 is not an integer

(C) For q=4, x=28 , x/7 can be an integer

(D) For q=3, x=22 , x/11 can be an integer

(E) For q=5, x=34 , x/17 can be an integer

ANSWER B
3. A positive integer N leaves a remainder of 4 when divided by 33. How many different
remainders can N have, when N is divided by 55 ?

A. 6
B. 5
C. 4
D. 3
E. 2

SOLUTION :
N = 33a + 4 = 37 or 70 or 103 or 136 or 169 or 202 and so on…(For a =1,2,3,... and so on)
Remainder when 37 is divided by 55 = 37
Remainder when 70 is divided by 55 = 15
Remainder when 103 is divided by 55 = 48
Remainder when 136 is divided by 55 = 26
Remainder when 169 is divided by 55 = 4
Remainder when 202 is divided by 55 = 37 (Started repeating)
i.e. Total 5 different remainders

ANSWER B
𝑠
4. If s and t are positive integers such that 𝑡
= 64.12 , which of the following could be the
remainder when s is divided by t ?

A. 2
B. 4
C. 8
D. 20
E. 45

SOLUTION :
As per the question: s= t×q+r
s/t = q + r/t
s/t = 64 + 0.12
So, r/t = 12/100
r/t = 3/25
r is a multiple of 3.
From the options , 45 could be the remainder.

ANSWER E
5. A number when divided by a divisor leaves a remainder of 24. When twice the original
number is divided by the same divisor, the remainder is 11. What is the value of the divisor?

A. 12
B. 13
C. 35
D. 37
E. 59

SOLUTION :
We know that Dividend= Divisor× Quotient+ Remainder
Let the number be x
x=Divisor×Quotient 1 + 24…..(i)
2x=Divisor × Quotient 2 + 11…..(ii)
Now, 2× Equation (i)- Equation (ii)
Divisor × (2 Quotient 1 - Quotient 2) = 37

As 37 is a prime number, divisor must be 37

ANSWER D
Power of Primes in Factorial

1. What is the highest power of 111 which divides


30! × 31! × 32! × 33! × 34! × 35! × 36! × 37! × 38! × 39! × 40! ?

A. 3
B. 4
C. 5
D. 6
E. 7

SOLUTION :

111 = 3×37
Now, 3 is available in plenty, so let us focus on 37
37! will have 1 power of 37.
Same goes for 38!, 39! and 40!

So, 37 occurs 4 times which implies that highest power of 111 that divides the expression is 4

ANSWER B
2. How many positive factors of the number 28×36×54×105 are multiples of 120 ?

A. 540
B. 594
C. 660
D. 668
E. 792

SOLUTION :

120k= 28*36∗54∗105
120 k=213*36∗59
Dividing it by 120 we get
k=210*35∗58
Total number of factors of k : (10+1)*(5+1)*(8+1)=11*6*9 = 594
So , we will have 594 such instances where we get multiples of 120.

ANSWER B
3. For every positive integer x, f(x) represents the greatest prime factor of x!, and g(x)
represents the smallest prime factor of 2x+1. What is (g(f(12)) ?

A. 2
B. 3
C. 5
D. 7
E. 11

SOLUTION :

f(x): greatest prime factor of x!


g(x): smallest prime factor of 2x+1

f(12): greatest prime factor of 12! = 12*11*10*9*8*7*6*5*4*3*2*1 -> 11


(g(f(12)) = g(11) = smallest prime factor of 211 + 1 = 2049

As sum of the digits is divisible by 3 , the number will be divisible by 3.

Therefore, (g(f(12)) = 3 (smallest prime factor)

ANSWER B
4. What is the highest integer power of 30 that can divide 31!?
A. 1
B. 7
C. 14
D. 15
E. 26

SOLUTION :

We know 30=2∗3∗5
The least frequent factor will be 5, so we only need to count the powers of 5 in 31!

Total powers of 5 = 6 + 1 = 7

So, the highest power of 5 that divides 31! is 7.

So, the highest power of 30 that divides 31! is 7.

ANSWER B
𝑦 2
5. If 6 is a factor of (10!) , What is the greatest possible value of y ?

A. 2
B. 4
C. 6
D. 8
E. 16

SOLUTION :

6y can be written as 2y∗3y


To find the greatest possible value of y, we need to find out how many 3's are there in (10!)2
As powers of 2 will be available in plenty.

Powers of 3 in 10! = 3 + 1 = 4
2
So , Powers of 3 in (10!) = 8 (Each 10! contains 4 powers of 3)

Therefore, the maximum number of 6's that we can make is 8


Hence, the value of y = 8

ANSWER D
Units digit Cyclicity

1. Calculate the units digit of the following expression:


1!1 + 2!2+ 3!3 + 4!4+ 5!5…..+10!10

A.1
B.3
C.5
D.7
E.9

SOLUTION :
The unit digit of the expression will depend on the unit digits of the factorials and the powers of
each term. While simplifying the factorials of the numbers we will observe the following:

1!=1
2!=2
3!=6
4!=24: Units digit =4
5!=120: Units digit =0
We do not need to calculate any further since after 4!, the value of the next terms will end with
zeroes.
The unit digit of the expression will be the sum of all the individual unit digits calculated.
Units digit of 1!1+2!2+3!3+4!4 = 1 + 4 + 6 + 6 = 17
It implies that the unit digit is 7

ANSWER D
2. The last digit of 1212 + 1313– 1414×1515 =
A. 0
B. 1
C. 5
D. 8
E. 9

SOLUTION :
The unit digits of powers of 2 follow a cycle of 2, 4, 8, 6. Therefore the unit digit of 1212 is 6.
The unit digits of powers of 3 follow a cycle of 3, 9, 7, 1. Therefore the unit digit of 1313 is 3.
The unit digits of 4 follow a cycle of 4, 6. Hence the unit digit of 1414 is 6.
The unit digit of any power of 5 remains 5 so the last digit of 1515 will be 5.

This implies that the last digit of 1212+1313 will be 9 (i.e. (A number ending in 6) + (A number
ending in 3) gives a number ending in 9) and that of 1414×1515 will be 0.

Since 1414×1515 is definitely much greater than (1212+1313),


So we have
Number ending in 9 - Number ending in 0 (much bigger) so the result will end in 1 and will be
negative. I.e. (1212+1313)–(1414×1515) will be a negative number with a units digit of 1.

ANSWER B
96
3. What is the remainder when 4 is divided by 6?

A. 0
B. 1
C. 2
D. 3
E. 4

SOLUTION :

The cyclicity rule for 4 is 4,6...

41 divided by 6 leaves remainder = 4


42 divided by 6 leaves remainder = 4
43 divided by 6 leaves remainder = 4
44 divided by 6 leaves remainder = 4

so at 496, we would get units digit as 6

ANSWER E
28 47 19
4. What is the units digit of the product 32 × 33 × 37 ?

A. 0
B. 2
C. 4
D. 6
E. 8

SOLUTION :

Cyclicity of number 2 = 4
So, 3228 will have units digit 6

Cyclicity of number 3 = 4
So, 3347=3344+3 will have units digit 7(i.e. For 33)

Cyclicity of number 7 = 4
So, 3719=3716+3 will have units digit 3(i.e. For 73)

Thus, the unit digit of 3228∗3347∗3719 = (XX6)∗(XX7)∗ (XX3) = 6

ANSWER D
Rounding Off

1. [x] denotes the greatest integer less than or equal to X. If X is a positive integer and

𝑥 𝑥
[ 5 ] − [ 7 ] = 1. If the minimum value of x is ‘a’ and the maximum value is ‘b’, then a + b = ?

A. 40
B. 35
C. 34
D. 33
E. 32

SOLUTION :
[x/5] − [x/7] should be such that the difference is always '1'.

So minimum possible value of x is 5 since


[5/5] = [1.0] = 1
[5/7] = [0.71] = 0

a=5

Similarly, maximum possible value of x is 29 since values beyond 29 doesn't satisfy the condition
[x/5] − [x/7] = 1
[29/5] = [5.8] = 5
[29/7] = [4.1] = 4

b = 29

a + b = 5 + 29 = 34

ANSWER C
2. [x] denotes the largest integer less than or equal to x. If [x] = -1, which of the following must
be true ?

A. x = 1
B. −2 ≤ x <−1
C. −2 < x ≤ −1
D. −1 ≤ x< 0
E. −1< x ≤ 0

SOLUTION :

Let’s break down the problem step by step.

The notation ([x]) represents the largest integer less than or equal to (x). For example:

([2.7] = 2) because 2 is the largest integer less than or equal to 2.7.

([3] = 3) because 3 is the largest integer less than or equal to 3.

([-1.7] = -2) because -2 is the largest integer less than or equal to -1.7.

Given that ([x] = -1), we know that -1 is the largest integer less than or equal to (x). This implies
that (-1 ≤ x < 0).

ANSWER D
3. If [X] denotes the greatest integer less than or equal to x, what is the value of
[√1]+[√2]+[√3]+...+[√36] ?

A. 100
B. 111
C. 121
D. 126
E. 131

SOLUTION :

Perfect squares from 1 to 36 = 1, 4, 9, 16, 25, 36

[√1], [√2], [√3] → 3 terms will have a value 1.


We have [√1], [√2], [√3].
Since √1=1, √2 approx 1.41) (rounded off to 1), and √3 approx 1.73) (rounded off to 1),
we get 3 terms with a value of 1.

[√4], [√5], …, [√8] → 5 terms will have a value 2.

[√9], [√10], …, [√15] → 7 terms will have a value 3

Continuing this pattern, we find that there are:


9 terms with a value of 4
11 terms with a value of 5
Finally, there is 1 more term, the last term, with a value of 6.

Now let’s calculate the sum of all 36 terms :


=> 1 x 3 + 2 x 5 + 3 x 7 + 4 x 9 + 5 x 11 + 6 x 1
=> 3 + 10 + 21 + 36 + 55 + 6
=> 131

ANSWER E
4. If X is the hundredths digit in the decimal 0.1X and if Y is the thousandths digit in the decimal
0.02Y, where X and Y are nonzero digits, which of the following is closest to the greatest possible
0.1𝑋
value of 0.02𝑌
?

A. 4
B. 5
C. 6
D. 9
E. 10

SOLUTION :
Given two decimals:

0.1X and 0.02Y

To make 0.1X the greatest, we can let X = 9 and we have: 0.19

To make 0.02Y the smallest, we can make Y = 1 (since Y = 0 is not allowed) and we have: 0.021

Thus: 0.19/0.021 = 190/21 = 9.XX (This value is not greater than 9.5)

So, the greatest possible value is about 9


ANSWER D
5. When x is rounded to the nearest tenth the result is 1.5, and when y is rounded to the
nearest tenth the result is 7.5. If z = y − x, which of the following inequalities gives all of the
possible values of z ?

A. 5 < z < 7
B. 5.5 < z < 6.5
C. 5.9 < z < 6.0
D. 5.9 < z < 6.1
E. 5.95 < z < 6.05

SOLUTION :
According to the conventional rule of rounding, x and y must be in the following intervals:

1.45 ≤ x < 1.55

7.45 ≤ y < 7.55

If we multiply the first inequality by -1, we get:

-1.45 ≥ -x > -1.55, or -1.55 < -x ≤ -1.45

Since the directions of the inequality signs now face the same directions, we can add the
inequality for y and the transformed inequality for x and get y-x :

7.45 ≤ y < 7.55


-1.55 < -x ≤ -1.45

=> 5.9 < y- x < 6.1, or


=> 5.9 < z < 6.1

ANSWER D
Factor Theorem

1. Check whether 2y + 1 is a factor of the polynomial 4y3 + 4y2 – y – 1 using the factor theorem.

SOLUTION :
Let's equate 2y + 1 = 0.
∴ y = -1/2
Substitute y = -1/2 in the given polynomial equation 4y3 + 4y2 – y – 1.
⟹ 4( -1/2)3 + 4(-1/2)2 – (-1/2) – 1
=> -1/2 + 1 + 1/2 – 1
=> 0
The remainder = 0, thus, 2y + 1 is a factor of the polynomial equation 4y3 + 4y2 – y – 1.

2. Use the factor theorem to check whether y + 1 is a factor of the polynomial 3y4 + y3 – y2 + 3y +
2, or not.

SOLUTION :

Given y + 1; then, y + 1 = 0, we get y = -1.


Substitute y = -1 in the given polynomial equation 3y4 + y3 – y2 + 3y + 2.
⟹ 3(–1)4 + (–1)3 – (–1)2 +3(–1) + 2
=> 3(1) + (–1) – 1 – 3 + 2
=> 3 -1 -1 -3 + 2
Adding all positive terms and all negative terms, we get,
=> 5 - 5
=> 0
Therefore, we can say that y + 1 is a factor of 3y4 + y3 – y2 + 3y + 2.
Algebra Extra Question Solutions

Functions
1. Which of the following functions satisfies f(a+b) = f(a) × f(b) for all positive numbers a and b ?

A. f(x) = x+1
B. f(x) = x2+1
C. f(x) = √x
1
D. f(x) = 𝑥
E. f(x) = 2x

SOLUTION :

Given a and b are positive numbers


Let us check the options:
A. f(x) = x+1
LHS
f(a+b)= a+b+1=4
RHS
f(a) × f(b)=(a+1) × (b+1)= ab+a+b+1
LHS≠RHS

B. f(x)=x2+1
LHS
f(a+b)= (a+b)2+1=a2+b2+2ab+1
RHS
f(a) × f(b)= (a2+1) × (b2+1)=a2b2+a2+b2+1
LHS≠RHS

C. f(x)=√x
f(a+b)=√(a+b)
f(a) × f(b)= √a × √b = √ab
LHS≠RHS
D. f(x)=1/x
f(a+b)= 1/(a+b)
1 1 1
f(a) × f(b)= 𝑎
× 𝑏
= 𝑎𝑏
LHS≠RHS

E. f(x)=2x
f(a+b)=2(a+b)
f(a) × f(b)= 2a × 2b= 2(a+b)
LHS = RHS
ANSWER E
2. The functions f(x) and g(x) are defined by f(x)= x2–1 and g(x)=1–2x. Given that f(g(k))= 3,
which of the following could be the value of k ?

1
A. 2
√3
B. 2
C. 1
3
D. 2
E. -1

SOLUTION :
g(k) = 1- 2k
f(g(k))=(1-2k)2 - 1
Now, f(g(k)) = 3
(1-2k)2-1 = 3
(1-2k)2= 4
2
4𝑘 - 4k + 1 = 4
2
4𝑘 - 4k - 3 = 0
2
4𝑘 - 6k + 2k - 3 = 0
2k(2k-3) + 1(2k-3) = 0
(2k+1)(2k-3) = 0
−1 3
k= 2
OR 2

ANSWER D
3. For which of the following functions does f(x) = f(2−x)?

A. f(x) = x+2
B. f(x) = 2x−x2
C. f(x) = 2−x
D. f(x) = (2−x)2
E. f(x) = x2

SOLUTION :
Let x = 0. We need to choose the option for which f(0)=f(2)
A. f(x)=x+2
f(0)=0+2=2
f(2)=2+2=4
LHS≠RHS

B. f(x)=2x−x2
f(0)=2(0)-(0)2=0
f(2)=2(2)-(2)2= 4-4 = 0
LHS=RHS

C. f(x)=2−x
f(0)=2-0 = 2
f(2)=2-2 = 0
LHS ≠ RHS

D. f(x)=(2−x)2
f(0)=(2-0)2= 4
f(2)=(2-2)2= 0
LHS ≠ RHS

E. f(x)=x2
f(0)= 02=0
f(2)=22=4
LHS≠RHS

ANSWER B
4. For every even positive integer m, f(m) represents the product of all even integers from 2 to
m, inclusive. For example, f(12) = 2 x 4 x 6 x 8 x 10 x 12. What is the greatest prime factor of
f(24) ?

A. 23
B. 19
C. 17
D. 13
E. 11

SOLUTION:
f(24)=2 × 4 × 6 × 8 × 10 × 12 × 14 × 16 × 18 × 20 × 22 × 24
f(24)= 212 (1 × 2 × 3 × 4 × 5 × 6 × 7 × 8 × 9 × 10 × 11 × 12)

Therefore the greatest prime factor is 11.

ANSWER E
5. The function f(m) is defined for all positive integers m as the product of m + 4, m+5, and m +
6. If n is a positive integer, then f(n) must be divisible by which one of the following numbers?

A. 4
B. 5
C. 6
D. 7
E. 11

SOLUTION :
f(m)= (m + 4)×(m + 5)×(m + 6)
f(n)= (n + 4)×(n + 5)×(n + 6)
So, f(n) is the product of three consecutive integers.
Out of the three integers at least one will be even and at least one will be divisible by 3
So, f(n) must be divisible by 6

ANSWER C
6. m and n are positive integers and n > m. Function $ is defined as m $ n = sum of all integers
greater than m but less than n.
What is the value of (107 $ 119)-(109 $ 120) ?

A. -123
B. -2
C. 45
D. 95
E. 98

SOLUTION:
107 $ 119 = 108 + 109 +110 +111 +112 +113 +114 + 115 + 116 + 117 +118
(Sum of all integers greater than 107 but less than 119.)

109 $ 120 = 110 +111 +112 +113 +114 + 115 + 116 + 117 +118 + 119
(Sum of all integers greater than 109 but less than 120.)

(107 $ 119)- (109 $120) = (108 + 109 +110 +111 +112 +113 +114 + 115 + 116 + 117 +118 ) - (110
+111 +112 +113 +114 + 115 + 116 + 117 +118 + 119)
= 108 + 109 -119
= 98

ANSWER E
Arithmetic Progression

1. In an arithmetic progression, the 10th term is 11 and the 11th term is 10.How many
consecutive terms(starting from the first term) of the arithmetic progression should be
considered so as to make their sum equal to zero?
A.33
B.41
C.37
D.39
E.42

SOLUTION :
a + 9d = 11…(i)
a + 10d = 10…(ii)
Subtracting (ii) and (i):
d = -1
Now, Let us substitute the value of d in equation (i)
a + 9(-1) = 11
a = 20
We need to find the number of terms such that the sum of the first n terms is equal to 0.
𝑛
( 2 )×[2a+(n-1)d]=0
𝑛
( 2 )×[40+(n-1)(-1)]=0
𝑛
( 2 )×[40-n+1]=0
𝑛
( 2 )×[41-n]=0
41n-n2=0
n(n-41)=0
n=0 or 41

ANSWER B
2. The sum of nine consecutive odd numbers of Set A is 621. What is the sum of a different set
of six consecutive even numbers whose lowest number is 15 more than the lowest number of
Set A?

A. 498
B. 488
C. 486
D. 478
E. 468

SOLUTION :
In an AP , Mean = Median = Middle term in this case as the number of terms is odd.
𝑆𝑢𝑚 621
Mean = 𝑛
= 9
= 69 = 5th number
So , the lowest number or the first number will be 69 - 4×2 = 61 (Distance between each
consecutive term is 2)

Now, First term of the second sequence(6 consecutive even integers) = 61+15 =76
Last term of the second sequence = 86
Sum = number of terms × Average of equidistant terms
𝑛
Sum= ( 2 ) × [First term + Last term]
6
Sum= ( 2 ) × [76 + 86]
Sum= 486

ANSWER C
3. If the sum of the first 51 terms of arithmetic progression is zero, then which of the following
must be true ? (An arithmetic progression is a sequence of numbers such that the difference
between the consecutive terms is constant)

I. 51st largest term is zero


II. 26th largest term is zero
III. All terms are non negative

A. I
B. II
C. III
D. I and II
E. None of the above

SOLUTION :
If the sum of the first 51 terms of arithmetic progression is zero :
Sum = number of terms × Average of equidistant terms
=> 0 = 51×(1st + 51st OR 2nd + 50th OR 3rd + 49th OR 4th + 48th OR 26th term which is the
MIDDLE TERM)
=> 1st + 51st = 2nd + 50th = 3rd + 49th = 4th + 48th = 26th term = 0

The sequence could be increasing or decreasing.


Example :
Sequence 1 : -25 , -24 , . . . . . . . . . . . .0(26th term) , 1 , 2 , 3 , . . . . . . . . . . . .+25.
(Increasing Sequence)
Sequence 2 : 25 , 24 , . . . . . . . . . . . .0(26th term) , -1 , -2 , -3 , . . . . . . . . . . . .-25.
(Decreasing Sequence)

Statement I is not true.


Statement II is true for both the cases.
Statement III is not true.

ANSWER B
4. A club consists of members whose ages are in arithmetic progression (an arithmetic
progression is a sequence of numbers such that the difference between the consecutive terms
is constant). The common difference being 3 months. If the youngest member of the club is just
7 years old and the sum of the ages of all the members is 250, then number of members in the
club are:

A. 18
B. 20
C. 22
D. 24
E. 25

SOLUTION :

First term = 7 years


Common difference = 3 months or 3/12 years = ¼ years
𝑛
Sum= ( 2 ) × [2a+(n-1)d]
𝑛 1
250= ( 2 ) × [2*7+(n-1) * 4
]
𝑛 1
500= n × [14 + 4
- 4
]
2000= 56n + n2- n
n2+55n-2000=0
n2+80n-25n-2000=0
(n+80)(n-25)=0
n= 25 or -80

ANSWER E
5. The nth term (𝑡𝑛) of a certain sequence is defined as 𝑡𝑛 = 𝑡𝑛−1 + 4 . If t1 = −7 , then 𝑡71 =
A. 273
B. 277
C. 281
D. 283
E. 287

SOLUTION :

The given sequence is an AP having a common difference of 4.


First term, t1= -7
nth term is given by a+(n-1)d
So, t71= a + (71-1)d = a + 70d
t71= -7 + 70 × 4 = 273

ANSWER A
6. The sum of three numbers in an arithmetic progression is 36. The sum of the squares of the
three numbers is 464. Find the smallest number ?

A.8
B.10
C.12
D.16
E.13

SOLUTION :

Let the three terms be a-d, a and a+d


a-d + a + a+d = 36
3a= 36
a = 12

(a - d)2+a2+(a + d)2 = 464


3a2+2d2 = 464
3(12)2+2d2= 464
2d2= 32
d2=16
d = 4 OR -4

For both the cases , the three terms will be 8, 12 and 16.

Smallest term = 8

ANSWER A
7. Dave drove a total of 2250 miles over a period of 15 days. Each day after the first he
increased the number of miles he drove in a single day by one mile. What is the difference
between the average (arithmetic mean) number of miles he drove per day and the median
number of miles driven over the course of the 15 days?

A. 0
B. 1
C. 5
D. 15
E. 25

SOLUTION :

Each day after the first he increased the number of miles he drove in a single day by one mile.
This sentence suggests that the sequence will be in AP.
We know that in an AP , Mean = Median.
=> Mean - Median = 0 (NO NEED TO CALCULATE)

ANSWER A
Geometric Progression

1. At the start of an experiment, a certain population consisted of 3 animals. At the end of each
month after the start of the experiment, the population size was double its size at the beginning
of that month. Which of the following represents the population size at the end of 10 months?

A. 23
B. 32
C. 2(310)
D. 3(210)
E. 3(102)

SOLUTION :
Initial population = 3
At the end of the first month, the population will be 3 × 2
Similarly, at the end of the second month, the population will be 3 × 22
At the end of the nth month, the population will be 3 × 2n
So, when n = 10 , the population will be 3 × 210
ANSWER D
2. The turnover of an organization named 'Coverage' was $640 crore per annum in 2001. In the
same year another organization named 'Miles', established just a few years back, had a turnover
of $40 crore per annum. The turnover of 'Miles' increased by 300% every year whereas
Coverage's turnover increased by 700% every two years. After how many years will both
companies have the same turnover?

A. 4 years
B. 6 years
C. 8 years
D. 10 years
E. 12 years

SOLUTION:
Say after ‘n’ years , both the companies have the same turnover.
Coverage :
Initial = 640
1
After 2 years = 640×8
2
After 4 years = 640×8
3
After 6 years = 640×8
𝑛
2
After n years = 640×8

Miles :
Initial = 40
1
After 1 years = 40×4
2
After 2 years = 40×4
3
After 3 years = 40×4
𝑛
After n years = 40×4

After how many years will both companies have the same turnover?
𝑛
2 𝑛
Equate : 640×8 = 40×4
𝑛
𝑛
=> 16×8 2 = 4
3𝑛
4 2 2𝑛
=> 2 × 2 =2
3𝑛
4+ 2 2𝑛
=> 2 =2
3𝑛
=> 4 + 2
= 2n
=> n = 8

ANSWER C
3.
v, w, x, y, z
A geometric sequence is a sequence in which each term after the first is equal to the product of
the preceding term and a constant. If the list of numbers shown above is a geometric sequence,
which of the following must also be a geometric sequence?

I. 2v, 2w,2x,2y,2z
II. v+2, w+2, x+2, y+2, z+2
III. √v, √w, √x, √y, √z

A. I only
B. II only
C. III only
D. I and II
E. I and III

SOLUTION :
Let the numbers be -1, -3, -9, -27, -81 with r = 3

Check the option choices;

I. -2 , -6 , -18 , -54 ,-162 : GP with common ratio 3


II. 3 , 5 , 11, 29 , 83 : Clearly not a G.P
III. Square root can’t be negative

ANSWER A
4. 5 and 15 are the first two terms in a geometric sequence. What is the absolute difference
between the 11th term and the 13th term?

A. 3×52
B. 5× 313 - 5× 311
C. 513
D. 40 × 310
E. 312 - 310

SOLUTION :
This is an increasing GP

For a given Geometric Sequence, the


nth term is tn=a×rn−1 , where a is the first term and r is the common ratio.

5 and 15 are the first two terms in a GP.


Second term , 15 = 5 ×r where r is the common term, Hence, r comes out to be 3.

From the given problem,


t11=5×310 and t13=5×312

Thus, the absolute difference is :


=>13th term - 11th term
=> 5×310×(9−1)
=> 40×310

ANSWER D
1 1 1 1
5. If r = 1 + 3
+ 9
+ 27
and S =1 + 3
r , then S exceeds r by

1
A. 3
1
B. 6
1
C. 9
1
D. 27
1
E. 81

SOLUTION:
Given:
1 1 1
r=1+ 3
+ 9
+ 27
1
S =1+ 3r
1 1 1 1
S = 1 + 3 (1 + 3
+ 9
+ 27
)
1 1 1 1
S =1+ 3 + 9
+ 27
+ 81
1
S = r + 81
1
S - r = 81

ANSWER E
6. x, y, z are three positive numbers in a geometric progression such that x < y < z, while 5x, 16y,
and 12z are in an arithmetic progression. What is the common ratio of the geometric
progression ?

3
A. 6
3
B. 2
5
C. 2
1
D. 6
3
E. 8

SOLUTION :

As x,y,z are in GP and x < y < z (Common ratio = r > 1)


x=a
y = ar
2
z = a𝑟

5x, 16y, and 12z are in an arithmetic progression


5𝑥 + 12𝑧
=> 16y = 2
=> 32y = 5x + 12z
2
=> 32ar = 5a + 12a𝑟 (Divide the equation by a)
2
=> 12𝑟 - 32r + 5 = 0
2
32 ± 32 −4(12)(5) 32 ± 28 60 4
=> r = 24
= 24
= 24
OR 24
(this will get rejected as r > 1)
60 5
r= 24
= 2

ANSWER C
7. The numbers –2, x, –32 are the first three terms in a geometric progression. Which of the
following could be the sixth term in the progression?

A. –4,096
B. –2,048
C. –1,024
D. 512
E. 1,024

SOLUTION :
Let the first three terms of the GP be a, ar and ar2
So, a = -2
ar2= -32
(-2)×r2 = -32
r2 = 16
r = 4 OR -4

5 5 5
Sixth term = a𝑟 = (-2)(4) OR (-2)(− 4) = - 2048 OR +2048

ANSWER B
User-Defined Sequence

1. The first two terms of a sequence are -3 and 2. Subsequent odd-numbered terms are given by
adding 1 to the previous term, and subsequent even-numbered terms are given by multiplying
the previous term by -1. What is the sum of the first 147 terms?

A. -3
B. -2
C. -1
D. 1
E. 2

SOLUTION :
Let’s find the pattern for this sequence.
𝑡1 = -3
𝑡2 = 2
𝑡3 = 2 + 1 = 3 (Subsequent odd-numbered terms are given by adding 1 to the previous term)
𝑡4 = - 3 (Subsequent even-numbered terms are given by multiplying the previous term by -1)
𝑡5 = -3 + 1 = -2 (Subsequent odd-numbered terms are given by adding 1 to the previous term)
𝑡6= 2 (Subsequent even-numbered terms are given by multiplying the previous term by -1)
𝑡7= 2 + 1 = 3 (Subsequent odd-numbered terms are given by adding 1 to the previous term)
𝑡8 = -3
𝑡9 = -2

Sum of 𝑡3 + 𝑡4 + 𝑡5 + 𝑡6 = 𝑡7 + 𝑡8 + 𝑡9 + 𝑡10 = 𝑡11 + 𝑡12 + 𝑡13 + 𝑡14= and so on = 0


Sum of 147 terms = 1st term + 2nd term + (Sum of the next 144 terms = 0) + 147th term
147th term will be the same as 𝑡3 = 3.

Sum of 147 terms = -3 + 2 + 0 + 3 = 2.

ANSWER E
2. A sequence is given by the rule 𝑎𝑛=|𝑎𝑛−2|−|𝑎𝑛−1| for all n ≥ 3, where a1= 0 and a2 = 3. What
is 𝑎99 ?

A. -3
B. 0
C. 3
D. 201
E. 303

SOLUTION :
Since a1 = 0 and a2 = 3 and 𝑎𝑛=|𝑎𝑛−2|−|𝑎𝑛−1| for all n ≥ 3, we see that:
a1 = 0

a2 = 3

a3 = |a1| - |a2| = |0| - |3| = -3

a4 = |a2| - |a3| = |3| - |-3| = 0

a5 = |a3| - |a4| = |-3| - |0| = 3

a6 = |a4| - |a5| = |0| - |3| = -3

We can see that the cycle {0, 3, -3} repeats every three terms, and an = -3 when n is a multiple of
3, So 𝑎99 = -3

ANSWER A
3. Find the sum of the first 20 terms of this series which begins this way: -12 + 22 - 32 + 42 - 52 + 62

A. 210
B. 330
C. 519
D. 720
E. 190

SOLUTION :

Identify the form of the equation: (x2−y2) =(x−y)(x+y)

Rewrite the sequence:


(22−12) + (42−32)… + . . . .- 192 + 202
(2−1)(2+1) + (4−3)(4+3) + . . . . .(20 - 19)(20 + 19)
(2 + 1)+(4 + 3) + . . . . .. (20 + 19)

Rearranging a bit :
1 + 2+ 3 + 4 + 5… + 19 + 20

(1+20)
The sum of first 20 numbers : 20 × 2
= 210

ANSWER A
4. Each number Sn in a sequence can be expressed as a function of the preceding number (Sn–1)
2
as follows: Sn = 3
×Sn-1 – 4. Which of the following equations correctly expresses the value of Sn
in this sequence in terms of Sn+2 ?

9
A. Sn= 4
×Sn+2 + 18
4
B. Sn= 9
×Sn+2 + 15
9
C. Sn= 4
×Sn+2 + 15
4
D. Sn= 9
×Sn+2 − 8
2
E. Sn= 3
×Sn+2 − 8

SOLUTION:

2
𝑆𝑛 = 3
𝑆𝑛−1 - 4 (GIVEN)

Putting n = n + 1 , we get

2
𝑆𝑛+1 = 3
𝑆𝑛 - 4 - - - - - - -(I)
2
𝑆𝑛+2 = 3
𝑆𝑛+1 - 4 - - - - - (II)

Put the value of 𝑆𝑛+1 from Equation (I) into (II)


2 2
𝑆𝑛+2 = 3
( 3 𝑆𝑛 - 4) - 4
4 8
𝑆𝑛+2 = 9
𝑆𝑛 - 3
- 4
4 20
𝑆𝑛+2 = 9
𝑆𝑛 - 3

20 4
𝑆𝑛+2 + 3
= 9
𝑆𝑛

3𝑆𝑛+2 + 20 4
3
= 9
𝑆𝑛

9
𝑆𝑛 = 4
𝑆𝑛+2 + 15

ANSWER C
5. The sequence a1, a2 , … , an is such that an= 2an−1− x for all positive integers n ≥ 2 and for a
certain number x. If a5 = 99 and a3 = 27, what is the value of x ?

A. 3
B. 9
C. 18
D. 36
E. 45

SOLUTION :

Given
an= 2an−1 − x

a5 = 99
a3 = 27

an= 2an−1 − x
a5 = 2a4 − x - - - - - - -(I)
a4 = 2a3 − x - - - - - - - (II)

Putting the value of a4 from Equation (II) into Equation (I)


a5 = 2a4 − x - - - - - - -(I)
99 = 2(2a3−x) − x
4a3−3x = 99
4(27) − 3x = 99
3x = 108 − 99
3x = 9
=> x = 3

ANSWER A
6. If each term in the sum a1 + a2 + a3 +... + an is either 2 or 22 and the sum equals 100, which of
the following could be equal to n?

A. 38
B. 39
C. 40
D. 41
E. 42

SOLUTION :
Let there be ‘a’ terms of 22 and ‘b’ terms for 2.
We know that there are a total of n terms => a+b = n

We are given 22a + 2b = 100


100 − 22𝑎
=> b = 2
= 50 - 11a
Non- negative Values of a and b that satisfy the equation :
a = 0 , b = 50 , total = 50
a = 1 , b = 39 , total = 40
a = 2 , b = 28 , total = 30
a = 3 , b = 17 , total = 20
a = 4 , b = 6 , total = 10

ANSWER C
7. What is the difference between the fourth and third terms of the sequence defined by
an = 3n − n2 ?

A.18
B. 23
C. 47
D. 65
E. 83

SOLUTION :

Let’s start with the given equation


an = 3n − n2

a4 = 34−42 = 81 − 16 = 65
a3 = 33−32 = 27 − 9 =18

a4 − a3 = 65 − 18 = 47

ANSWER C
𝑛
8. A sequence is defined as follows: an= 𝑛+1
. If n is a positive integer, then how many of the first
100 terms of this sequence are less than 0.891 ?

A. 7
B. 8
C. 9
D. 10
E. 12

SOLUTION :
Write down the first 10 terms of the sequence :
1 2 3 4 5 6 7 8 9 10
, ,
2 3 4
, 5
, 6
, 7
, 8
, 9
, 10
, 11

Notice that any term in this sequence is larger than its predecessor. To determine how many
terms are less than 0.891, we identify the first term that exceeds this value.

9 8
The 9th term, 10
, equals 0.9. Since the term before it, 9
, is only 0.888...,

Hence only the first 8 terms of the sequence are less than 0.891.

ANSWER B
9. In a certain sequence, Sn represents the sum of all terms from t1 to tn , inclusive (for
n≥1) . If Sn= 3n2, what is the value of t7 ?

A. 36
B. 39
C. 45
D. 49
E. 108

SOLUTION:
Sn= 3n2
𝑆7 = 3(7)2
𝑆6 = 3(6)2

First, recognize that

𝑆7 = Sum of first 7 terms = 𝑡1 + 𝑡2+ 𝑡1 + . . . . . . . + 𝑡6 + 𝑡7 = 3(7)2


𝑆6 = Sum of first 6 terms = 𝑡1 + 𝑡2+ 𝑡1 + . . . . . . . + 𝑡6 = 3(6)2

2 2 2 2
𝑆7 - 𝑆6 = 3(7 ) - 3(6 ) = 3 ( 7 - 6 ) = 39

ANSWER B
Linear Equation
­1. 5 friends - Ajax, Betty, Cody, Donna and Elsie - decide to contribute to a charity. Ajax
1 1
contributes 6
th as much as all the others combined, Betty contributes 5
th as much as all the
1
others combined, Cody contributes 4
th as much as all the others combined and Donna
1
contributes 3
rd as much as all the others combined. If the total contribution of the friends is
$42,000 , then how much did Elsie pay ?

A $8100
B $9100
C $10000
D $10100
E $11000

SOLUTION :

A+B+C+D+E= $ 42,000…(i)

1
A= 6
× (B+C+D+E)
6A= B+C+D+E….(ii)

1
B= 5
× (A+C+D+E)
5B = A+C+D+E…..(iii)

1
C= 4
× (A+B+D+E)
4C= A+B+D+E…..(iv)

1
D= 3
× (A+B+C+E)
3D= A+B+C+E…..(v)

From (i) and (ii)


A+6A= $ 42,000
7A= $42,000
A= $ 6,000
From (i) and (iii)
B+5B= $42,000
6B= $ 42,000
B= $ 7,000

From (i) and (iv)


C+4C= $ 42,000
5C= $ 42,000
C= $ 8400

From (i) and (v)


D+3D= $ 42,000
4D= $ 42,000
D= $ 10,500

Substituting the values of A,B,C and D in (i)


A+B+C+D+E=$ 42,000
$ 6000 + $ 7000 + $ 8400 + $ 10,500 + E= $ 42,000
E= $ 10,100
Elsie paid $ 10,100

ANSWER D
2. Thabo owns exactly 140 books, and each book is either paperback fiction, paperback
nonfiction, or hardcover nonfiction. If he owns 20 more paperback nonfiction books than
hardcover nonfiction books, and twice as many paperback fiction books as paperback nonfiction
books, how many hardcover nonfiction books does Thabo own?

A.10
B.20
C.30
D.40
E.50

SOLUTION:
Let x = the number of paperback fiction books
y = the number of paperback nonfiction books
z = the number of hardcover nonfiction books

There are a total 140 books


x + y + z = 140…..(i)
There are 20 more paperback nonfiction books than hardcover nonfiction books
y = 20 + z…..(ii)
There are twice as many paperback fiction books as paperback nonfiction books
x = 2y…..(iii)
We need to find z
So, x + y + z = 140
2y + y + y - 20 = 140
4y = 160
y = 40

Substituting y in equation (ii)


40= 20 + z
z = 20

ANSWER B
3. If A + B = C, A + D = B, 2C = 3D and A, B, C, and D are positive integers, what is the least
possible value of B?

A. 1
B. 4
C. 5
D. 6
E. 10

SOLUTION:
A,B,C, and D are positive integers
A+B = C
A = C - B …(i)

A+ D = B…(ii)
Putting the value of A From (i), C-B + D = B
2B = C+D

We know, 2C = 3D
3
C= ( 2 ) D
3
So, 2B = ( 2 ) D + D
5
B=(4)D

B is a multiple of 5
So, the least value of B is 5

ANSWER C
4. Alice has 3 times the number of stamps that Doris does and Jane has 7 stamps more than
Doris does. If Alice, Doris, and Jane each increase their number of stamps by 4, which of the
following must be true after each person increases her number of stamps?

I.Alice has more stamps than Jane.


II.Jane has 3 more stamps than Doris.
III.The sum of the numbers of stamps that Alice and Doris have is a multiple of 4.

A.None
B.III only
C.I and III only
D.II and III only
E.I, II, and III

SOLUTION :
Let the number of stamps with Doris be x
So, Alice will have 3x stamps and Jane will have x+7
After the increase, Number of stamps with
Doris: x+4
Alice: 3x+4
Jane: x+7+4= x+11
Let us now look at the statements:

I. Alice has more stamps than Jane


3x + 4 > x +11
2x > 7
x >7/2
But , x may be greater than or less than 7/2.
So, the statement may or may not be TRUE.

II. Jane has 3 more stamps than Doris.


(x+11) - (x+4) = 3
7≠3
Not TRUE
III. The sum of the numbers of stamps that Alice and Doris have is a multiple of 4.
3x+4 + x+4
4x+8
4(x+2) : multiple of 4.
TRUE.

ANSWER B
5. Three years from now, Dathan will be three times as old as Ellen and Ellen will be six years
younger than Famke. If Dathan's age is three years less than twice Famke's age, how old is
Famke ?

A. 9
B. 15
C. 21
D. 27
E. 33

SOLUTION :
Let the present ages of Dathan, Ellen and Famke be x,y, and z respectively
If Dathan's age is three years less than twice Famke's age
x = 2z - 3….(i)
Three years from now, Dathan will be three times as old as Ellen
x+3 = 3(y+3)
x+3= 3y+9
x = 3y+6….(ii)

Three years from now, Ellen will be six years younger than Famke
And, (y+3) = (z+3)-6
y = z - 6….(iii)

From (ii) and (iii)


x = 3(z-6)+ 6
x = 3z-18 + 6
x = 3z-12 ,Substituting x in equation (i)
x = 2z - 3….(i)
3z - 12 = 2z - 3
z=9
Famke is 9 years old.

ANSWER A
Quadratic Equation
1. x2 + bx + 72 = 0 has two distinct positive integer roots. How many values are possible for b?

A. 3
B. 6
C. 8
D. 12
E. 24

SOLUTION :

x2 + bx + 72 = 0
Let the roots be α and β.
In our case, a is 1, b is the unknown we’re trying to find, and c is 72.
𝑐 72
Product of the roots = 𝑎
= 1
= αβ
3 2
αβ = 72 = 2 ×3
Total number of factors = (3+1)(2+1) = 12
(1,2,3,4,6,8,9,12,18,24,36,72)
αβ = 1×72 OR 2×36 OR 3×24 OR 4×18 OR 6×12 OR 8×9 (6 pairs)

So, Sum of the roots = - b = α + β =


How many values of α + β are possible ? => 6.
So , 6 values are possible for b.

ANSWER B
2. The temperature inside a certain industrial machine at time t seconds after startup, for 0 < t <
2𝑡+1 𝑡+2
10, is given by h(t) = 4 - 4 degrees Celsius. How many seconds after startup is the
temperature inside the machine equal to 128 degrees Celsius?

3
A. 2
B. 2
5
C. 2
D. 3
7
E. 2

SOLUTION:
We are given that the temperature inside a certain industrial machine at time t seconds after
2𝑡+1 𝑡+2
startup, for 0 < t < 10, is given by h(t) = 4 - 4 degrees Celsius. We need to determine how
many seconds after startup it takes the temperature inside the machine to equal 128 degrees
Celsius.

2𝑡+1 𝑡+2
So , 4 - 4 = 128

(42t)(41) - (4t)(42) = 128

4(42t) - 16(4t) - 128 = 0

Let x = 4t, the above equation becomes :


4x2 - 16x - 128 = 0
x2 - 4x - 32 = 0
(x - 8)(x + 4) = 0
x = 8 or x = -4
Since x = 4t, we have 4t = 8 or 4t = -4.
However, since 4 is positive, any power it raises will be positive also, so 4t can’t be equal to -4.
Thus, 4t = -4 IS REJECTED.
4t = 8
(22)t = 23
22t = 23
2t = 3
t = 3/2
ANSWER A
3. Which of the following equations has a root in common with x2− 6x + 5=0 ?
A. x2+1=0
B. x2−x−2=0
C. 2x2−2=0
D. x2−2x−3=0
E. x2−10x−5=0

SOLUTION:
The quadratic equation x2 - 6x + 5 = 0 factors into (x - 1)(x - 5) = 0
Thus, the roots are x = 1 and x = 5

Let’s do the same for each option:

Option A: The equation x2 + 1 = 0 simplifies to x2 = -1. The roots are imaginary.REJECT

Option B: The equation x2 - x - 2 = 0 factors into (x - 2)(x + 1) = 0

The roots are x = 2 And x = -1


None of these roots match the original equation.REJECT

Option C: The equation 2x2 - 2 = 0 simplifies to 2(x2 - 1) = 0 and then to (x2 - 1) = 0.This factors
into (x - 1)(x + 1) = 0

The roots are x = 1 And x = -1


The root x = 1 matches the root of the original equation.

There’s no need to do the rest, as we have our ANSWER.

Option D: The equation x2 - 2x - 3 = 0 factors into (x - 3)(x + 1) = 0

The roots are x = 3 And x = -1


There are no roots in common with the original equation.

Option E: The equation x2 - 10x - 5 = 0 doesn’t factor into integers, so there are no roots in
common with the original equation.

ANSWER C
4. What are the values of K for which the equation (3K+1)x2 + 2(K+1)x + 1 = 0 has equal roots?

A. 0 or 1
B. 0 or 2
C. 0 or 3
D. 1 or 2
E. 1 or 3

SOLUTION :
The given equation is of the form ax2+bx+c=0
The roots are equal, i.e., D = b2 - 4ac = 0
(2(K+1))2 - 4 × (3K+1) × 1 = 0
4(K2 + 2K + 1) = 4(3K + 1)
This further simplifies to K2 - K = 0, which factors into K(K - 1) = 0

Therefore, the roots are K = 0, 1

ANSWER A
5. Which of the following equations has 3+2√3 as one of its roots?

A. x2 + 6x + 3 = 0
B. x2 − 6x + 3 = 0
C. x2 + 6x − 3 = 0
D. x2− 6x − 3 = 0
E. x2− 4√3x − 3 = 0

SOLUTION :
If 3 + 2√3 is a root of the equation , it will satisfy the equation.
Put x = 3 + 2√3 in the option choices and check.
You will find that only Option D satisfies.

ALTERNATE :
If one of the roots of the equation is 3+2√3, then the other root will be 3-2√3 (THINK WHY !!)
So, Sum of the Roots = 6 and Product of the Roots = (3+2√3)(3-2√3) = -3
Only Option D satisfies.

ANSWER D
6. If q is one root of the equation x2 + 18x + 11c = 0, where –11 is the other root and c is a
constant, then q2-c2 =

A. 98
B. 72
C. 49
D. 0
E. It can't be determined from the information given

SOLUTION :
Since -11 is one of the roots of the equation, it satisfies the equation.

2
𝑥 + 18x + 11c = 0
=>(−11)2 + 18(−11) + 11c = 0
=> 121 - 198 + 11c = 0
=> 11c = 77
=> c = 7
So the equation becomes x2 + 18x + 77 = 0
We know that -11 and q are the 2 roots of the equation.
So, Product of the roots = -11q = 77 => q = -7
2 2 2 2
𝑞 - 𝑐 = (− 7) - ( 7 ) = 0

ANSWER D
Coordinate Geometry Graphs Extra Question Solutions

1.

In the rectangular coordinate system shown above, which quadrant, if any, contains no point
(x,y) that satisfies the inequality 2x − 3y ≤ −6 ?

A. None
B. I
C. II
D. III
E. IV

SOLUTION :
2x - 3y ≤ -6
2
y ≥ 3
x+2

Let us find the x-intercept and y-intercept of the line 2x-3y = -6


Put x = 0 to find the y-intercept
-3y =-6
=> y = 2
Putting y = 0 to find the x-intercept
2x = -6
=> x =-3
So, the line passes through the points (-3,0) and (0,2)
All the points that satisfy the above inequality lie above the given line.
So, Quadrant IV contains no point that satisfies the above inequality.

ANSWER E
2. A student draws 35 lines in the XY plane, none of which are vertical and 15 of which have a
positive slope. Of those with positive slope, 1/3 also have a positive Y-intercept. If 23 lines have
a Y-intercept less than or equal to zero, how many lines contain no point in Quadrant I ?

A. 0
B. 7
C.13
D.20
E. 35

SOLUTION :
Following are the types of lines that doesn’t pass through the Quadrant I
- Line having a negative slope and passing through the origin
- Line having a negative slope and a negative y-intercept
- Horizontal line, parallel to x-axis, having a negative y-intercept(Slope= Zero)

There are a total 35 lines, out of which 15 have positive slopes


(All the positive sloped line will pass through the 1st Quadrant )

So, (35-15)= 20 Lines have a negative slope or a zero slope.


1
Out of 15 lines , 3 rd i.e., 5 lines have positive y-intercept.
So, the remaining 10 lines with positive slope have negative y-intercept or pass through the
origin and they will pass through the Quadrant I

Now, Out of the 23 lines, remaining 23 - 10 = 13 lines will have y-intercept less than or equal to
zero. These lines will have a negative slope with negative y-intercept OR will pass through the
origin or will be parallel to the x axis with a negative y-intercept.

These 13 lines will not pass through Quadrant I.

ANSWER C
3. In the coordinate plane, Line H has a positive slope, and Line J never passes through the first
quadrant. Which of the following could be true ?

I. line J is perpendicular to line H


II. line J is parallel to line H
III. line J intersects line H on the x-axis

A. I only
B. II only
C. I and III only
D. II and III only
E. I, II, and III

SOLUTION :
Line H will always pass through quadrant I as it has a positive slope.

I. J is perpendicular to H..
Since J will have a negative slope and can be made in a way that it does not cross quadrant I.
It could be perpendicular to Line H.

II. J is parallel to H.
If line J is parallel to line H, then their slopes are equal. Since line H has a positive slope, line J
must also have a positive slope and will pass through the first quadrant. However, this
contradicts the information that line J never passes through the first quadrant.
Not possible.

III. Line J intersects H on the X axis.


Yes possible. J and H can cross on the negative x axis.

So, I and III are possible

ANSWER C
1
4. A line in the xy-plane passes through the origin and has a slope of 7
. Which of the following
points lies on the line ?

A. (0, 7)
B. (1, 7)
C. (7, 7)
D. (14, 2)
E. (7, 14)

SOLUTION:
1
The line passes through (0,0) and (x,y) and has a slope 7
(𝑦2−𝑦1) 1
Slope = (𝑥2−𝑥1)
= 7
(𝑦−0) 1
(𝑥−0)
= 7
𝑦 1
𝑥
= 7

Let us look at the options


𝑦 7
A. 𝑥
= 0
= Reject
𝑦 7
B. 𝑥
= 1
= Reject
𝑦 7
C. 𝑥
= 7
= Reject
𝑦 2 1
D. 𝑥
= 14
= 7 = Valid
𝑦 14
E. 𝑥
= 7
= Reject

ANSWER D
5. In the coordinate plane, Line J passes through the origin as well as points (a, 2) and (8, a).
Which of the following could be the slope of Line J ?

1
A. 4
1
B. 2
C. 1
D. 2
E. 4

SOLUTION:
The line passes through the points (0,0), (a,2) and (8,a)
Let us find the unknown variable a
For points (0,0) and (a,2)
(𝑦2−𝑦1) (2−0) 2
Slope = (𝑥2−𝑥1)
= (𝑎−0)
= 𝑎 …..(i)

For points (0,0) and (8,a)

(𝑦2−𝑦1) (𝑎−0) 𝑎
Slope = (𝑥2−𝑥1)
= (8−0) = 8
….(ii)

From (i) and (ii)


2 𝑎
𝑎
= 8

a2= 16
a= 4 or -4

2
From (i), Slope = 𝑎
1 1
Slope = 2
or − 2

ANSWER B
6. In the xy-plane, line n passes through the origin and has slope 4. If points (1, c) and (d, 2) are
𝑐
on line n , what is the value of 𝑑
?

A. 0.25
B. 0.5
C. 2
D. 4
E. 8

SOLUTION:

The line passes through the origin i.e., (0,0) and has a slope 4
Let us take the points one by one to find the unknown variables c and d

For (1,c) and (0,0)


Slope = 4
(𝑦2−𝑦1)
(𝑥2−𝑥1)
=4
(0−𝑐)
(0−1)
=4
c=4

For (d,2) and (0,0)


Slope = 4
(𝑦2−𝑦1)
(𝑥2−𝑥1)
=4
(0−2)
(0−𝑑)
= 4
2
𝑑
=4
1
d= 2

𝑐 4
So, 𝑑
= (1/2)
=8

ANSWER E
7. The y-intercepts, P and Q, of two perpendicular lines intersecting at the point A(6, 8) have a
1
sum of zero. What is the area of triangle APQ ? (Area of the triangle = 2
×Base× Height)

A. 45
B. 48
C. 54
D. 60
E. 72

SOLUTION :
Since the sum of y-intercepts is 0, let the coordinates of y-intercepts of P and Q be (0,c) and
(0,-c) respectively.
The line intersects at point A (6,8)
(𝑦2−𝑦1) (𝑐−8) (𝑐−8)
Slope of Line PA, m1= (𝑥2−𝑥1) = (0−6) = − 6
(𝑦2−𝑦1) (−𝑐−8) (𝑐+8)
Slope of Line QA, m2= (𝑥2−𝑥1) = (0−6)
= 6

We know that the product of slopes of two perpendicular lines is -1


m1×m2=-1
(𝑐−8) (𝑐+8)
− 6
× 6
= -1
c2-64 = 36
c2=100
c = 10 or -10
The Distance between the intercepts is the length of the base of the triangle APQ= 20 units
Height of the triangle= x coordinate of the intersecting point A i.e., 6 units
1 1
Area of the triangle APQ= 2
×Base× Height= 2
×20× 6= 60 sq units

ANSWER D
8. If each of the two lines p and q are parallel to line r , which of the following MUST be correct
?

I. Lines p, q and r lie in the same plane.


II. Lines p and q are parallel to one another.
III. Line p is the same as line q.

A. Only I
B. Only II
C. Only III
D. Only I and II
E. Only II and III

SOLUTION:
Each of the two lines p and q are parallel to line r

I. Lines p, q and r lie in the same plane. NOT NECESSARILY TRUE because p and r may be in one
plane and q may be in other plane

II. Lines p and q are parallel to line r , then p and q will be parallel to each other as well.
MUST BE TRUE

III. Line p is the same as line q. NOT NECESSARILY TRUE as we don’t have any such information
given in the question.

ANSWER B
9. If line m is perpendicular to line L and line L passes through points (6, 5) and (6, 8) , what is
the slope of line m ?

A. -1
B. 0
C. 1
D. the slope is undefined
E. the slope cannot be determined from the information given

SOLUTION :
line L passes through points (6, 5) and (6, 8)
(𝑦2−𝑦1) (8−5) 3
Slope of Line L = (𝑥2−𝑥1)
= (6−6)
=0
The slope of Line L is undefined which implies that Line L is a vertical line parallel to the x axis.
So, Line m will be parallel to the x axis and its slope will be 0

ANSWER B
10. Line K is tangent to the circle with center (0, 0) at (-3, y) such that its y-intercept is positive.
If the radius of the circle is 5, what is the slope of Line K ?

−4
A. 3
−3
B. 4
3
C. 4
1
D. 2
E. Cannot be determined

SOLUTION:

The distance between the center (0,0) and the point (-3,y) is 5
Let us find the unknown variable y
Equation of Circle : x2+ y2 = 52
(-3)2+y2=25
y2 = 16
y=4
The required point is (-3, 4)
Let us find the equation of the line joining the center of the circle and the point (-3,4)
(𝑦2−𝑦1) (4−0) 4
Slope = (𝑥2−𝑥1) = (−3−0) =− 3
The tangent at any point of a circle is perpendicular to the radius at the point of contact.
The product of Slopes of two perpendicular lines is -1.
So, m1 × m2= -1
4
− 3
× m2= -1
3
m2 = 4

ANSWER C
11. Which of the following is the equation of the line in the xy-plane that has slope 0 ?

I. x = 2
II. y = 3
III. x + y = 0

A. I only
B. II only
C. III only
D. I and II only
E. II and III only

SOLUTION:
Equation of the line is given by y = mx+c
Now, any line parallel to the x axis will have slope 0 i.e., m=0
The equation of such line will be of the form y = c

From the given choices only y = 3 will have slope 0.

ANSWER B
12.

In the figure above, lines k, m and p have slopes r, s and t respectively. Which of the following is
a correct ordering of these slopes ?

A. r < s < t
B. r < t < s
C. s < r < t
D. s < t < r
E. t < s < r

SOLUTION :
Line k is a horizontal line, so the slope r is 0.
Line m is going downwards, so the slope s is negative.
Line p is going upwards, so the slope t is positive.

So, s < r < t

ANSWER C
13. If line k passes through the points (48, 33) and (31, 22), what is the x-intercept of line k?

A. -2
B. -3
C. -1
D. 1
E. 2

SOLUTION:
(𝑦2−𝑦1) (22−33) 11
Slope of the Line k = (𝑥2−𝑥1)
= (31−48)
= 17
The equation of the line is given by (y-y1)= m(x-x1)
Equation of Line k:
11
(y-33) = 17 (x-48)

We can find the x-intercept by putting y = 0


11
-33 = 17 (x-48)
-51 = x-48
x = -3

ANSWER B
14. If a line passes through the point (3, 5) and has a positive slope, then which of the following
must be true about the y-intercept (c) of the line ?

I. c < 0
II. c = 0
III. c > 0

A. I only
B. II only
C. I & II only
D. I, II & III
E. None of the above

SOLUTION:
A line has a positive slope when it moves from the 3rd Quadrant to the 1st quadrant. The point
shown is (3,5)

We can see that the y-intercept can satisfy all of the three options.
All the three statements COULD be true.
Question asked MUST be true.
Hence, None of the above
ANSWER E
15. Line D passes through point (-2,5), and the product of its x-intercept and y-intercept is
positive. Which of the following points could be on line D?

A. (5,10)
B. (-3,2)
C. (-1,7)
D. (-1,2)
E. (-3,1)

SOLUTION:
Say a is the x-intercept and b is the y-intercept of Line D.
So, It passes through the points (a,0) and (0,b)

𝑏
The slope of this line will be − 𝑎

The product of x-intercept and y-intercept is positive, it implies that both the intercepts are
having the same sign i.e., either both are positive or both are negative.

𝑏
Which means that − 𝑎
is a negative number
The slope of Line D is negative.

From the answer choices, only (-1,2) gives us a negative slope


(5−2)
m= (−2−(−1))
= -3

ANSWER D
4
16. The equation of line n is y = 3
x - 100. What is the smallest possible distance in the xy-plane
from the point with coordinates (0, 0) to any point on line n?

A. 48
B. 50
C. 60
D. 75
E. 100

SOLUTION:
The shortest distance between any two points is the perpendicular distance.
Let us find the intercepts of the line n
Putting x = 0, y = -100
Putting y = 0, x = 75
So, the line n passes through the points (0,-100) and (75,0) and it forms a right angled triangle,
third coordinate being the origin
Let us plot the graph below (not drawn exactly as it should be):

x is the shortest distance from the point (0,0) to the line n.


The right angled triangle has a base of 75 units and height of 100 units.
So, Hypotenuse = 125 (Pythagoras theorem)
1 1
Now, 2
× x × 125 = 2
× 75 × 100 (Area of triangle)
x = 60

ANSWER C
17. In the xy-coordinate system, the distance between the point (0,0) and point P is
√40. Which of the following could be the coordinates of point P?

A. (4,7)
B. (4,10)
C. (5,6)
D. (6,2)
E. (20,20)

SOLUTION:
Let the coordinates of point P be (x,y)
2 2
Distance of the point (x, y) from the origin (0, 0) is 𝑥 + 𝑦
Let us check the answer choices and check which point gives us the distance √40
2 2
A. 4 + 7 = 65
2 2
B. 4 + 10 = 2 29
2 2
C. 5 + 6 = 61
2 2
D. 6 + 2 = 40
2 2
E. 20 + 20 = 20 2

ANSWER D
18. In the XY-coordinate system, line k passes through points (-5m, 0) and (0, 2m). Which of the
following is a possible equation of line k ?

5
A. y = − 2
x + 2m
2
B. y = 5
x - 5m
5
C. y = 2 x + 2m
2
D. y = 5 x + 2m
2
E. y = − 5 x - 5m

SOLUTION:
The line k passes through the points (-5m, 0) and (0, 2m)
(2𝑚−0) 2𝑚 2
Slope of the line k = (0−(−5𝑚))
= 5𝑚
= 5
The y-intercept (c) of the line k is 2m
The equation of the line is y = mx+c , where c is the y-intercept.
2
y = 5 x + 2m

ANSWER D
19. What is the equation of the line parallel to the line 2x - 7y = 20 and making an intercept of 7
on the x axis ?

A. 2x - 7y =7
B. x - y = 7
C. 2x - y = 7
D. x - 7y = 14
E. 2x -7y =14

SOLUTION:
2 20
Expressing the line 2x - 7y = 20 as y = 7 x - 7
Parallel lines have same slopes
The equation of the required line can be expressed as
2
y= 7
x+ c
At the x-intercept i.e., (7,0) y = 0
2
0= 7
×7+c
c = -2
2
Now, y = 7
x- 2
2x - 7y = 14

ANSWER E
20. What is the equation of the line that is the perpendicular bisector of the line segment
connecting points (4,–2) and (–3,5) in the xy plane ?

3
A. y = –x + 2
1
B. y = x + 2
3
C. y = 2
x–1
D. y = –x + 2
E. y = x + 1

SOLUTION:
Let us find the slope of the line segment connecting the points (4,–2) and (–3,5)
(5−(−2)) 7
Slope = (−3−4)
= −7
= -1
the product of slopes of two perpendicular lines is -1
So, m1 × m2= -1
-1 × m2= -1
m2= 1

The perpendicular bisector passes through the mid point of the two points (4,–2) and (–3,5)

𝑥1+𝑥2 𝑦1+𝑦2
The coordinates of the mid points is given by ( 2
, 2
)
4−3 −2+5 1 3
Midpoint= ( 2
, 2
)= ( 2 , 2 )

Equation of the line is given by y - y1= slope × (x-x1)


3 1
y- 2
= 1 (x - 2
)
2y - 3 = 2x-1
2y = 2x + 2
y = x+1

ANSWER E
21. Line k is in the rectangular coordinate system. If the x-intercept of k is -2, and the y-intercept
is 3, which of the following is an equation of line k ?

A. -3x + 2y = 6
B. 3x + 2y = -6
C. 3x - 2y = 6
D. 2x - 3y = 6
E. -2x - 3y = 6

SOLUTION:
The line k passes through (-2,0) and (0,3)
(3−0) 3
Slope, m = (0−(−2))
= 2

y-intercept is 3
The equation of the line is given by y = mx + c , where c is the y-intercept.
3
y=2x+3
2y = 3x+6
2y - 3x = 6

𝑥 𝑦
Alternately , equation of a line is also given by 𝑎
+ 𝑏
= 1 , where a is the x-intercept and b is
the y-intercept.

ANSWER A
22. The standard equation of a circle is (x−h)2 + (y−k)2 = r2 , where the point (h,k) is the center of
the circle and r represents the length of the radius. If a particular circle can be represented by
the equation x2 + y2 − 8x + 2y = −1, what is the radius of that circle ?

A. -1
B. 1
C. 2
D. 4
E. 8

SOLUTION :
Let us represent the given equation of the circle in the standard form
x2 + y2 − 8x + 2y = −1
x2 + y2 - 8x + 16 - 16 + 2y + 1 = 0
( Try rearranging with respect to the standard equation of circle )
(x - 4)2 + (y + 1)2 = 42
So, the center of the circle is (4,-1) and the radius is 4.

ANSWER D
23. The coordinates of points A and C are (0, -3) and (3, 3), respectively. If point B lies on line AC
between points A and C, and if AB = 2BC, which of the following represents the coordinates of
point B ?

A. (1, -√5)
B. (1, -1)
C. (2, 1)
D. (1.5, 0)
E. (√5, √5)

SOLUTION :
We know that SECTION FORMULA :
If a point R(x,y) divides the line segment PQ (coordinates of P and Q : (𝑥1, 𝑦1) and (𝑥2 , 𝑦2) ) in
the ratio m : n , then

Coordinates of point R(x,y)


𝑚𝑥2 + 𝑛𝑥1
x= 𝑚+𝑛

𝑚𝑦2 + 𝑛𝑦1
y= 𝑚+𝑛

Here , B divides AC in the ratio 2 : 1. So m = 2 , n = 1


2×3 + 3×0 2×3 − 1×3
Coordinates of point B will be ( 3
, 3
) = (2,1)

ANSWER C
24.

Which of the following could be the equation of line m ?

A. 6y + 6x = 7
B. 3y = -4x - 3
C. 5y + 10 = -4x
D. y = 2
E. x = -2

SOLUTION:
y-intercept of line m is positive.

Line depicted cuts both the x and y axis at positive sides. Thus both x intercept and y intercept
of given lines are positive.
Options B and C both have a negative y-intercept so they are eliminated.
Options D and E don’t have negative slopes as line m.
Hence, Option A is the correct choice.

ANSWER A
25. Which of the following points (x, y) is NOT on the graph of y < 2x ?

A. (-3, -7)
B. (3, 3)
C. (2, -9)
D. (2, 2)
E. (2, 5)

SOLUTION:
For a point to lie on a graph, it should satisfy the inequality given.Here, we need to find (x,y)
coordinate, which won't satisfy the equation. Substitute the coordinates in the equation one by
one.

Option 1: (-3, -7). y = -7 and 2x = -6. -7 < -6, True.

Option 2: (3, 3). y = 3 and 2x = 6. 3 < 6, True.

Option 3: (2, -9). y = -9 and 2x = 4. -9 < 4, True.

Option 4: (2, 2). y = 2 and 2x = 4. 2 < 4, True.

Option 5: (2, 5). y = 5 and 2x = 4. 5 < 4, False.

ANSWER E
26. If a circle circumscribes a triangle bounded by the x-axis, y-axis and the line 3y - 4x = -24,
what is the radius of the circle ?

A. 2.5
B. 3
C. 4
D. 5
E. 10

SOLUTION:
The x and y intercepts of 3y − 4x = −24 are 6 at y = 0 and −8 at x = 0 respectively.
So, the lengths of the legs of the triangle will be 6 and 8.

The triangle we get will be a right triangle (x and y axis are at 90°). Now, a right triangle
inscribed in a circle must have its hypotenuse as the diameter of the circle. Angle in a
semi - circle is 90 degrees.

2 2
The hypotenuse, is thus 6 + 8 = 10
Radius is half i.e. 5

ANSWER D
27. A parabola has one x-intercept at (– 4, 0). If the vertex is at (2, 5), find the other x-intercept.

A. 5
B. 6
C. 7
D. 8
E. 9

SOLUTION:
The line of symmetry of a parabola always passes through the vertex, so the equation of the line
of symmetry is the vertical line x = 2. The point (– 4, 0) is six units i.e. 2-(-4)=6 to the left of the
symmetry line, so the other point(root) should be six units to right i.e. 2+6 = 8, at (8, 0).

ANSWER D
28. On the x-y coordinate plane there is a parabola, y = x(6-x). Which of the following points is
in the region that surrounds this parabola and x-axis ?

I. (1, 4)
II. (3, 6)
III. (5, 4)

A. I only
B. II only
C. III only
D. I & II
E. I, II & III

SOLUTION:

y= x(6-x)
y= 6x-x2
Since the coefficient of the x2 is negative, the parabola will be facing downwards.
The parabola will be cutting the x-axis at x = 0 and x = 6
Let us put x =1,3 and 5 in the above equation and check the corresponding y coordinates
For x = 1, y = 1(6-1) = 5
For x = 3, y = 3(6-3) = 9
For x = 5, y = 5(6-5) = 5

(1,5), (3,9) and (5,5) will lie on the parabola


So, the points (1,4), (3,6) and (5,4) will lie below the points (1,5), (3,9) and (5,5) respectively.
Hence, they will be enclosed in the parabola and the x-axis.

ANSWER E
29.

A graph of the function g(x) is shown above. g(x) is defined by which of the following equations?

A. g(x) = (x+1)2 − 2
B. g(x) = (x−1)2 − 2
C. g(x) = (x+1)2 + 2
D. g(x) = (x−1)2 + 2
E. g(x) = (x−2)2 + 1

SOLUTION :
2
y = 𝑥 will start through the origin as its vertex.

2
y = (𝑥 − 1) will start through the point (1,0) as its vertex.
Shift this graph by 2 units down to get the graph shown.
2
y = (𝑥 − 1) - 2 will start through the point (1,-2) as its vertex.

g(x) = (x−1)2 − 2

ANSWER B
30. Which of the following points are on the graphs of both the equation y = x + 2 and the
equation y = x2 in the xy-plane?

I. (-1,1)
II. (0, 0)
III. (2,4)

A. I only
B. II only
C. III only
D. I and III only
E. I, II and III

SOLUTION:
y = x+2

y = x2

To find the common points of these equations, let us equate them:

x + 2 = x2
x2 - x − 2 = 0
x2 - 2x + x − 2 = 0
x (x−2) + 1(x−2) = 0
(x−2)(x+1) = 0
x = 2 or x = −1

Substitute x = -1 and 2 in any one of the given two equations to get y as 1 and 4 respectively.

So the common points are (-1 , 1) and (2 , 4)

ANSWER D
31.

In the rectangular coordinate system above, the area of triangular region PQR is
1
( Area of Triangle = 2
×base×height )

A. 12.5
B. 14
C. 10√2
D. 16
E. 25

SOLUTION:
The triangle is enclosed within a rectangle with dimensions 7 x 4

Area of the △PQR


= Area of rectangle - Area of triangle C - Area of triangle B - Area of triangle A
1 1 1
= (7 × 4) − ( 2 × 3 × 4) − ( 2 × 4 × 3) − ( 2 × 1 × 7)
= 28 − 6 − 6 − 3.5
= 12.5
Alternate Approach:
Let us find the slope of QP and PR
(3−0) 3
Slope of QP, m1= (0−4)
=− 4
(4−0) 4
Slope of PR, m2= (7−4)
=3
The product of the slopes of line QP and PR is equal to -1
So, the lines are perpendicular to each other.

Let us find the length of QP and PR as they represent the base and height of the triangle PQR
2 2
QP = (4 − 0) + (3 − 0) = 16 + 9= 25 = 5
2 2
PR = (7 − 4) + (4 − 0) = 9 + 16= 25 = 5

1 1 25
Area of a right angled triangle is given by 2
×Base×Height = 2
×5×5 = 2
= 12.5

ANSWER A
32. What is the area of the region bounded by the lines y = |x| - 2 and y = 2 - |x| ?

A. 2
B. 6
C. 8
D. 12
E. 16

SOLUTION:
The region bounded by lines y = |x| - 2 and y = 2 - |x| is a square ABCD.

Area of ABCD = 2 × Area(△ABD)


1 1
Area of △ABD = 2
× BD × AO = 2
×4×2=4
Area of ABCD = 2 × 4 = 8

ANSWER C
𝑥
33. What is the area of the triangle formed by the intersection of lines y = 2x – 2, y = − 2
+8,
and y = 0 ?
A. 20
B. 30
C. 40
D. 45
E. 60

SOLUTION:

𝑥
Intersection of two lines: y = 2x - 2 and y = - 2
+8
𝑥
2x - 2 = - 2
+8
4x - 4 = -x + 16
5x = 20
x=4

Substituting x in any of the above equations to solve for y


y = 2x - 2
y=8-2
y=6 ....height of the triangle

One vertex is (4,6)


−𝑥
The two lines y = 2x - 2 and y = 2
+ 8 will pass through (1,0) and (16,0) respectively.
So, Length of base = 16 - 1 = 15 and height = 6.
1
Area of triangle = 2
× 15 × 6 = 15 × 3 = 45

ANSWER D
34.

In the rectangular coordinate system above, triangle ABC has an area of 27 square units. Vertex
A is at point (0, 9), vertex B is at point (0, y), and vertex C is at point (9, y). What is the value of y
?
A. 1
B. 2
C. 3
D. 4
E. 6

SOLUTION:
Side BC is the base of the triangle ABC
Subtracting x-coordinates of vertex B and vertex C will give Base = 9
1
Area of triangle ABC = 2
× 9 × height = 27
height = 6

Point A has coordinates as (0,9)


y coordinate of A - y coordinate of B = Height of triangle ABC
9-y=6
y=9-6=3

ANSWER C
35.

Point P is on the line y = 2x+a. The x-coordinate of P is 4. The area of the shaded quadrilateral is
24. What is the value of a ?
1
[ Area of Trapezium = 2
× (Sum of parallel sides) × height ]

A. 1
4
B. 3
C. 2
7
D. 6
E. 3

SOLUTION:
Substitute value of x i.e. 4 in vertex P, we have the point P(4,a+8).

1 1
The area of the trapezium = 2
× (Sum of parallel sides) × height = ( 2 ) × (a + a + 8) × 4
=> 24 = 2(2a + 8)
=> 2a + 8 = 12
=> a = 2

ANSWER C
36.

In the rectangular coordinate system above, the line y = x is the perpendicular bisector of
segment AB (not shown), and the x-axis is the perpendicular bisector of segment BC (not
shown). If the coordinates of point A are (2,3), what are the coordinates of point C ?

A. (-3,-2)
B. (-3,2)
C. (2,-3)
D. (3,-2)
E. (2,3)

SOLUTION:

D will be the midpoint of AB. So it divides AB in the ratio 1 : 1 .


As AB is perpendicular to y = x , its slope will be -1.
Equation of line AB passing through (2,3) having a slope -1 will be x + y = 5.
Solving y = x and x + y = 5 will give the coordinates of D (2.5, 2.5).
Hence, Coordinates of B will be (3,2) (Apply section formula to find this)
As x-axis is the perpendicular bisector of BC , the coordinates of C will be (3,-2)

ANSWER D
18
(1+2 )
37. Which of the following is closest to 4 10 ?
(2 +2 )

A. 25
B. 26
C. 27
D. 28
E. 29

SOLUTION :
18
(1+2 )
= 4 10
(2 +2 )

18
(2 )
= 4 6
2 (1 + 2 )

18
(2 )
= 4 6
2 (2 )

18
(2 )
= 10
(2 )

=>28

ANSWER D
1
38. Which of the following is closest to 1.001
?

A. 0.997
B. 0.998
C. 0.999
D. 1.000
E. 1.001

SOLUTION :
1
1.001
1000
= 1001
(1001−1)
= 1001
1
= 1 - 1001
= ~ 1 - 0.001
= ~ 0.999

ANSWER C
39. AB + CD = AAA , where AB and CD are two-digit numbers and AAA is a three digit number ;
A, B, C, and D are distinct positive integers. What is the value of C ?

A. 1
B. 3
C. 7
D. 9
E. Cannot be determined

SOLUTION :
AB and CD are two-digit integers, their sum can give only one three digit integer : 111
(The sum of any two 2-digit numbers cannot be more than 99 + 99 = 198 )

So, A = 1 and we have 1B + CD = 111

Now, C cannot be less than 9, because no two-digit integer with the first digit 1 (1B) can be
added to a two-digit integer less than 90 to have the sum 111.
Hence C = 9.

ANSWER D
40.

In the correctly worked addition problem shown, P, Q, R, and S are digits. If Q = 2P, which of the
following could be the value of S ?

A. 3
B. 2
C. 1
D. -1
E. -2

SOLUTION :
Units digit: 5 + R results in 4 => The only possible scenario is that R = 9 and 5+R = 14, of which 1
gets carry forward in the tens position.
For the tens place: 1 + P + Q = S
However, in the hundreds place: 3 + 4 gives 8 => there was a carry of 1
Hence : => In the tens place: 1 + P + Q = S + 10
=> P + Q = S + 9
=> P + 2P = S + 9
=> 3P = S + 9
=> S must be a multiple of 3

=> S = 0 or 3

ANSWER A
PnC, Probability Extra Question Solutions

PERMUTATIONS AND COMBINATIONS

1. There’s a party in town and Mike can’t decide what to wear. However, he has decided to
follow the following:
a. He will wear tie and shoes
b. He may or may not wear socks.
Mike has 3 ties, 4 pairs of shoes and 5 pairs of socks. In how many ways can Mike dress for the
party?

A. 12
B. 60
C. 72
D. 144
E. 288

SOLUTION:
Mike can dress for the the party in two ways

Case 1 : Mike Wears tie and shoes and no socks


There are 3×4×1 = 12 ways
Case 2 : Mike wears tie and shoes and socks
There are 3×4×5 = 60 ways

Total number of ways Mike could dress for the party = 12 + 60 = 72 ways

ANSWER C
2. In her class of 10 girls and 8 boys, the teacher has to select either a girl OR a boy. In how
many ways can she make her selection?
A. 12
B. 18
C. 40
D. 80
E. 160

SOLUTION :
Here the teacher has to choose either a girl OR a boy (Only 1 student)

A Boy can be selected in 8 ways


A Girl can be selected in 10 ways

By fundamental principle of addition(RULE OF OR), the teacher can make the selection of a
student in (10+8) =18 ways.

ANSWER B
3. How many 3 digits even numbers are possible using the digits 0,3,1,6,7,9 if repetition of digits
is allowed ?

A. 20
B. 30
C. 40
D. 60
E. 120

SOLUTION:
let abc is three digit number:
a can be filled in 5 ways(3,1,6,7,9)
b can be filled in 6 ways (0,3,1,6,7,9).
As the three digit number is an even number , c can be filled in 2 ways (0 and 6).
So, Total ways = 5×6×2 = 60 ways

ANSWER D
4. Of the three-digit integers greater than 660, how many have two digits that are equal to each
other and the remaining digit different from the other two?

A. 47
B. 60
C. 92
D. 95
E. 96

SOLUTION :
Three digit number can have only following 3 patterns:

A. all digits are distinct;


B. all three digits are alike;
C. Two digits are alike and the third digit is different.
We need to calculate C.
C = Total - A - B

Total numbers from 661 to 999 = 339 (3-digit numbers greater than 660) ;

A. all digits are distinct = 3×8 + 3×9×8 = 240.


If the first digit is 6 then the second digit can take only 3 values (7, 8, or 9) and the third digit
can take 8 values. If first digit is 7, 8, or 9 (3 values) then second and third digits can take 9 and 8
values respectively;

B. all three digits are alike = 4 (666, 777, 888, 999).

So, C = Total - A - B = 339 - 240 - 4 = 95.

ANSWER D
5. A teacher wishes to seat 5 boys and 3 girls in a row at a bench. In how many ways can the
teacher seat the boys and the girls so that all the girls do not sit together?

A. 720
B. 2160
C. 4032
D. 18000
E. 36000

SOLUTION :

Let the 5 Boys be B1, B2, B3, B4, B5 and the 3 Girls be G1, G2, G3
Number of ways in which all the 3 girls do not sit together = Total no of ways in which 5 boys
and 3 girls can sit - No. of ways in which all the three girls sit together.

Total no. of ways in which 5 boys and 3 girls can sit = 8! = 40320

Total no. of ways in which all the 3 girls sit together


Consider three girls as one entity
Now we have 5 boys and one girls group to sit i.e., B1, B2, B3, B4, B5, {G1G2G3}
Ways of arranging 6 people= 6! = 720
The three girls can arrange themselves in 3! ways= 6 ways
Total no. of ways in which three girls sit together= 720×6 = 4320

So, The Number of ways in which all the 3 girls do not sit together= 40320 - 4320 = 36000

ANSWER E
6. In how many ways 4 boys and 3 girls can be seated in a row so that they are alternate ?

A. 12
B. 24
C. 72
D. 96
E. 144

SOLUTION :

Let the 4 boys be denoted as B1 B2 B3 B4 and the 3 girls be denoted as G1 G2 G3


We need to arrange them alternately in a row
One sample arrangement is B1, G1, B2, G2 ,B3, G3 ,B4
So, The boys can arrange themselves in 4! = 24 ways
The girls can arrange themselves in 3! = 6 ways
Total ways to arrange the Boys and Girls in a row such that they are alternate is 24 × 6 = 144
ways.

ANSWER E
7. A certain company places a six-symbol code on each of their products. The first two symbols
are one of the letters A–E and the last four symbols are digits. If repeats are allowed on both
letters and numbers, how many such codes are possible?

A. 250000
B. 350000
C. 450000
D. 550000
E. 600000

SOLUTION :
Let the six-symbol code be represented by L1L2D1D2D3D4
We have 5 letters A,B,C,D, and E
L1 and L2 can be filled in 5×5=25 ways
For each digit we have 10 ways to fill them.
Number of ways to fill 4 digits = 104 = 10000 ways
Total number possible codes = 25×10000 = 250000 ways

ANSWER A
8. If all the letters of the word RATE are taken and arranged in alphabetical order as in a
dictionary , then what is the rank of the word TEAR ?

A. 20
B. 23
C. 22
D. 21
E. 19

SOLUTION :

When arranged in an alphabetical order, the words starting with A will come first followed by
words starting with E, R and T
Number of words starting with A: 1×3×2×1= 6 words
Number of words starting with E: 1×3×2×1= 6 words
Number of words starting with R: 1×3×2×1= 6 words
Now, Let us arrange the words starting with T in alphabetical order: TAER, TARE, TEAR
So, TEAR will be the third word
RANK of the word TEAR= 6 + 6 + 6 + 3 = 21

ALTERNATE SOLUTION :
Total arrangements of the word RATE = 4! = 24
Words coming after TEAR : TERA, TRAE, TREA
So, TEAR will be ranked 24 - 3 = 21

ANSWER D
9. In how many ways can 6 people be seated around a circular table if 2 particular people are
always separated ? (2 seating arrangements are considered different only when the positions of
the people are different relative to each other.)

A. 24
B. 48
C. 72
D. 96
E. 120

SOLUTION:

Total number of ways of arranging 6 people in a circular table: (n−1)! = (6−1)! = 120 ways
Number of ways such that 2 particular people are always together: (5-1)!×2! = 4!×2! = 48 ways
(Considering those 2 people as a single unit)
Number of ways such that 2 particular people are always separated => Total number of ways -
Number of ways such that 2 particular people are always together
=> 120 - 48
=> 72 ways

ANSWER C
10. In how many ways can a party of 4 men and 4 women be seated at a circular table so that
no two men are adjacent? (Two sitting arrangements are considered different only when the
positions of the people are different relative to each other.)

A. 24
B. 48
C. 72
D. 144
E. 288

SOLUTION :

The 4 women can be seated at the circular table such that there is a vacant seat between every
pair of women in (4-1)! = 3! Ways.
Now 4 Alternate vacant seats can be occupied by 4 men in 4! Ways.

Hence the required number of seating arrangements = 3! × 4! = 144

ANSWER D
11. If a committee of 3 men and 3 women is to be selected from a group of 7 men and 7
women, how many different committees are possible ?

A. 36
B. 48
C. 242
D. 352
7!
E. ( 3! )2

SOLUTION :
Number of ways of selecting 3 men and 3 women form a group of 7 men and 7 women
= 7C3×7C3
= 35 ×35
=352

ANSWER D
12. A committee of four is to be chosen from seven employees for a special project at ACME
Corporation. Two of the seven employees are unwilling to work with each other. How many
committees are possible if the two employees do not work together ?

A. 15
B. 20
C. 25
D. 35
E. 50

SOLUTION :

Let A and B be the employees who do not want to work with each other.
Total number of committees that can be formed = 7C4 = 35
Total number of committees such that A and B work together = 5C2 = 10
Total Committees that can be formed such that A and B do not work together = 35-10 = 25

ANSWER C
13. A committee needs to be formed of 3 women and 2 men from a group of 6 women and 5
men. Among the chosen members, one needs to be selected for the post of the President, and
one for the post of Secretary. In how many ways can the President and Secretary be chosen
from this committee formed ?

A. 200
B. 400
C. 600
D. 800
E. 4000

SOLUTION :
We can select 3 women from a group of 6 women in 6C3 ways and select 2 Men from a group of
5 Men in 5C2 ways.

Total number of ways= 6C3× 5C2 ways

Among the total selected 5, President can be selected in 5C1 ways and Secretary can be selected
in 4C1 ways
So, Total ways is given by 6C3× 5C2 × 5C1 × 4C1= 4000 ways

ANSWER E
14. What is the total number of ways in which Kosha can distribute 9 distinct gifts among her 8
distinct girlfriends such that each of them gets at least one gift ?

A. 2592*8!
B. 144*8!
C. 72*8!
D. 96*7!
E. 72*7!

SOLUTION :
Distribute 8 distinct gifts to 8 distinct girlfriends.

Total ways of Selecting 8 gifts from 9 distinct gifts = 9C8 = 9 ways.


We can distribute the 8 gifts to 8 girlfriends in 8! ways.
Number of ways to distribute 8 distinct gifts one each among his 8 distinct girlfriends = 9×8!

The remaining one gift can be given in 8 different ways, as there are 8 girlfriends.

Total number of ways to distribute 9 distinct gifts among 8 distinct girlfriends = 9 ×8! × 8 = 72 ×
8! ways [each gets at least one gift]

ANSWER C
15. The number of ways in which 5 different books can be distributed among 10 people if each
person can get at most one book is

A. 252
5
B. 10
10
C. 5
D. 10C5 * 5!
E. 10C5 * 10!

SOLUTION :
Each person can get at most one book.
i.e. 5 books will be given to 5 different people out of 10.

5 people out of 10 who get the book can be chosen in 10C5 ways = 252

But we need to distribute the book among them which can happen in 5! ways
So total outcomes = 10C5∗5!

ANSWER D
16. In how many ways can 9 identical balls be distributed among four baskets such that each
basket gets at least one ball ?

A. 35
B. 56
C. 63
D. 70
E. 126

SOLUTION :
There are 9 Identical balls and 4 baskets.
Number of ways to distribute 9 identical balls such that each basket gets at least 1 ball =
(n-1)C(r-1)= (9-1)C(4-1) = 8C3 = 8!/(5!3!) = 56 ways

ANSWER B
17. How many triangles can be inscribed in the heptagon pictured, where the three vertices of
the triangle are also vertices of the heptagon ?

A. 9
B. 21
C. 35
D. 140
E. 210

SOLUTION :
No 3 vertices in given heptagon are collinear
So, The number of triangles that can be formed = 7C3 = 7!/(3!4!) = 35

ANSWER C
18. If 4 points are indicated on a line and 5 points are indicated on another line that is parallel to
the first line, how many triangles can be formed whose vertices are among the 9 points ?

A. 20
B. 30
C. 40
D. 70
E. 90

SOLUTION :
Triangles can formed in two possible ways

(i) Two vertices on the line with 4 points and the third vertex will be on the line with 5 points
Total number of triangles : 4C2×5C1 = 30

(ii) Two vertices on the line with 5 points and the third vertex will be on the line with 4 points
Total number of triangles : 5C2×4C1 = 40

Total = 30 + 40 = 70

ANSWER D
19. What is the number of ways in which a person can move from point A to B walking along the
grids but not moving upwards or left of the grid or even retracing any grid ?

A. 45
B. 210
C. 35
D. 70
E. 25

SOLUTION:
We need to move 3 blocks Down and 4 blocks Right in order to reach B from A.

Total ways of arranging "R R R R D D D" = 7!/(4!3!) = 35

ANSWER C
20. What is the number of the shortest routes from X to Y through Z ?

A. 8
B. 12
C. 16
D. 18
E. 20

SOLUTION :
Let us find the number of routes to travel from X to Z and Z to Y and then multiply them to find
the final answer

X to Z
The route is a combination of 4 blocks i.e., 2 Up and 2 Right
Total number of routes= 4!/(2!2!) = 6 routes

Z to Y
The route is a combination of 2 blocks i.e., 1 Up and 1 Right
Total number of routes= 2!/(1!1!)= 2 routes

Number of shortest routes from X to Y through Z = 6×2 = 12

ANSWER B
21. Halle, Julia and Drew have 5 donuts to share. If one of them can be given any number of
donuts from 0 to 5, in how many different ways can the donuts be distributed ?

A. 19
B. 20
C. 21
D. 23
E. 37

SOLUTION :
We have 5 donuts, and that needs to be divided amongst 3 people
Using Partition method:
Let the donuts be denoted by D and the partition be represented by |
We have: DD|DD|D
We have 7 objects out of which 5 are identical and the other two bars are identical as well.
7!
So, Total number of ways: 5!2!
= 21

ANSWER C
22. Find the number of positive integer solutions to a + b + c = 10, where none of a, b or c is 0.

A. 36
B. 72
C. 144
D. 288
E. 576

SOLUTION:
Here none of a,b or c is 0
So, a = x+1 , b = y+1 , c = z+1
So, a+ b + c = 10
=> (x+1) + (y+1) + (z+1) = 10
x+y+z=7

Using partition method:


**|**|***
It is same as arranging 9 objects out of which 7 are same and the other 2 are also same
9!
So, we have : 7!2!
= 36

ANSWER A
23. In how many ways can 7 identical balls be placed into four boxes P,Q,R,S such that the two
boxes P and Q have at least one ball each ?

A.84
B.70
C.120
D.56
E.54

SOLUTION :
P+Q+R+S=7
Now, P and Q must have at least 1 ball
So, (p+1) + (q+1) + R + S = 7
p+q+R+S=5

Using partition method:


The balls can be represented by * and the partitions can be represented by |
*|*|**|*
So, we have 8 objects out of which 5 *’s are identical and 3 |’s are identical
8!
Total number of ways = 5!3!
= 56 ways

ANSWER D
PROBABILITY

1. If John throws a coin until a series of three consecutive heads or three consecutive tails
appears, what is the probability that the game will end on the fourth throw ?

1
A. 16
1
B. 8
3
C. 16
1
D. 4
3
E. 8

SOLUTION:
1
Probability of getting Heads = 2
1
Probability of getting Tails = 2

There are two possibility


(i) HTTT
1 1
Probability= ( 2 )4 = 16

(ii) THHH
1 1
Probability= ( 2 )4 = 16

1 1 2 1
Required probability = 16
+ 16
= 16
= 8

ANSWER B
2. A gambler rolls three fair six-sided dice. What is the probability that two of the dice show the
same number, but the third shows a different number?

1
A. 8
5
B. 18
1
C. 3
5
D. 12
5
E. 6

SOLUTION:

Total number of outcomes = 6×6×6=216


Now, for two dice showing the same number and the third dice showing different number is
represented by XXY
Number of ways of selecting X = 6C1 = 6
Number of ways of selecting Y = 5C1 = 5

3!
Number of ways of arranging XXY = 2!1!
= 3 ways
Favourable outcomes = 6×5×3 = 90
90 5
Probability = 216
= 12

ANSWER D
3. If two fair, six-sided dice are rolled, what is the probability that the sum of the numbers will
be 5 ?

1
A. 36
1
B. 18
1
C. 9
1
D. 6
1
E. 4

SOLUTION :
There are a total 6×6 = 36 possibilities
We can get 5, if we get the outcomes: (2,3), (3,2), (1,4) and (4,1)
So, There are a total 4 favourable cases
4 1
P(sum of numbers is 5) = 36
= 9

ANSWER C
4. A bag contains 5 red balls and 4 white balls. Three balls are drawn without replacement.
What is the probability of drawing at least 2 red balls?

5
A. 42
5
B. 18
17
C. 42
25
D. 42
13
E. 18

SOLUTION :

There are a total of 9 balls.


We need to to draw at least 2 red balls i.e., P(2 red balls) + P(3 red balls)

(i) P(2 Red balls). Let's assume we are drawing in the RRW order.

5 4 4 10
P(first red, second red and 3rd white) = 9
× 8 × 7 = 63

3!
Now, RRW can be arranged in 2!1!
= 3 ways

10 10
Therefore, Probability of drawing 2 Red balls = 63
×3 = 21

5 4 3 5
(ii) P(3 Red Balls) = 9
× 8 × 7 = 42

10 5 25
Therefore, the required probability = 21
+ 42
= 42

ANSWER D
5. A bag contains 3 red, 4 black and 2 white balls. What is the probability of drawing a red and a
white ball in two successive draws, each ball NOT being put back after it is drawn ?

2
A. 27
1
B. 6
1
C. 3
4
D. 27
2
E. 9

SOLUTION :

The favourable outcomes are RW and WR


There are a total 9 balls
3 2
P(R) = 9
, P(W)= 8
3 2 1
Required probability = P(RW) + P(WR) = 2× 9 × 8 = 6

ANSWER B
6. In a certain game, you pick a card from a standard deck of 52 cards. If the card is a heart, you
win. If the card is not a heart, the person replaces the card to the deck, reshuffles, and draws
again. The person keeps repeating that process until he picks a heart, and the point is to
measure how many draws it takes before the person picks a heart and wins. What is the
probability that there will be at least three draws involved in a win, i.e. someone picking her
first heart on the third draw or later?

1
A. 2
9
B. 16
11
C. 16
13
D. 16
15
E. 16

SOLUTION :
In a deck of 52 cards, there are 13 hearts
13 1
Probability of picking a heart on any draw = 52
= 4
Probability that there will be at least three draws involved in a win =
1 3 1 9
1 - (person wins in the 1st draw + person wins in the 2nd draw ) = 1 - ( 4
+ 4
×4 )= 16

ANSWER B
7. From a pack of 52 cards, two are drawn one by one without replacement. What is the
probability that both of them are jacks ?

1
A. 221
11
B. 850
25
C. 204
5
D. 24
5
E. 21

SOLUTION :
In a deck of 52 cards, there are 4 jacks
4
The probability that first card is Jack = 52
Now we are left with 51 cards in which there are 3 Jacks
3
After the first card is removed, The probability that second card is Jack = 51
4 3 1
Probability that both the cards are Jacks= 52
× 51 = 221

ANSWER A
8. If events E and F are independent and each has a probability p of occurring, which of the
following represents the probability that E will occur but F will not occur?

2
A. 𝑝
2
B. 1 - 𝑝
2
C. 𝑝 - 1
2
D. 𝑝 - p
2
E. p - 𝑝

SOLUTION:
Event E
Probability of Event E occurring = p
Probability of Event E not occuring = (1-p)

Event F

Probability of Event F occurring = p


Probability of Event F not occuring = (1-p)

Required Probability = P(Event E occurring) × P(Event F not occurring)


= p × (1-p)
= p−p2

ANSWER E
9. The probability of event p occurring is 0.4. The probability of event q occurring is 0.25. If
events p and q are independent, what is the probability that at most 1 of the events occur ?

A. 0.1
B. 0.35
C. 0.325
D. 0.90
E. 1.15

SOLUTION:
Probability of Event p occurring = 0.4
Probability of Event q occurring = 0.25
probability that at most 1 of the events occur =
P(p occurs and q doesn’t) + P(q occurs and p doesn’t) + P(none occurs)
=> (0.4)(0.75) + (0.25)(0.6) + (0.6)(0.75) => 0.3 + 0.15 + 0.45 => 0.90

ANSWER D
10. If a certain coin is flipped, it has probability 0.5 of landing on heads and probability 0.5 of
landing on tails. If the coin is flipped 4 times, what is the probability that it will land on tails at
least once?

3
A. 8
1
B. 4
1
C. 2
5
D. 16
15
E. 16

SOLUTION:
1
Probability of getting Head = 2
1
Probability of getting Tails = 2

Now, when the coin is tossed 4 times, the probability of landing tails at least once is given by : 1
- P (getting NO TAILS)
1 4 1 15
P(at least 1 tail) = 1- ( 2
) = 1- 16
= 16

ANSWER E
11. When tossed, a certain coin has equal probability of landing on either side. If the coin is
tossed 3 times, what is the probability that it will land on the same side each time?

1
A. 8
1
B. 4
1
C. 3
3
D. 8
1
E. 2

SOLUTION:
The Favourable outcome is if we'll have either three tails (TTT) or three heads (HHH):
1 1
P(HHH)= ( 2 )3= 8
1 1
P(TTT)= ( 2 )3= 8
1 1 1
Required Probability = 8
+ 8
= 4

ANSWER B
12. In a class, 60% of students play soccer or basketball. If 10% play both sports and 60% do not
play soccer, what is the probability that a student chosen at random from the class plays neither
soccer nor basketball?

A. 0.3
B. 0.4
C. 0.5
D. 0.6
E. 0.7

SOLUTION :
P(Soccer or Basketball)= 0.6
Now, P(Soccer or Basketball)+ P(neither Soccer or Basketball) = 1
0.6 + P(neither Soccer or Basketball) = 1
P(neither Soccer or Basketball) = 1 - 0.6
P(neither Soccer or Basketball)= 0.4

ANSWER B
13. A fair die is rolled once and a fair coin is flipped once. What is the probability that either the
die will land on 3 or that the coin will land on heads?

1
A. 2
5
B. 12
7
C. 12
2
D. 3
3
E. 4

SOLUTION :
The events are independent.
So, P(3 OR heads) = P(3) + P(heads) - P(3 AND heads)
1
P(heads) = 2
1
P(3) = 6
1 1 1
P(3 𝑎𝑛𝑑 heads) = 6
×2 = 12

1 1 1 7
So,P(3 OR heads) = 6
+ 2
- 12
= 12

ANSWER C
14. The probability of P hitting the target is 0.8 and that of Q hitting the target is 0.9 . If both of
them fire independently in a shooting competition, what is the probability that the target will
get hit ?

A. 0.72
B. 0.98
C. 0.7
D. 0.8
E. 0.28

SOLUTION :
Probability of P hitting the target = 0.8
Probability of P not hitting the target= 1-0.8 = 0.2

Probability of Q hitting the target = 0.9


Probability of Q not hitting the target = 1-0.9 = 0.1

Now, Probability that target will get hit = 1- (probability that none of them hit the target)

P(Target will get hit) = 1 - (0.2×0.1)


P(Target will get hit)= 0.98

ANSWER B
15. Three snipers shoot a certain target. Their probabilities of hitting the target are 0.9, 0.7, and
0.5 respectively. What is the probability that exactly one sniper hit the target ?

A. 0.015
B. 0.185
C. 0.485
D. 0.515
E. 0.985

SOLUTION :
Let the three snippers be A, B and C.
P(A) = 0.9
P(B) = 0.7
P(C) = 0.5

The probability that exactly one sniper hit is given by


Only A HITS : 0.9 × 0.3 × 0.5 = 0.135
ONLY B HITS : 0.1 × 0.7 × 0.5 = 0.035
ONLY C HITS : 0.1 × 0.3 × 0.5 = 0.015

Probability = Only A HITS + Only B HITS + Only C HITS = 0.135 + 0.035 + 0.015 = 0.185

ANSWER B
16. The probability of rain is 1/2 for every day next week. What is the chance that it rains on at
least one day during the workweek (Monday through Friday)?

1
A. 2
31
B. 32
63
C. 64
127
D. 128
5
E. 2

SOLUTION:
Probability of an Event Happening + Probability of that Event Not Happening = 1

Probability of Rain = 1 /2
Probability of No Rain = 1- 1/2 = 1/2
P(at least 1 day) + P(none) = 1
=> P(at least 1 day) = 1 - P(none)
1 1 1 1 1 1
For the five day period the Probability of No Rain = ( 2 )*( 2 )*( 2 )*( 2 )*( 2 ) = 32
1 31
=> P(at least 1 day) = 1 - 32
= 32

ANSWER B
17. At an archery tournament, eight of the twenty competitors were left-handed and the
remainder were right-handed. Due to the weather conditions it is more difficult for left-handers
to hit the target. Practice sessions indicated that the probability of a left-hander hitting the
target was 0.7, while for right-handers it was 0.9. If an archer hits the target, what is the
probability that the person was right-handed ?

41
A. 50
27
B. 41
3359
C. 9500
37
D. 100
7
E. 25

SOLUTION:

Out of 20 candidates 8 were left handed


Right handed candidates = 20 - 8 = 12
8 2
P(that the person is Left handed) = 20
= 5
12 3
P(that the person is Right handed) = 20 = 5

P(Left hander hitting the target) = 0.7


P(Right hander hitting the target) = 0.9

2
Probability that the archer hits the target was left handed= 0.7× 5
= 0.28
3
Probability that the archer hits the target was right handed= 0.9× 5
= 0.54
Probability of hitting the target = 0.28 + 0.54 = 0.82
0.54 27
Required Probability = 0.82
= 41

ANSWER B
Inequality Modulus Extra Question Solutions

Inequalities

1. If x2 − 9 < 0, which of the following is true?

A. x < −3
B. x > 3
C. x > 9
D. x < −3 or x > 3
E. −3 < x < 3

SOLUTION:

𝑥2 - 9 < 0
x2 - 32 < 0
(x + 3)(x - 3) < 0
-3 < x < 3

ANSWER E
2. If (|x| - 2)(x + 5) < 0, then which of the following must be true?

A. x > 2
B. x < 2
C. -2 < x < 2
D. -5 < x < 2
E. x < -5

SOLUTION:
(|x| − 2)(x + 5) < 0

Case 1 : x is positive or 0.
(x − 2)(x + 5) < 0
=> -5 < x < 2
As x ≥ 0 , final range will be 2 > x ≥ 0

Case 2 : x is negative.
(-x − 2)(x + 5) < 0
(x + 2)(x + 5) > 0
=> x > -2 OR x < -5
As x < 0 , final range will be -2 < x < 0 OR x < -5

Final Answer : -2 < x < 2 OR x < -5


Choice B covers both the ranges of x.

ANSWER B
3. Which of the following is a value of x for which x−9 – x−7 > 0?

A. -2
B. -1
1
C. − 2
D. 1
E. 2

SOLUTION:
1 1
9 – 7 >0
𝑥 𝑥
7 9
𝑥 −𝑥
16 >0
𝑥
7 2
𝑥 (1 − 𝑥 )
16 >0
𝑥
16
𝑥 can be excluded as it will always be positive.(Here, x ≠ 0)
7
𝑥 can be treated as x.

2
So, we have 𝑥 (1 − 𝑥 ) > 0
2
=> x ( 𝑥 - 1) < 0
=> x (x - 1)(x + 1) < 0
=> x < -1 OR 0 < x < 1

ANSWER A
4.

Which of the following inequalities specifies the shaded region ?

2
A. 𝑥 + 1 < 3
B. |2x - 3| < 5
C. |x + 1| > -1
D. x - 2 < 2
E. |x - 1| < 4

SOLUTION:
Interpretation from graph: −1 < x < 4

2
A. 𝑥 + 1 < 3
=> x2 + 1 < 9 (squaring both sides as both the LHS and RHS are positive)
=> x2 < 8
=> −2√2 < x < 2√2 Does not satisfy the given range, Eliminate

B. |2x - 3| < 5
−5 < 2x − 3 < 5 ..add 3 to all sides
−2 < 2x < 8 ..divide by 2
−1 < x < 4 Does satisfy the given range, Correct

C. |x + 1| > -1
The absolute value i.e. |x + 1| is always more than or equal to zero, thus this inequality holds
for any value of x. Eliminate

D. x - 2 < 2
x<4 Does not satisfy the given range, Eliminate
E. |x - 1| < 4
−4 < x − 1 < 4 ..add 1 to all sides
−3 < x < 5 Does not satisfy the given range, Eliminate

ANSWER B

5. If -13 < 7a + 1 < 29 and 19 < 2 - b < 23, what is the maximum possible integer value of a + b?

A. -23
B. -18
C. -14
D. -13
E. -12

SOLUTION:

Get the range of a and b in simplified form:


-13 < 7a + 1 < 29
-14 < 7a < 28 ..Subtracting 1
-2 < a < 4 ..Dividing by 7

19 < 2 - b < 23
-23 < b - 2 < -19 ..Multiplying by -1
-21 < b < -17 ..Adding 2

-23 < a + b < -13 ..[Adding inequalities 1 and 2]

The maximum integer values of a + b should be less than -13 and that can only be -14.

ANSWER C
6 .If x3 > y2 > z, which of the statements could be true?

I. x < y < z
II. x < z < y
III. y < x < z

A. I only
B. III only
C. I and II only
D. II and III only
E. I, II and III

SOLUTION:
Evaluate the options:

I. x < y < z
Say x = 3, y = 4, z = 5, which satisfy the inequalities x3 > y2 > z i.e. 27 > 16 > 5
Hence, x < y < z could be true.

II. x < z < y


Say x = 3, y = 5, z = 4, which satisfy the inequalities x3 > y2 > z i.e. 27 > 25 > 4
Hence, x < z < y could be true.

III. y < x < z


Say x = 4, y = 3, z = 5, which satisfy the inequalities x3 > y2 > z i.e. 64 > 9 > 5
Hence, y < x < z is true.

ANSWER E
7. If (x + 3)2 = 225, which of the following could be the value of x - 1 ?

A. 13
B. 12
C. -12
D. -16
E. -19

SOLUTION :
(x+3)2 = 225
..[square root on both sides]
|x+3| = 15
x + 3 = 15 or x + 3 = -15
x = 12 or x = -18

Now, x - 1:
(12 - 1) or (-18 - 1)
= 11 or = -19

x = -19

ANSWER E
(𝑥+2)(𝑥+3)
8. How many of the integers that satisfy the inequality (𝑥−2)
≥ 0 are less than 5 ?
A. 1
B. 2
C. 3
D. 4
E. 5

SOLUTION:
Critical points: x = -3, -2, 2

At x = 2, the inequality is not defined.

−3 ≤ x ≤ −2 OR x > 2, satisfy the equation.

Only 4 integers that are less than 5 satisfy given inequality: -3, -2, 3, and 4.

ANSWER D
9. If x < y , 2x = A, and 2y = B, then

A. A = B
B. A < B
C. A > B
D. A < x
E. B < y

SOLUTION:
x<y ..multiply both sides by 2

2x < 2y ..substitute 2x = A and 2y = B

A<B

ANSWER B
(𝑥+𝑘)
10. If x, y, and k are positive and x is less than y, then (𝑦+𝑘)
is

A.1
B.greater than x/y
C.equal to x/y
D.less than x/y
E.less than x/y or greater than x/y, depending on the value of k

SOLUTION :
Given: x < y
𝑥
𝑦
<1

TRY x = 2 , y = 3 AND k = 1
𝑥
𝑦
= 0.66
(𝑥+𝑘)
So , (𝑦+𝑘)
will be 0.75
(𝑥+𝑘) 𝑥
In other words, (𝑦+𝑘)
is greater than 𝑦

ANSWER B
11.
5𝑝 + 3𝑞 ≤ 17,000
7𝑝 + 9𝑞 ≤ 43,000

A manufacturer wants to make p screws and q nails from a shipment of raw material. Material
constraints require p and q to satisfy the inequalities given above. What is the maximum total
number of screws and nails that can be created, given the material constraints?

A. 4,000
B. 5,000
C. 6,000
D. 7,000
E. 8,000­

SOLUTION:
Maximize p and q: where p , q is the number of screws and nails respectively.

5𝑝 + 3𝑞 ≤ 17,000 .. (1)
7𝑝 + 9𝑞 ≤ 43,000 .. (2)

Adding both the inequalities


12𝑝 + 12𝑞 ≤ 60,000
12(𝑝 + 𝑞) ≤ 60,000
(𝑝 + 𝑞) ≤ 5,000

Hence, (𝑝 + 𝑞)Max = 5,000

ANSWER B
12. If x ≥ 8 and y ≤ 3, then it must be true that

A. x + y ≥ 5
B. x + y ≤ 11
C. x − y ≥ 5
D. x − y ≤ 5
E. x − y ≤ 11

SOLUTION:
Approach: Get an inequality in the form of x + y
x≥8 ..Inequality 1

y≤3 ..multiply -1 to set the inequality in the desired form


-y ≥ -3 ..Inequality 2

x-y≥5 ..Adding both the inequalities 1 and 2

ANSWER C
13. If 3x + 7y = 8x − 18y and y > 7, then which of the following must be true?

7
A. x < 5
B. x < 5
C. x > 18
D. x < 35
E. x > 36

SOLUTION :

3x + 7y = 8x - 18y
5x = 25y
𝑥
x = 5y or y = 5

Given, y >7
𝑥
5
>7 ..substitute value of y
x > 35

Be careful in selecting the correct option, x could be 35.00..001 to infinity.


7
A. x < 5
=> x can be equal to 36, inequality fails, Eliminate
B. x < 5 => x can be equal to 36, inequality fails, Eliminate
C. x > 18 => x > 35, Hence this satisfies
D. x < 35 => x can be equal to 35.1, inequality fails, Eliminate
E. x > 36 => x can be equal to 35.1, inequality fails, Eliminate

ANSWER C
14. If 1 < x < 7 and 2 < y < 9, what is the largest integer value of x + y?

A. 4
B. 13
C. 14
D. 15
E. 16

SOLUTION :
Approach: Get the inequalities in the x + y form.

1<x<7 ..(1)
2<y<9 ..(2)

1+2<x+y<7+9 ..Adding inequalities (1) and (2)

3 < x + y < 16

x + y has to be an integer that is less than 16, so x = 15

Here Trap would be Option C, x = 6 and y = 8. However, x and y may not be integers, say x = 6.5
and y = 8.5

ANSWER D
15. If a > b and c < a , which of the following must be true ?

I. a > b > c
II. 2a > b + c
III. a2 > bc

A. I only
B. II only
C. III only
D. II and III only
E. I, II, and III

SOLUTION:
a>b ..(1)
a>c ..(2)

I. a > b > c
Say a = 5, b = 2, and c = 3
a > b =>True
a > c =>True
However, a > b > c, => False
Eliminate

II. 2a > b + c
a+a>b+c ..add inequalities 1 and 2
2a > b + c
TRUE

III. a2 > bc
Say a = -6, b = -7, c = -9
a > b => True
a > c => True
a2 = 36 and bc= 63
Here, a2 < bc
Eliminate
Only statement II is valid.

ANSWER B
16. Which of the following inequalities is true?

1 1
A. 11
< 0.08 < 9
1 1
B. 10 < 0.11 < 8
1 1
C. 7 < 0.17 < 6
1 1
D. 5 < 0.26 < 4
1 1
E. 3 < 0.08 < 2

SOLUTION :
Convert the fractions into corresponding approximate decimal values then compare:
HINT: Fraction values are standard; Will be comparatively easy to approx and eliminate

1 1
A. 11
< 0.08 < 9
=> 0.09 < 0.08 < 0.11, 0.08 is not greater than 0.09, Eliminate

1 1
B. 10
< 0.11 < 8
=> 0.10 < 0.11 < 0.125, TRUE

1 1
C. 7
< 0.17 < 6
=> 0.142 < 0.17 < 0.167 0.167 is not greater than 0.17, Eliminate


1 1
D. 5
< 0.26 < 4
0.20 < 0.26 < 0.25, 0.25 not greater than 0.26, Eliminate

1 1
E. 3 < 0.08 < 2
0.33 < 0.08 < 0.5, 0.08 not greater than 0.33, Eliminate

ANSWER B
17. For real numbers x and y,

−5 ≤ x − y ≤ 7
−9 ≤ x + y ≤ 6

What is the greatest possible value of x2−y2 ?

A. 35
B. 42
C. 45
D. 54
E. 63

SOLUTION :
−5 ≤ x − y ≤ 7
−9 ≤ x + y ≤ 6

Case 1: x + y = -9 ; x - y = -5
(x + y)(x - y)
= (-9) x (-5)
= 45

Case 2: x + y = 7 ; x - y = 6
(x + y)(x - y)
=7x6
= 42

Maximum value of x2 − y2 = 45

ANSWER C
18. ­At a certain store, all notebooks have the same price and all pencils have the same price.
The price of four notebooks and three pencils is more than $12 and less than $13. The price of
two notebooks and five pencils is more than $8 and less than $9. If a notebook costs x more
than a pencil, which of the following could be the value of x?

A. $0.60
B. $1.05
C. $1.30
D. $2.20
E. $2.70

SOLUTION:
Say, Price of a Pencil => p, Price of a Notebook => n
A notebook costs x more than a pencil i.e. n = x + p

The price of four notebooks and three pencils is more than $12 and less than $13
12 < 4n + 3p < 13 ..(1)

The price of two notebooks and five pencils is more than $8 and less than $9
8 < 2n + 5p < 9 ..(2)

12 < 4(x+p) + 3p < 13 [Substituting value of n in (1)]


12 < 7p + 4x < 13 .. (3)

8 < 2( x + p) + 5p < 9 [Substituting value of n in (1)]


8 < 7p + 2x < 9
Multiplying by -1 , we get
-9 < -7p - 2x < -8 .. (4)

Adding (3) and (4) , we get


3 < 2x < 5
1.5 < x < 2.5 ..[Divide by 2]

Cost of pencil x = $2.20

ANSWER D
3 5
19. Which of the following inequalities, if true, is sufficient alone to show that 𝑥 < 𝑥 ?

A. −1< x < 0
B. x > 1
C. |x|< −1
D. |x|> 1
E. x < −1

SOLUTION:
3 5
𝑥< 𝑥
Raising the inequality to the power of 15 won’t change the inequality
3 5
( 𝑥 )15<( 𝑥)15
x5<x3
x5-x3< 0
x3(x2-1) < 0
3
x (x+1) (x-1) < 0 [ 𝑥 can be considered as x ]
The critical points are -1,0 and 1

Thus, x must be either less than -1 or between 0 and 1.

3 5
So, x <-1 is sufficient to show that 𝑥 < 𝑥

ANSWER E
20. If x < y < 0, which of the following inequalities must be true ?

A. y + 1 < x
B. y - 1 < x
C. xy2 < x
D. xy < y2
E. xy < x2

SOLUTION:
Both x and y are negative and x is less than y
Evaluate the options:

A. y + 1 < x
Say x = -2 and y = -1
-1 + 1 < -2
0 < -2, Not true, Eliminate

B. y - 1 < x
Say x = -2 and y = -1
-1 - 1 < -2
-2 < -2, Not true, Eliminate

C. xy2 < x
x - xy2 > 0
x( 1 − y2) > 0
As x < 0 then ( 1 − y2) < 0, ..to keep x( 1 − y2) positive
( 1 − y2) < 0
y2 > 1 Not true, as we don’t know the value of y. y could be -0.5 / -2
Eliminate

D. xy < y2
y2 − xy > 0.
y( y − x ) > 0
As y < 0, then ( y − x ) < 0
( y − x ) < 0 => y < x Not true as : x < y < 0
E. xy < x2
x2 − xy > 0
x( x − y ) > 0
As x < 0, then ( x − y ) < 0
( x − y ) < 0 => x < y True, given: x < y < 0

ANSWER E
Modulus

1.

­The number line below is equivalent to which of the following inequalities?

A. |x| ≤ 5
B. |x| ≥ 6
C. |3x – 1| ≤ 14
D. |4x + 2| ≤ 22
E. |10x – 6| ≤ 54­­

SOLUTION:
From the number line: -6 ≤ x ≤ 5
Let us solve the answer choices and find the range of x
A. |x|≤ 5: -5 ≤ x ≤ 5 ; Eliminate
B. |x| ≥ 6: x ≤ - 6 or x ≥6 ; Eliminate
C. |3x – 1| ≤ 14
=> -14 ≤ 3x - 1 ≤ 14 [Adding 1 to all the sides]
=> -13 ≤ 3x ≤ 15 [Divide the inequality by 3]
13
=> - 3
≤ x ≤ 5 ; Eliminate

D. |4x + 2| ≤ 22
=> -22 ≤ 4x + 2 ≤ 22 [Subtract 2 from all the sides]
=> -24 ≤ 4x ≤ 20 [Divide the inequality by 4]
=> -6 ≤ x ≤ 5 ; TRUE

ANSWER D
2. When |x|>1 and |x-3|>1, which of the following is contained in the scope of x ?

A. -0.5
B. 0
C. 0.5
D. 1.5
E. 2.5

SOLUTION:
We need to find which of the choices fit in the range of x
|x|>1
=> x < -1 OR x > 1

|x-3|>1
=> x - 3 > 1 OR x - 3 < -1
=> x > 4 OR x < 2

So , the common range will be x < -1 OR 1 < x < 2 OR x > 4.


Only 1.5 falls in the above range

ANSWER D
3. Which of the following inequalities represents the entire solution set for x if |x| > x ?

A. x ≥ 0
B. x > 0
C. x ≤ 0
D. x < 0
E. x < −1

SOLUTION:
Since |x|>x,
It is only possible when x < 0

ANSWER D
4. If a + b > 0 and ab < 0, which of the following must be true ?

I. a < 0
II. b > 0
III. |b|>|a|

A. I only
B. I and II only
C. I and III only
D. I, II and III
E. None of these

SOLUTION :
ab < 0
Let consider a as positive, Positive(+,-, fraction) > 0, so ‘a’ can’t be positive
a = negative value

Next, since a + b > 0 and a < 0, then b must be positive, and the absolute value(modulus) of b
should be greater than a.

Evaluate options:
I. a < 0 a has to be negative, True
II. b > 0 b has to be positive, True
III. |b|>|a| a + b > 0 and a < 0, so |b|>|a| is valid, True

ANSWER D
5. Find the number of real solutions of the equation x2 - 2|x| - 8 = 0.

A. 0
B. 1
C. 2
D. 3
E. 4

SOLUTION :
Case 1: x ≥ 0
x2-2x-8 = 0
x2- 4x + 2x-8 = 0
x(x-4) + 2(x-4) =0
(x+2) (x-4)=0
x = -2 or 4
x = 4 is the valid solution

Case 2: x< 0
x2 + 2x - 8 =0
x2 + 4x - 2x - 8 = 0
x(x+4)- 2(x+4) = 0
(x-2) (x+4) = 0
x = 2 or -4
x= -4 is the valid solution
So, There are only two valid solutions x = -4 and 4

ANSWER C
6. If x > 0 and 3x + 4|y| = 33, then how many integer pairs of (x, y) are there ?

A. 6
B. 5
C. 4
D. 3
E. 2

SOLUTION :
Given x > 0
3x + 4|y|=33
33−3𝑥
|y|= 4
3(11−𝑥)
|y|= 4

Three values of x satisfies the above equation:


x = 3,7,11
Putting the values in the above equation, 5 values of y are possible: -6, -3 , 0 , 3 and 6.
Integer pairs are: (3,-6), (3,6) ,(7,-3), (7,3) , (11,0)

ANSWER B
7. If |x| + |y| = – x – y and xy does not equal 0, which of the following must be true ?

A. x+y>0
B. x+y<0
C. x–y>0
D. x–y<0
E. x2 – y2 > 0

SOLUTION :
If |x| + |y| = – x – y ;
Then we can say that x is negative and y is negative as the mods open with a negative sign.
So x < 0 and y < 0.
Hence , x + y < 0.

ANSWER B
8. If |12x−5|>|7−6x| , which of the following CANNOT be the product of two possible values of
x?

A. -12
B. -7/5
C. -2/9
D. 4/9
E. 17

SOLUTION:
Squaring both sides, we get
2 2
144𝑥 - 120x + 25 > 49 - 84x + 36𝑥
2
108𝑥 - 36x - 24 > 0
2
9𝑥 - 3x - 2 > 0
(3x + 1)(3x - 2) > 0
x > 2/3 OR x < -1/3

It will not be possible to make the product of the two values as -2/9 from the given range. All
the other option choices are possible. (CHECK!)

ANSWER C
9. If y = |2 + x| - |2 – x| and |2x – 15| < 2, how many integer values can y take?

A. 0
B. 1
C. 2
D. 4
E. Cannot be determined

SOLUTION :
|2x – 15| < 2
-2 < (2x-15) < 2 [Add 15 to all the sides]
13 < 2x < 17 [Divide by 2]
=> 6.5 < x < 8.5

y =|2+x|- |2-x|
Now, |2+x| will always be positive for the range of x and |2-x| will be negative as x > 2
So, y = (2+x) - { - (2-x) }
y=2+x+2-x
y=4

So, y can take 1 integer value.

ANSWER B
10. Which of the following inequalities is equal to |2x-|x||<6?

A. -2 < x < 0
B. 0 < x < 6
C. -2 < x < 6
D. -2 < x < 2
E. -6 < x < 6

SOLUTION:

Case 1: x ≥ 0
|2x-|x||<6 ⬄ | 2x – x | < 6 ⬄ |x| < 6 ⬄ -6 < x < 6.
From the assumption of the case, x ≥ 0, we have 0 ≤ x < 6.

Case 2: x < 0
|2x-|x||<6 ⬄ | 2x + x | < 6 ⬄ |3x| < 6 ⬄ |x| < 2 ⬄ -2< x < 2.
From the assumption of the case, x < 0, we have -2 < x < 0 .

Combining, we have -2 < x < 6.

ANSWER C
11. The equation |2x - 3| = x - 5 has how many solutions for x ?

A. 0
B. 1
C. 2
D. 3
E. 4­

SOLUTION :
Case 1: 2x - 3 is positive or 0.
2x - 3 = x - 5
x = -2
Invalid solution as it won’t satisfy the equation. (CHECK !)

Case 2: 2x - 3 is negative
-(2x - 3) = x - 5
-2x + 3 = x - 5
3x = 8
8
x= 3
Invalid solution as it won’t satisfy the equation. (CHECK !)

ANSWER A
12. The sum of all solutions for x in the equation x2 – 8x + 21 = |x – 4|+ 5 is equal to:

A. –7
B. 7
C. 10
D. 12
E. 14

SOLUTION:
Case 1: x - 4 ≥ 0
x2– 8x + 21 = |x–4|+5
x2– 8x + 21 = x-4 + 5
x2 – 9x + 20 = 0
(x - 5) (x - 4) = 0
x = 5 or 4

Case 2: x - 4 < 0
x2– 8x + 21 = |x–4|+ 5
x2– 8x + 21 = -(x-4) + 5
x2 – 8x + 21 = -x + 4 + 5
x2 –7x + 12 = 0
( x- 4) ( x -3 ) = 0
x = 4(REJECT) or 3

So, Sum of different solutions = 5 + 4 + 3 = 12

ANSWER D
13. If x and y are integers and |x - y| = 12, what is the minimum possible value of xy ?

A. -12
B. -18
C. -24
D. -36
E. -48

SOLUTION :
We have |x - y| = 12
To get the minimum possible value of xy, you need a negative sign with the maximum possible
absolute value of the product. So exactly one of x and y should be negative. Let's assume y is
negative

If y is negative, x - y becomes the sum of two positive integers which is given to be 12. We need
the maximum absolute value of xy. This will happen when x and y have the same absolute value.
So both should have an absolute value of 6.
So minimum product is 6×(-6) = -36

ANSWER D
14.

The absolute value of (x + a) will be the greatest if x is the coordinate of which of the five points
P, Q, R, S and T on the number line above?

A. P
B. Q
C. R
D. S
E. T

SOLUTION:

We need to maximize the value of |x+a|.

|x+a| represents the distance of x from -a, i.e. | x - (-a)| = |x + a|. Point T is farthest from -a.

Hence |x+a| will be maximum at T.

ANSWER E
15. If -8 ≤ a ≤ 1 and -5 ≤ b ≤ 10. Find the sum of minimum and maximum value of a - |b|.

A. -19
B. -23
C. -18
D. -22
E. -17

SOLUTION:

Since |b| is nonnegative, maximum value of the expression will occur at b = 0


The maximum value of a-|b| is 1 - 0 = 1.

The minimum value of a -|b| is -8 - 10 = -18.

Thus, the Sum of the values is -18 + 1 = -17.

ANSWER E
16. Which of the following inequalities has a solution set that, when graphed on a number line,
is a single, finite line segment ?

A. x ≥ 4
B. x2 ≥ 4
C. x3≥ 64
D. |x|≥ 4
E. |x|≤ 4

SOLUTION :

Option A will have an infinite line segment.


Option B will have two infinite line segments.
Option C will have an infinite line segment.
Option D will have two infinite line segments.
Option E : |x|≤ 4 has the solution set -4 ≤ x ≤ 4
When graphed, it is a single finite line segment.

ANSWER E
2
17. If (3𝑥 + 3) = 900, which of the following could be the value of x -1 ?

A. -12
B. -11
C. -8
D. 9
E. 11

SOLUTION :
Let 3x + 3 = a
a2 = 900
So, a can be either 30 or -30

CASE 1 : a = 30
3x + 3 = 30
3x = 27
=> x = 9

CASE 2 : a = -30
3x + 3 = -30
3x = -33
=> x = -11

So, x can be 9 or -11


Hence, x - 1 = 8 or -12

ANSWER A
2
18. What is the product of all the solutions of (𝑥 + 2) = |x + 2| ?

A. -6
B. -2
C. 2
D. 6
E. 12

SOLUTION :
Let a = x + 2

2
(𝑎) = |a|
Case 1 : a is positive or 0.
2
(𝑎) = a
2
𝑎 -a=0
a(a - 1) = 0
a = 0 OR 1
x + 2 = 0 OR 1
=> x = -2 OR -1 [Both Solutions are Valid]

Case 2 : a is negative.
2
(𝑎) = - a
2
𝑎 +a=0
a(a + 1) = 0
a = 0 OR -1. [0 is an INVALID SOLUTION here]
a = -1
x + 2 = -1
=> x = -3
Product of the solutions = -6.

ANSWER A
19. If |a+b|=|a-b|, then a×b must be equal to

A. 1
B. -1
C. 0
D. 2
E. -2

SOLUTION:
If |a + b|=|a - b| ,
then a + b = a - b OR a + b = - (a - b)
=> b = 0 OR a = 0

We can conclude that a×b must be zero.


ANSWER C
20. If |z|/w = 1, which of the following must be true ?

A. z = -w
B. z = w
C. z2 = w2
D. z2 = w3
E. z3 = w3

SOLUTION :
|z|= w
Now, z = -w or w

However, regardless of whether z = w or z = -w, when we square both sides of either equation,
we have:

z2 = w2

ANSWER C

You might also like